CARS Practice Exam

¡Supera tus tareas y exámenes ahora con Quizwiz!

Given the information in the passage, one would most reasonably assume that Surrealism tries to: A. put hallucinations and dreamlike visions on canvas. B. encourage artists to act in childlike ways in public. C. reflect the world as clearly as possible. D. give an outlet for artists to relieve stress caused by their creativity.

Solution: The correct answer is A. A) This Comprehension question asks you to draw conclusions from explicit statements in the passage. The author writes, "Dali seized on the guiding principles of Surrealism and gave them their most extreme interpretation; he transformed its interest in the revelations of the unconscious and psychopathological states into a way of living and thinking" (paragraph 4). The implication here is that the Surrealists' interest in the unconscious and in mental states was generally confined to their art (to the canvas), rather than extended to public performance, as it was in Dali's case. B) The passage actually suggests that the Surrealists were concerned with the reputation of their movement and began to worry about Dali's effect on that reputation; childlike behavior in public, then, was not something the passage suggests the Surrealists advocated. C) This is explicitly contradicted in the passage, with its references to the Surrealists' focus on the unconscious and on psychopathology; this would suggest that they did not believe exclusively in the dictates of direct, realistic representation. D) There is no support for this claim in the passage.

According to the passage, recent immigrants are particularly likely to experience a loss of socioeconomic status if: A. qualification policies prevent them from practicing their profession. B. their limited education forces them to work in low-paying fields. C. they are too demoralized to pursue opportunities for advancement. D. language barriers reduce their usefulness to potential employers.

Solution: The correct answer is A. A) This Comprehension question tests your understanding of a point made explicitly in the passage. Paragraph 4 describes the process by which recent immigrants may not be able to practice in their new countries the professions for which they were originally trained, and the text attributes this to the standards or policies that are used to measure qualification. B) The passage explicitly describes the situations of highly educated professionals who are not able to practice their professions in the new countries in which they live, so this option is clearly not supported by the passage. C) Although the passage describes mental health problems that may confront recent immigrants, it does not suggest that these problems are particularly likely to result in a loss of socioeconomic status; to the contrary, the passage contends that mental health problems are particularly prevalent among people with lower socioeconomic status. D) The passage does not focus on the effects of language barriers.

The passage implies that "some observers" (first paragraph) believed that: A. there was no real increase in heart disease after World War II. B. new diagnostic techniques were mistakenly identifying heart disease. C. the increase in death rates after World War II had little to do with the increase in heart disease. D. the increase in heart disease after World War II was somehow related to the decline in communicable disease.

Solution: The correct answer is A. A) This Comprehension question tests your understanding of content that is implied, but not explicit, in the passage. According to paragraph 1, "Some observers cautioned that the apparent increase might be the result of diagnostic advances." The author's use of "apparent" suggests that the increase may not be a "real" one, and the reference to "diagnostic advances" posits an alternative explanation. B) The passage suggests that the apparent increases might be a result of better diagnoses, but not that the diagnoses themselves were "mistaken." C) The death rates referred to in the passage are not general death rates after World War II, but death rates due to chronic heart disease, so this option is explicitly contradicted in the passage. D) The decline in communicable disease is mentioned in the same paragraph as the "apparent" increase in heart disease, but the passage does not link communicable disease and heart disease causally. It only mentions that public medicine began paying more attention to heart disease once communicable diseases had declined.

According to the passage, Britain's complacency toward the colonies resulted in a: A. lack of knowledge about colonial life. B. buildup of American forces. C. rivalry between British political factions. D. base of power for the British Empire.

Solution: The correct answer is A. A) This Comprehension question tests your understanding of information that is presented in the passage with more complex language than is contained in the passage. Paragraph 5 maintains that the "danger in complacency is that it causes the possessor to ignore as unimportant the local factors and conditions that govern other people with whom it deals. Britons faced with the American Revolution were not interested in Americans or in their magnificent continent." The paragraph goes on to detail the British ministers' lack of interest in—and thus lack of knowledge of—life in the colonies. This option clearly specifies what the passage describes as the effects of complacency. B) There is no implication that British complacency itself led to the buildup of American forces. C) The introductory paragraphs refer to "disunity" within the British military, but they do not link that disunity to "complacency," which is discussed later in the passage. D) The passage suggests that the British assumed that their Empire could not be challenged by the colonies, not that their complacency actually created a base of power.

The author's arguments about government imply that he or she would be likely to oppose which of the following? A. Legislation limiting the number of hours one can work per week B. A decrease in governmental subsidies to colleges and universities C. Legislation loosening regulations on lobbyists D. A decrease in governmental oversight of school lunches

Solution: The correct answer is A. A) This Reasoning Beyond the Text question asks you to apply passage claims to a situation that is not discussed in the passage. In this case, to get the question correct, you must identify the author's arguments about the government, then test these against each scenario presented, in order to determine which the author is most likely to oppose. Paragraphs 6 and 7 indicate that the author is against what he or she sees as governmental "paternalism," and this legislation would be an example of that—"protecting" people from "overwork," rather than letting them choose how much to work. B) There is nothing in the passage to suggest that the author is for (or against) subsidies. C) The passage mentions lobbying (paragraph 6), but does not indicate that the author is in favor of it. D) As the author doubts that government intervention in food policy has real public health benefits (paragraphs 6 and 7 and throughout), it is likely that the author would support—not oppose—a decrease in governmental oversight of school lunches.

The expression "This is driving me around the bend" would best support a metaphor that compares madness to: A. a location. B. a vehicle. C. a road. D. a force.

Solution: The correct answer is A. A) This Reasoning Beyond the Text question asks you to consider a metaphor that was not presented in the passage in terms of the arguments made in the passage. (Here, the authors claim metaphors shape our very thought processes are relevant.) This metaphor suggests that madness takes the speaker from one place to another—madness is conceived of as a place "around the bend," somewhere incompatible with sanity, and hence, as a location. B) While the metaphor includes a reference to driving, it does not compare madness to the actual vehicle that drives the speaker "around the bend." C) While the metaphor suggests that the speaker travels on a road, in order to arrive "around the bend," it does not compare madness to the road itself. D) The metaphor suggests that something is pushing (or driving) the speaker "around the bend," and that something could be conceived of as a "force." It is not the force itself that represents madness in the metaphor, however; it is being "around the bend" that represents madness.

Which of the following discoveries would most strengthen the hypothesis that Folsom hunters killed the bison they ate? A. Bone breaks consistent with the shapes of the Folsom hunters' pointed tools B. No evidence of an alternative animal food source C. Bison bones at a Folsom site D. Similar accumulation of bison bones at many Folsom sites

Solution: The correct answer is A. A) This Reasoning Beyond the Text question asks you to consider the effect on the passage of a piece of evidence that was not presented in the passage. The passage makes clear that the presence of tools at the sites, in conjunction with animal bones, is what led archaeologists to the conclusion that the Folsom Indians were hunters. If the bone breaks found in the bison were consistent with the shape of the Folsom hunters' tools, then this would provide further evidence that the tools were used to kill the bison. B) This might suggest the importance of bison as a food source for the Folsom hunters, but it doesn't provide any specific evidence that the hunters killed the bison that they ate. C) This provides evidence only that the Folsom hunters may have eaten the bison, not that they necessarily killed them. (Another predator could have killed the bison, for example, and the Folsom hunters could have eaten the remains.) D) This would provide evidence that bison was a food source for different bands of Folsom hunters, but it does not provide any evidence that the Folsom hunters killed the bison that they ate.

Which of the following findings would most weaken the claim that the use of symbolic imagery is unique to humans? A. Chimpanzees are capable of learning at least some sign language. B. Certain species of birds are able to migrate great distances by instinct alone. C. Human beings have larger frontal lobes than do other animals. D. Some animals have brains that are larger than human brains.

Solution: The correct answer is A. A) This Reasoning Beyond the Text question asks you to consider the effects on the passage of a piece of information not in the passage. The author argues, "Of all the distinctions between man and animal, the characteristic gift which makes us human is the power to work with symbolic images: the gift of imagination." The passage treats language as symbolic imagery (as in any language, written, spoken, or sign, one thing stands for another). Thus, if chimpanzees are indeed capable of learning at least some sign language, then they are capable of using symbolic imagery, and the author's claim is weakened. B) This does not weaken the author's claim, as it does not relate to the use of symbolic imagery, but to instinctive behaviors. C) If anything, this would strengthen, rather than weaken, the author's claim, since the passage contends that frontal lobes are "probably the seat of our images." D) The passage claim that the use of symbolic imagery is unique to humans does not rest on any claims about the size of the human brain.

The ideas discussed in this passage would likely be of most use to: A. an ambassador to a different culture. B. a senator engaged in a serious debate. C. a financial analyst for a large corporation. D. a general preparing for battle.

Solution: The correct answer is A. A) This Reasoning Beyond the Text question asks you to think about how you would apply the arguments in the passage to a circumstance outside of the passage. The author contends that metaphors profoundly shape human thought. He/she then argues that the way argument is conceived of in U.S. culture (as "war") means that people in the U.S. think very differently about argument than would those in a culture where argument was conceived of as a "dance." This argument, then, would be useful to those who work with other cultures besides their own, and needed insight into the cultural dimensions of cognitive processes. B) It is not clear how understanding the importance of metaphor to thought, or even understanding the way metaphors of argument may structure one's understanding of argument itself, would be of specific use in this scenario. C) The arguments presented in the passage do not have any particular application here. D) The passage talks about the military metaphors that people in the U.S. use to talk and think about argument; this does not necessarily have any application for someone who is preparing for a real, rather than a metaphorical, war.

An experiment found that dogs can remember a new signal for only five minutes, whereas six-year-old children can remember the same signal much longer. Based on the information in the passage, this finding is probably explained by the fact that: A. a human being possesses a larger store of symbolic images than a dog possesses. B. the human brain evolved more quickly than the brain of a dog. C. the children were probably much older than the dogs. D. most dogs are color-blind.

Solution: The correct answer is A. A) This Reasoning Beyond the Text question poses a hypothetical scenario not contained in the passage and then asks you to analyze it based on passage information. Paragraphs 4 and 5 contend that the symbolic images that the human brain creates "play out for us events which are not present to our senses, and thereby guard the past and create the future. By contrast, the lack of symbolic ideas, or their rudimentary poverty, cuts off an animal from the past and the future alike, and imprisons him in the present." Passage information would likely explain the child's longer memory of the signal in these terms and suggest that the longer memory is a result of symbolic thinking. B) The passage discusses the differences between the human brain and the animal brain, but does not say anything about the speed of evolution. C) The passage does not specifically link symbolic thinking (and thus memory) to age, so this point would not explain the scenario. D) This is irrelevant to the scenario, as it does not discuss a visual image.

A new U.S. citizen who was trained as an engineer is unable to find employment in this field. What advice would the author be most likely to give this person? A. Consider a new occupation that offers other advantages. B. Look for work that offers the respect due to an engineer. C. Try to find work in a field that is related to engineering. D. Conceal your background when applying for entry-level jobs.

Solution: The correct answer is A. A) This Reasoning Beyond the Text question presents a hypothetical scenario not discussed in the passage and asks you to use passage information to determine the author's most likely approach to the scenario. The passage discusses those people who must find new lines of work when they immigrate because they are unable to gain accreditation in their previous fields. The author suggests that in some cases, these new lines of work present unexpected advantages (paragraph 5). Thus, the author would be most likely to suggest a strategy like the one described in this option. B) The passage depicts "flexible responses" to a situation like this as both necessary and sometimes advantageous; the author would be unlikely to recommend this level of rigidity as a response. C) The passage does not describe this as a strategy that people are able to employ or, for that matter, that they should try to employ. D) The passage does not suggest that high levels of training or education disadvantage those who are seeking entry-level jobs.

The author mentions "easily digestible carbohydrates" (paragraph 5) most likely in order to: A. imply that the 1977 guidelines may actually have harmed public health. B. suggest that nutritional thinking changes over time. C. explain one cause of the current obesity crisis in the U.S. D. criticize the make-up of the dominant diet in the U.S.

Solution: The correct answer is A. A) This Reasoning Within the Text question asks you to determine how the author uses a piece of evidence he presents in the passage. Since the guidelines "advised raising carbohydrate intake" (paragraph 3), the author mentions that "easily digestible carbohydrates . . . increase inflammations that can cause heart attacks" (paragraph 5), most likely in an effort to suggest that those guidelines may actually be dangerous. B) This would be correct if the reference to the 1977 guidelines suggested that they were connected to nutritional thinking or science at the time, but that they were not is exactly one of the author's criticisms. C) The author does not mention the current obesity crisis. D) Although the author does mention the "American diet" (paragraph 2), he does not do so critically.

Which of the following views is most contrary to the author's opinion regarding the largely unedited sections of music retained in Callas performances? A. They are considered beautiful by most listeners. B. They were left unedited to highlight Callas's voice. C. They contain some necessary editing. D. They do not help "keep the action moving."

Solution: The correct answer is A. A) This Reasoning Within the Text question asks you to determine the author's opinion about an element of the passage, even when it is not stated explicitly. The author comments that "inexplicably, long and dreary sections of music were retained in Callas performances largely untouched." If these "largely unedited sections" are dull and their retention is "inexplicable," then the author clearly does not presume that "most listeners" find them "beautiful." B) As the author contends that the retention of these "largely unedited" sections is "inexplicable," then this particular reason is not necessarily "contrary" to the author's opinion. C) The author suggests that these sections were "inexplicably" left intact, and laments some of the "long and dreary" sections that are retained. The author does not consider that they contain much editing at all, but certainly not that they contain unnecessary editing, which would be a point most contrary to this one, as the question asks. Option A most clearly presents a point contrary to the author's main point about the section. D) This is consistent with—not contrary to—the author's point of view on the unedited sections (which he or she calls "long and dreary").

Which of the following conclusions would be most in accord with a theme of the passage? A. Politics often detracts from a war effort. B. Settlement by treaty is the only option in a colonial war. C. A country's leader should decide if war is justified and then delegate strategy to the military. D. Powerful countries throughout the ages have promoted peace through compromise.

Solution: The correct answer is A. A) This Reasoning Within the Text question asks you to extrapolate from the passage to determine what kind of conclusion would be consistent with one theme of the passage. The final paragraph locates the British preoccupation with politics as a problem for their war effort: "Politics as much as anything defeated the British in the American war. The British have always been obsessed with politics, not so much in terms of opposing systems of belief as in terms of who's in and who's out . . . 'So violent was the spirit of party and faction' in the fleet 'as almost to extinguish every patriotic sentiment.'" This last point suggests that an excessive, self-interested concern with politics can undermine more collective goals. You can readily move from this point to the conclusion stated here. B) The passage does not discuss treaties per se, or suggest that these would have been the only option for the British. C) The passage suggests only that the British lacked strategy and sometimes deferred important decisions. It does not suggest that any particular division of power in the war effort was a problem. D) The passage does not raise the question of a possible compromise that the British could or should have pursued.

How would the author be most likely to interpret the finding that the assertiveness of immigrants and their eventual incomes are positively correlated? A. Stress level is not the only factor in immigrant adjustment. B. Lower-income immigrants may be less likely to join labor unions. C. The ability of immigrants to communicate determines the prestige of their jobs. D. Employers assume that immigrants with this trait are high in status.

Solution: The correct answer is A. A) This Reasoning Within the Text question asks you to place the finding described in the question in the context of the author's argument and logic. The argument emphasizes the kinds of stress that can be experienced as a result of immigration and reports that this stress is highest for immigrants with the least status. A finding that introduces another factor into the analysis of immigrant adjustment and, in this case, economic success would likely be considered by the author as just that—another factor in addition to the ones that the argument addresses. B) Labor unions are not discussed in the passage, and there is no implication that lower-income immigrants would be more likely to form collective enterprises, in any case. C) This point is not directly related to "assertiveness" and does not seem consistent with the author's logic. D) There is no suggestion in the passage that the status that employers attribute to immigrants would play a role in determining their incomes, so whether or not employers make this assumption about immigrants would not be relevant to the immigrants' eventual incomes.

According to the passage, bands of Paleoindians did not trade with one another. What is the evidence for this statement? A. Tools of a band came only from local resources. B. Tool shapes were unique to each band. C. Food sources were unique to each band. D. Each band had its unique language and customs.

Solution: The correct answer is A. A) This Reasoning Within the Text question tests your understanding of the way in which the passage author uses evidence and reaches conclusions. The author explains that the Folsom sites contain tools that were made from stone and other resources that could be gathered from within 50 miles of the sites (paragraph 3). The author thus concludes that bands of Paleoindians were not trading with distant bands; otherwise, presumably, we would find stone from farther away involved in the production of tools. B) The author does not emphasize differences in tool shape from band to band; rather, he or she emphasizes the different, local materials that each band used to make tools as evidence for the lack of trading among the different bands. C) The passage suggests that the bands ate bison in common, so this does not provide evidence for the lack of trade. D) This is not evidence for a lack of trade, which concerns material objects, rather than culture. In any case, the author does not appeal to this evidence to support his or her conclusion.

Implicit in the use of the phrase "betrayal by biography" (paragraph 4) is the idea that biography: A. often explores the writer's personal life against the writer's wishes. B. reveals the true intentions writers had for their works. C. offers no rewards to the interested reader. D. is only worthwhile when it glorifies its subject.

Solution: The correct answer is A. A) This is a Comprehension question that asks you to understand the meaning of a phrase in the passage by considering passage context. The answer is A. When the author describes the possibility that authors will suffer "betrayal by biography," she elaborates on the plight of the famous author: "After all, you belong to the world, not to yourself." This quotation, together with the passage claim that aging authors increasingly fear being judged by a "foreign code of conscience," supports the idea that "betrayal by biography" implies that biography is something inflicted on the author against his wishes. B) The passage author explicitly argues that readers should study an author's works, not his life (especially in paragraphs 5-7), so this option is at odds with the passage. C) This option is not implied by "betrayal by biography," so it is not supported by the passage. D) This option is never suggested by the author, who is not interested in "glorification," so it is not supported by the passage.

Based on information in the passage, which of the following would be the most likely reason behind the author's suggested academic reforms? A. To pay more careful attention to the relationship between humans and the world outside the classroom B. To encourage students to attend colleges and universities in their home states C. To replace a cosmopolitan university education with one that emphasizes the local D. To give a voice to the particular at the expense of the voice of the abstract

Solution: The correct answer is A. A) This is a Comprehension question, because it asks you to recognize the author's purpose in the passage. The answer is A, because the author says that to do anything less (than the suggested academic reforms) is to do a disservice to the future of human culture "in an age when humanity's relationship to nature is so in need of careful, farsighted attention" (paragraph 6). Other parts of this paragraph, as well as the preceding one, emphasize the need for attention to the world outside the classroom. The author mentions, for example, "watersheds, growing seasons" and the importance of "transforming the world immediately outside the classroom into a laboratory." B) According to paragraph 2, students already attend colleges and universities in their home states. C) The author does not advocate replacement, but addition: "academics ought to work to acquire a kind of dual citizenship - in the world of ideals and scholarship, yes, but also in the very real world of watersheds. . ." (paragraph 6). D) This is incorrect for reasons similar to the ones that apply to option C; the author does not want to do away with the "abstract," or theoretical; rather, she wants to correct what she sees as the over-emphasis on abstraction by drawing new attention to the particular and the local.

The passage assertion that a dead poet is "owned without opposition" (paragraph 3) probably means that: A. the poet is no longer able to challenge the public's response to the poet's life and work. B. other writers will no longer oppose the poet's views. C. readers will no longer intrude on the poet's private life. D. the poet's admirers will buy up all the poet's books and relics.

Solution: The correct answer is A. A) This is a Comprehension question, because it requires you to understand the meaning of a phrase in the context of the passage. The answer is A, because, before claiming that a dead poet is "owned without opposition," the author writes, "if you [have] the misfortune to be dead as well as famous, matters are less under your control . . . The dead poet has been ingested by his readers; from now on he will be read, admired, and loved in a different way." "Ingested by his readers" clearly implies that readers have taken over the subjectivity of the dead author. The dead author is unable to determine how he is perceived and represented by fans. B) This is a misreading of the quotation, which does not address an author's treatment by other writers, and there is no implication that those other writers "own" him. C) The quotation implies that readers have more, not less, capacity to intrude on a writer when he is dead. D) The quotation implies that readers may gain a figurative kind of "ownership" over the dead writer, who can become (in the readers' minds) what the readers imagine or desire him to be—the dead author can no longer respond to or contest this version of himself. This option involves a literal ownership of objects, which is not what the quotation implies.

The author's primary purpose is to: A. argue for the value of architectural paint research. B. critique unqualified practitioners of architectural paint research. C. provide advice to those who commission architectural paint research. D. present findings of architectural paint research conducted on 17th century buildings.

Solution: The correct answer is A. A) This is a Comprehension question, because it tests your understanding of the central argument, or purpose, of the passage. The answer is A, because the passage describes the valuable information that can be gained by doing this kind of research, including insight into the "development and use of a building, long after" its structural work is finished. While the author expresses some concern about the poor work of some architectural paint researchers, this is more of a caveat; the larger point of the passage is to establish the "potential" of the research "to offer detailed insight into the social as well as the structural and decorative history of a building." B) The author concludes by acknowledging that some people have legitimate concerns about the discipline, as some do not perform or present their research well, but this is not the main point of the passage. C) The passage concludes by referring to problems that may be experienced by some people who commission this research, but this is not the main point of the passage. D) The passage refers to some research done in the seventeenth century, but this is not the main point of the passage.

Assuming the truth of this author's understanding of widespread fantasy violence as an improvement in the human condition (paragraph 5), which behavior pattern would be LEAST consistent with that understanding? Children watching more than average amounts of television violence: A. behave similarly to those who do not. B. exhibit more purpose and competence. C. exhibit greater moral courage in confronting wrongs. D. engage in fewer criminal activities as adults.

Solution: The correct answer is A. A) This is a Foundations of Comprehension question because it asks you to apply a central passage argument to a correlation between child behavior and watching violence. The answer to this question is A because the author implies that children who watch more television violence may exhibit less "powerlessness or anger or frustration or aggression" (paragraph 5), and therefore their aggressive behaviors would likely be less than other children. B) B is incorrect because the author maintains that television violence may allow "well-adjusted human beings [to] vent in a civilized society" (paragraph 5). C) C is incorrect because the author posits that through fantasy violence "the child viewer becomes momentarily omnipotent, transcending all obstacles of an oversized world" (paragraph 2). D) D is incorrect because the author implies that, as some countries with American television have lower rates of violent crime, adult crime may not be linked to childhood watching of television violence (final paragraph).

Why is the San Luis site being investigated urgently? A. Artifacts are few in number. B. Artifacts are being eroded by the wind. C. Bison bones are few in number. D. Excessive rainfall is damaging the site.

Solution: The correct answer is B. A) The passage suggests the opposite, as the wind continuously exposes "new artifacts." B) This Comprehension question assesses understanding of information that is presented explicitly in the passage—in the first sentence of the final paragraph. This question shows that the end of the passage may contain important information, which is helpful to remember when you are looking back at or skimming a passage after a first reading. C) The passage does not imply this. D) While the passage refers to the climate at the time when the Folsom Indians occupied the site, it does not state or imply that "excessive rainfall" was a problem.

Based on the passage, the author's ideal world would best be described as one in which: A. there is a blend of randomness and predictability. B. the truly unpredictable could be easily understood. C. the semblance of order is largely a pleasant illusion. D. the fabric of life reveals unprecedented complexity.

Solution: The correct answer is A. A) This is a Foundations of Comprehension question because it asks you to discern the author's point of view about the passage topic. A is correct because the author explains that a world where nothing happens, where life is "one long, monotonous nothingness" would be "truly boring" (paragraph 3). At the same time, the author also describes a world where everything is completely random and there is "no shred of continuity" as "again, a boring world" (paragraph 4). He or she goes on to describe a "much more interesting" world, one in which the magnetic universe is near the critical temperature and where events "would be neither quite orderly nor quite random" (paragraph 5). B) The author does not describe a desire to understand the truly unpredictable. C) The passage describes a lack of order and an excess of order but never suggests that order is illusory. D) The author does not idealize "unprecedented complexity," but rather appears to seek a world that is "neither quite orderly nor quite random."

Which of the following methods for redressing economic inequities is NOT mentioned in the passage? A. Monopoly breaking and antitrust regulation B. Union building and strengthening C. Manufacturing and sales oversight D. Redistributive economic policy promotion

Solution: The correct answer is A. A) This is a Foundations of Comprehension question because it asks you to identify arguments or points that are, or are not, mentioned in the passage. A is the correct answer because monopolies and antitrust regulation are not discussed. B) The first sentence mentions building and strengthening unions. C) The first sentence mentions "regulating production and retail practices" as means for redressing economic inequities. D) Redistributive policies are also mentioned in the first sentence.

With which of the following statements about Brink Lindsey's comments would the passage author most likely agree? A. They are not without some merit. B. They are inhumane. C. They are internally consistent. D. They are justifiably biased.

Solution: The correct answer is A. A) This is a Foundations of Comprehension question because it asks you to use cues in the passage to determine the author's point of view on a particular viewpoint (in this case, Lindsey's comments). The answer to this question is A, because the author explicitly says that there is "some justification" for Lindsey's explanation for the fall in coffee prices (paragraph 3) and also in paragraph 4 says that Lindsey "rightly points to" certain problems with wealthier nations' trading practices. However, the author is an advocate of Fairtrade and certainly does not dismiss it as a dead end. B) B is incorrect because there is no reference to or support for it in the passage. C) C is incorrect because the author's response to Lindsey in paragraph 5 (when he says that the "Fairtrade coffee campaign . . . can be seen as doing just what Linsey recommends") suggests that, if anything, Lindsey may be internally inconsistent. D) D is incorrect because although the author may imply that the views of "pro-market economists" certainly tolerate using some mechanisms to raise prices (such as "snob appeal" (paragraph 5) and may mean to put Lindsey in this category), the author does not go on to indicate that such bias is "justified."

What is the most likely explanation of the slower rate of performance observed in Experiment 1? A. A desire to maintain an appearance of relaxed competence before friends B. A feeling of being judged more harshly by friends than by strangers C. An inability to concentrate on mental tasks when friends are present D. A belief that accuracy is more important than speed on certain tasks

Solution: The correct answer is A. A) This is a Foundations of Comprehension question because it requires you to understand passage claims and think through the relationship of these claims to passage options. The answer to this question is A because Experiment 1 found that participants' poorer task performance when a supportive audience was present was the result of a slower rate of responding (paragraph 3); this best supports option A (a "desire to maintain an appearance of relaxed competence before friends"). B) Experiment 1 does not examine the effects on performance of a potential benefit to an audience member. C) Those in the supportive-audience condition were less likely to report distraction than those in the neutral-audience condition. D) The participants were told to perform the tasks as "quickly and accurately as possible" (paragraph 2). Neither quickness nor accuracy was prioritized, so this option is not supported in the passage.

Given the information in the passage, the Embers' theory about the cause of war would best fall into which of the following categories of causes mentioned in paragraph 2? I. "Ultimate" causes II. "Proximate" causes III. Consequences of previous conflicts A. I only B. II only C. I and II only D. I, II, and III

Solution: The correct answer is A. A) This is a Foundations of Comprehension question because it tests your understanding of concepts presented in the passage. The answer is A because the Embers' theory as described in the passage contends that "the societies that engage in the most warfare express considerably more fear of food shortages caused by expected but unpredictable natural disasters, such as drought, flood, or infestation" (paragraph 5). This fits the passage discussion of "ultimate" causes of war in paragraph 2; such causes "influence the goals people fight for, such as competition within a society for scarce resources." The other causes of war described in paragraph 2 do not describe the Embers' theory. B) B is wrong because "proximate" causes spring from a "society's military preparedness and the goals of its leaders." Although the passage includes "the desire for land, natural resources, or control of trade routes," under proximate causes, these desires come from the leaders' goals, which is not the way that the Embers' theory frames the fears of food shortages and other fears. C) C is wrong because as described above, although the Embers' theory would fall into the category of "ultimate" causes, it does not fall into "proximate" causes. D) D is wrong because the description of the "consequences of war" does not fit the Embers' theory, nor does the description of "proximate" causes.

The passage suggests that literary historians changed their assumption that there was a "long lull" in English poetry (final paragraph) because of: A. a reevaluation of the poets of the Great War. B. the advent of modernism in literature. C. the publication of Yeats's Oxford anthology. D. a new emphasis on the contributions of Kipling and Hardy.

Solution: The correct answer is A. A) This is a Foundations of Comprehension question because it tests your understanding of passage claims about the re-evaluation of Great War poets using language that is not in the original passage. The answer to this question is A because the author states in paragraph 5 that poets from the Great War—Graves, Sassoon, and Owen—have only recently received their "just recognition." B) B is incorrect because the author does not specifically discuss modernism in literature. C) C is incorrect because the author maintains that Yeats "could not have been more mistaken" (paragraph 5) in his opinion expressed in the Oxford anthology. D) D is incorrect because the author does not state that there is a new emphasis on Kipling and Hardy; rather, there is a new emphasis on Great War poets such as Graves, Sassoon and Owen (paragraph 5).

The author mentions "Fairtrade Fortnight" (first paragraph) most likely in order to: A. provide background information. B. imply a judgment. C. question a practice. D. include British terminology.

Solution: The correct answer is A. A) This is a Humanities passage that falls under the content category "Ethics." It is a Reasoning Within the Text question because it asks you to evaluate the purpose of a particular piece of evidence in the passage (in this case, the reference to "Fairtrade Fortnight"). The correct answer to this question is A because "Fairtrade Fortnight" is simply the time period when the two previously mentioned events took place, and the whole first paragraph of the passage is merely an introduction to the issue of Fairtrade and its merits. B) B is incorrect because the author doesn't make any judgments until later in the passage, and they involve Fairtrade and Lindsey's opinions, not Fairtrade Fortnight per se. C) C is incorrect because the author doesn't question the practice of Fairtrade, let alone Fairtrade Fortnight. D) D is incorrect because although "fortnight" might be specifically British, "Fairtrade" is not, given the author's last two sentences (paragraph 8); in any case, although there is an explanation of Fairtrade (paragraph 2), the mention of Fairtrade Fortnight does not serve that function.

The "game" that the passage author describes in the first sentence is a: A. necessary strategy for a writer composing in solitude. B. competitive contest of skill between writers and readers. C. deception perpetrated by writers on unsuspecting audiences. D. historical practice that dates from the time when literature was an oral art form.

Solution: The correct answer is A. A) This is a Humanities passage that falls under the content category "Literature." The answer is A, because the "game" the author describes is one that the author says "writers necessarily engage in." The final paragraph suggests that audiences have had to be fictionalized in authors' minds since the time when writing became separate from oral performance and there was no longer an immediate audience for the writer; instead, as the passage explains, the author had to imagine, or fictionalize, an audience that he or she addressed. The game, as the author indicates throughout the passage, requires the reader to learn to understand the terms on which he or she is addressed ("What year? What river? Apparently the reader is supposed to know") (paragraph 5). This is a Foundation of Comprehension question because it asks you to understand the context in which the author uses a particular term in a way it is not ordinarily used and what the author means to indicate when he or she uses this term. B) There is no implication in the passage that the reader is in competition with the writer; rather, he or she is trying to learn the terms of the game, as the author indicates in the first paragraph. C) There is no suggestion in the passage of deception or dishonesty on the author's part. D) The final paragraph indicates the fictional audience has existed only since the time when "writing broke away from oral performance," because when literature was an oral performance, the author did not have to imagine or fictionalize an audience.

According to the passage author, the typical explanations for the drop in creativity of aging rock songwritersare characterized by: A. a failure to generalize. B. a failure to investigate individual life histories. C. an overemphasis on the stereotyped lifestyle of rock artists. D. the intrinsic limitations of rock music as a genre.

Solution: The correct answer is A. A) This is a Humanities passage that falls under the content category "Music." This is a Reasoning Within the Text question, because it asks you to consider the author's approach to evidence and to look for a pattern in that approach or position. The answer is A, because the passage author maintains that the explanations that other music historians for the loss of creativity with age "have invariably been specific to the individual artists in question" (paragraph 1). By contrast, the author maintains that "the loss of creativity with age . . . is not a phenomenon caused by factors unique to these individuals . . . Instead, these artists appear to be prime examples of the loss of conceptual creativity with age, one of the most powerful and pervasive patterns in human creativity" (paragraph 1). So, according to the author, other music historians have erred by looking at the specific experiences of individuals, instead of looking for a general pattern or trend. That is, it is a failure to generalize that has generated the typical explanations. B) To the contrary, this has been the (flawed) approach to the question that most music historians have taken. C) Incorrect. Although a plausible argument, no mention is made in the passage, let alone by the author, of the rock artist lifestyle, stereotyped or otherwise. D) The author specifically argues to the contrary: "the loss of creativity with age . . . is not a phenomenon caused by factors unique . . . to rock music" (paragraph 1).

Which of the following is most like the ability to appreciate art as it is described in the passage? A. The ability to enjoy dance improves the more one dances. B. The ability to enjoy music tends to be inherited from one's parents. C. The ability to run well is hampered by thinking about running well. D. The ability to do any one thing well decreases when one tries to do several things.

Solution: The correct answer is A. A) This is a Reasoning Beyond the Text question because it asks you to apply passage claims to unfamiliar scenarios not discussed in the passage. A is the answer because the author writes: "That capacity is to be trained by the exercise of itself. Each new harmony that someone is enabled to perceive intensifies the power to feel and widens the range of vision" (paragraph 5). B) The passage does not suggest that the capacity to appreciate art is hereditary. C) The author says that thinking is required to appreciate art ("But after nerves have ceased to tingle so acutely, we begin to think—paragraph 2), not that it inhibits our appreciation. D) The author mentions a widening of "the range of [one's] vision" but does not suggest that such widening is counterproductive.

Does the author explain the statement "the future is perpetually giving birth to true novelty?" A. Yes, by comparing it with biological evolution B. Yes, by referring to mass extinctions C. Yes, by using the example of a magnetic world with a temperature below the critical point D. No, because the idea is presented without further explanation

Solution: The correct answer is A. A) This is a Humanities passage that falls under the content category "Philosophy." This is a Reasoning Within the Text question because it asks you to evaluate the support for a claim in the passage. A is the correct answer because the author explains that "Human history is a bit like biological evolution" (paragraph 2). The author offers an extended description of the kinds of novelty that evolution produces and suggests that in human history we similarly encounter things that "have never before existed," such as "the increase in . . . the technological complexity of our world." So the author does explain and elaborate on the claim that humans continually encounter true novelty. B) The mass extinctions are one instance of the "wild fluctuations of the critical state" in evolution, but these fluctuations are not described as unprecedented. C) The example of a magnetic world with its temperature below the critical point (paragraph 3) is used to illustrate a scene of stasis, not novelty. D) This is incorrect because the author offers an extended comparison between human history and biological evolution.

The author's use of the term kinetic immediacy (first paragraph) to describe Winogrand's photographs most likely refers to the photographs': A. ability to capture the hustle and bustle of the city. B. incorporation of roadside scenes. C. historically significant details and context. D. unique compositional strategies.

Solution: The correct answer is A. A) This is a Humanities passage that falls within the content category of "Art History." It is a Foundations of Comprehension question because it tests your ability to use passage context to understand the author's use of an artistic term: "kinetic immediacy." The answer to this question is A because the author uses the term to describe Winogrand's "street photographs," many of midtown Manhattan (first paragraph), which is an urban setting. B) B is incorrect because the author uses the term to describe streets in midtown Manhattan, which does not suggest "roadside" or rural scenes (first paragraph). C) C is incorrect because this term is used to describe "everyday" scenes from midtown Manhattan, not historically significant events that are described later in the first paragraph. D) D is incorrect because although Winogrand's tilted frame effect is described at the end of the first paragraph, this compositional strategy is not discussed in reference to the Manhattan photographs.

The passage implies that Yeats would have been most likely to agree with which of the following statements about English poetry? A. Much outstanding poetry was produced between 1892 and 1935. B. Little valuable poetry was written prior to 1914. C. The war poetry of 1914 through 1918 is among the finest England has produced. D. Kipling and Hardy were the premier English poets of their generation.

Solution: The correct answer is A. A) This is a Reading Beyond the Text question because it asks you to apply a concept from the passage to a conclusion that is not discussed in the passage. The answer is A because W.B. Yeats was the editor of The Oxford Book of Modern Verse [from the years] 1892-1935 (first paragraph), which "purported to be a collection of the finest poetry in our language produced in modern times." B) B is incorrect because Yeats does not maintain that little valuable poetry was written prior to 1914, but instead specifically dismisses the poetry of the Great War. C) C is incorrect because Yeats suggests the opposite: that readers should disregard poetry of the Great War because "passive suffering is not a theme for poetry" (first paragraph). D) D is incorrect because although the passage does not discuss Yeats' views of Kipling and Hardy, based on the passage discussion, he most likely would not have considered them as premier poets as they wrote realistically about war.

The authors imply that the Portuguese ambassador applauded the French National Assembly because of genuine sympathy for their ideals. Alternatively, the ambassador's action could be explained as indicating that: A. living in France, he was in fear of the new revolutionary French government. B. his life outside of Portugal had broadened his political sympathies. C. he was at odds with the medieval policies of Queen Maria. D. he had no strong political commitments one way or the other.

Solution: The correct answer is A. A) This is a Reasoning Beyond the Text question as it asks you to identify the novel situation most consistent with passage information. This is the most plausible: pretending support for those with power over you can offer protection from them. Option A is correct. B) Since this is noted in paragraph 4, it cannot be an alternative explanation, which is what the question requires. C) Although being at odds with the medieval policies of Queen Maria might imply having genuine sympathy for the ideals of the French National Assembly (an opposition between the two is implied in the passage), "imply" doesn't mean "explain"—and the question asks what can explain the ambassador's action. D) If he had no strong political commitments one way or the other, one would think he would be neutral, abstaining from applause one way or the other (unless he feared a power).

Given the information in the passage, the Parthenon's columns most likely "seemed to spring directly from" the stylobate (paragraph 2) because: A. engineering gave the Parthenon an appearance of seamless construction. B. mathematical regularity gave the Parthenon an appearance of balance and harmony. C. entasis gave the columns an appearance of elasticity. D. fluting gave the columns an appearance of shading.

Solution: The correct answer is A. A) This is a Reasoning Beyond the Text question because it asks you to apply a principle from the passage to draw a conclusion that is not discussed in the passage. The answer is A because the passage explains that the Parthenon's columns seemed to spring directly from the stylobate (or the top of the platform on which the temple rests). The use of the word "seemed" implies that the columns do not, in fact, "spring directly from" the platform, and in paragraph 5, the author details the "astonishing" technical skill involved in building the temple: "every block was fitted together with such precision as to defy detection." This information suggests that the effect of the columns appearing to "spring directly from" the stylobate is itself an instance of engineering at work, creating the "appearance," not the reality, of "seamless construction." B) B is incorrect because an appearance of balance and regularity would not necessarily mean that the columns appeared to come directly out of the platform. C) C is incorrect because "entasis" refers to the deviations in the curvature of the columns as they rise, which gives the columns an air of elasticity (paragraph 4), but would not necessarily explain anything about why they appeared to emerge directly from the platform. D) D is incorrect because while the fluting does, among other things, provide shadows, or shading, this would not explain why the columns appear to spring directly from the platform.

It may be inferred from the passage that Portugal might have avoided the unfavorable attention of Napoleonif it had: A. refused to have commerce with Britain. B. supported the French Revolution more enthusiastically. C. not expelled French aliens. D. not followed Spain into war in the Mediterranean.

Solution: The correct answer is A. A) This is a Reasoning Beyond the Text question because it asks you to use passage information to assess the effects of a hypothetical scenario not discussed in the passage. The last sentence of the passage contains the statement that "Napoleon, after conquering half of Europe, reached out for the little state that was refusing to join in his continental blockade of Britain." The implication, then, is that if Portugal had joined the blockade, it might have avoided Napoleon's action. Option A is correct. B) Some constituents of Portugal did support the French Revolution enthusiastically, according to paragraph 4. C) The passage does mention that Portugal arrested, expelled, or kept under surveillance "every important alien" (paragraph 5), but this mention is not made in connection with Napoleon's action. D) Again, the passage does say that "the Portuguese government followed Spain in a holy war against France" (paragraph 5), but Spain negotiated a peace with France. When "Portugal asked for a like accommodation. . . . France refused" (paragraph 5). However, nothing in the passage suggests that this refusal was based on Portugal's entry into war.

While pondering the restructuring of American government during the period from 1786 to 1787, John Adams wrote that collecting the legislation of the Native Americans would be "well worth the pains" and would aid in the process of creating a new constitution. This supports which of the following, as their views are presented in the passage? A. Grinde and Johansen B. Levy C. Payne D. The passage author

Solution: The correct answer is A. A) This is a Reasoning Beyond the Text question because it presents information that is not discussed in the passage and asks you to consider how that information relates to ideas that are presented in the passage. The answer to this question is A because Adams is one of the "Anglo-American founders" (as explained in the first paragraph); Adams's statement would clearly support Grinde and Johansen's thesis that Native American political ideas influenced the U.S. Constitution. B) B is incorrect because Levy questions the influence thesis (paragraph 4), and this statement clearly supports it. C) C is incorrect because Payne draws a conclusion ultimately in opposition to Adams's statement: "Had Anglo-American leaders known more about the Iroquois League, Payne goes on to say, they would probably have concluded that it could not serve them as a model" (final paragraph). D) D is incorrect because the author neither supports nor challenges the thesis that Native American political ideas influenced the U.S. Constitution, while this statement supports it.

Which of the following is the most reasonable inference that can be drawn from the information presented in the passage? A. War is not inevitable; people fight because of needs that could be met in other ways. B. War can only be eliminated by uniting all societies under a single governing council. C. War is inevitable; people will always find good reasons to fight. D. Scientists now completely understand the reasons for war.

Solution: The correct answer is A. A) This is a Reasoning Beyond the Text question because it requires that you extend a central passage argument to a conclusion not articulated in the passage. The answer is A because both the Embers' study and Ferguson support the argument that war is not the natural human condition and therefore not inevitable; rather, the passage suggests that if basic needs such as hunger and essential resources are met in other ways, the main motivation for war may be greatly diminished. B) B is incorrect because the passage does not indicate that a single governing council would be the way to eliminate war. C) C is incorrect because the passage overall supports the opposite inference; for example, Ferguson states in paragraph 7 that "we need to dispense with the idea that people love violence and are doomed to fight." D) D is incorrect because the passage does not suggest that the reasons for war are understood completely by scientists; for example, paragraphs 3 and 6 offer differing scientific opinions about the genetic tendency toward physical violence and about interpersonal aggression.

Based on the passage, of the following, which would be the best example of "mass movements initiated by one or a few magnets that would unexpectedly sweep across the entire world"? A. One country's revolution leads to unrest and revolution in other countries around the world. B. A disease in one country spreads to other countries, causing a pandemic. C. Technological advances in one country are marketed to other developed countries. D. A strengthening of democratic principles in one country is commended by the leadership of many countries around the world.

Solution: The correct answer is A. A) This is a Reasoning Beyond the Text question, because it asks you to apply a passage concept to scenarios not discussed in the passage. The answer is A because the passage constructs an analogy between the behavior of magnets and the circumstances in which historical events occur and become interesting. The author maintains that under such circumstances: "your society would . . . [witness] alliances shifting constantly and in a way that would be neither quite orderly nor quite random." He or she also describes the way "no movement would be a simple copy of what had just happened." Option A describes this kind of a process, in which a revolution in one country influences the history and politics of other countries, but these instances of "unrest and revolution" in other countries could never be carbon copies of the first revolution. B) This is not as good an example of random and unpredictable historical influence as A. Instead, it is an example of a more straightforward biological process and describes a pandemic in which disease appears in multiple locations. C) Again, this is not an example of a random and unpredictable process, but an orchestrated, strategic process. Further, the arrival of a new product in an already developed country is not a particularly dramatic form of change. D) This is simply an example of a historical process or event in one country being noticed and praised in other countries; the element of unpredictable or random influence is absent.

The passage implies that, according to Levy, to say A influenced B, A and B must: A. be more or less similar. B. be the same. C. have at least one feature in common. D. have more features in common than not.

Solution: The correct answer is B. A) A is incorrect because in fact, A (The Iroquois Grand Council, with 50 members) and B (the American Grand Council, with 48 members), are "more or less similar," but Levy considers the discrepancy between them to cast doubt on the influence thesis, so it's not enough, in his view, that they are more or less similar. B) This is a Reasoning Within the Text question because it asks you to identify the nature of the logic that underwrites a particular passage point. The answer to this question is B because one of Levy's objections to the influence thesis is that the Iroquois Grand Council has 50 members, whereas the American Grand Council has 48 members; this objection suggests that for A to influence B, A and B must be exactly the same. C) C is incorrect because Levy implies that they must be the same, not simply share a feature. D) D is incorrect because Levy implies that they must be exactly the same, not just have more features than not in common.

Herbert Gorman's 1939 biography of the writer James Joyce was written with Joyce's cooperation and published in Joyce's lifetime. These facts would appear to challenge which of the following assertions made in the passage? A. Writers prefer that readers concentrate on the writer's work rather than read a biography of the writer. B. As they grow older, writers fear what will become of their reputation after death. C. Biographies cannot explain the meaning of a writer's work. D. Most readers are interested in reading about the lives of their favorite writers.

Solution: The correct answer is A. A) This is a Reasoning Beyond the Text question, because it introduces information that is not in the passage and asks you to consider how that information would affect passage claims. The answer is A, because the quotation describes a biography of James Joyce that Joyce agreed to and saw published in his own lifetime. The passage author argues that writers are usually uncomfortable with the prospect of biography because they want "to concentrate the reader's mind on the work" (paragraph 5). So if Joyce cooperated with the biography, this would challenge the author's claim that writers prefer to avoid biographers and to emphasize their works instead. B) This claim is not challenged by Joyce's participation in the biography. In fact, one could argue that he might have participated in an effort to exert control over what the biography would contain. C) Joyce's participation in the project does not challenge (or support) this claim. D) Joyce's participation in the biography does not challenge this assertion of reader interest.

After the Napoleonic Wars, the Portuguese people demanded democratic reforms, including major restrictions on the power of monarchs. Together with information in the passage, this fact most clearly indicates that: A. John had good reason to fear that new ideas from abroad would undermine monarchial power. B. the Catholic Church had lost its stabilizing social influence. C. the policies of Napoleon had become increasingly popular in Portugal. D. Freemasonry was coming into the social and political mainstream.

Solution: The correct answer is A. A) This is a Reasoning Beyond the Text question, because it provides information that is not included in the passage, and asks you to integrate it with passage information to draw a conclusion. The answer is A. The passage describes John's repressive reaction after the French king Louis XVI was deposed by a popular uprising in 1792: "the future John VI turned with fury on the liberals of Portugal" (final paragraph). So, if the Portuguese people "demanded . . .major restrictions" on monarchical power after the Napoleonic Wars, this certainly suggests that John's earlier fears that the power of the throne could be undermined by "new ideas from abroad" had some rational basis. B) The passage says that the monarchs depended on the Church for its support of "social order and absolute, divine-right monarchy," but the demands of the Portuguese people for democratic reforms cannot necessarily be seen as an indication that the Catholic Church "had lost its stabilizing social influence" in Portugal. It may have been that other influences just became stronger at that moment. Further, the Church may have retained some aspects of its influence, without maintaining its sway on the question of divine right. C) The passage provides no conclusive link between the policies of Napoleon and support for "democratic reforms." Because these demands for reform came after the Napoleonic Wars does not mean that they were inspired by support for Napoleon's positions. D) Although the Freemasons were part of the "minority" who were "irked by the despotism of the past," a move away from such despotism does not necessarily imply support for the Freemasons' ideas specifically.

In Experiment 1, which of the following hypotheses would most reasonably account for the post-experimental statements made by those in the neutral condition? A. A performer's anxiety need not adversely affect performance. B. A performer's anxiety can be reduced with no effect on performance. C. An audience can affect a performance by reducing performance anxiety. D. An audience can affect a performance by causing performance anxiety.

Solution: The correct answer is A. A) This is a Reasoning Within the Text item because it asks you to consider the logical relationships between the options and passage claims, or the implications of passage claims. The answer to this question is A because those in the supportive-audience condition were "less likely than those in the neutral-audience condition to report feeling distracted by the observer or feeling stress while doing the task" (paragraph 4). This must mean that those in the neutral condition were relatively more likely to report distraction or stress, but the passage also tells us that they had better performance than those in the supportive condition. So, if the statement in option A is true and a performer can feel anxiety without a negative effect on performance, then this would explain the apparent discrepancy (between reported anxiety and performance) of those in the neutral condition. B) This is incorrect because those in the neutral condition did not report reduced anxiety. C) Again, those in the neutral condition reported relatively more anxiety than those in the supportive audience condition. D) Although those in the neutral condition reported more anxiety than those in the supportive condition, they performed better relative to those in the supportive condition; thus, this statement, which implies that anxiety negatively affects performance, would not explain the statements by those in the neutral condition.

Which of the following passage assertions suggests most strongly that the author is critical of capitalist society? A. Freedom to consume is equated with freedom itself. B. Photographic images make reality problematic. C. Social change is expressed by a change in images. D. Individual perception is a mythical realm.

Solution: The correct answer is A. A) This is a Reasoning Within the Text question because it asks you to assess the way that the author is using a piece of evidence in the passage. A is the correct answer. This statement is made to explain, in a capitalist society, the constant production and consumption of images ("The narrowing of free political choice to free economic consumption [that] requires the constant production and consumption of images" in paragraph 4.) Because the author contends that this increasing supply of images is what leads to the 'disease' of a depleted real world (paragraph 5) and our inability to distinguish between images and real objects (paragraph 6), it does suggest that the author is critical of capitalist society. Freedom or liberty, by definition, entails much more than the right to purchase commodities. B) The term "problematic" can have the connotation of something "debatable," this can be interpreted as remark that means reality is hard to define or pin down. Such a statement, therefore may not necessarily be a criticism of capitalism. For example, in philosophic inquiries life, love, God, truth, and many other abstractions are considered problematic, or open to inquiry and probing examination. C) Whether or not this would imply criticism depends on the exact nature of the "social change." If, for example, it involved increasing equality for disadvantaged segments of the population, its expression in images, helping to implement widespread perception of equality, might earn the passage author's approval. D) This observation, which is presented in paragraph 3, pertains to the author's discussion of the aesthetic and instrumental approaches. Capitalist society isn't mentioned at all until the next paragraph (4), and the two things are never discussed in relation to each other.

The curator who used the example of Max Brod refusing to destroy the manuscripts of Franz Kafka (final sentence) was most likely implying that: A. the individual rights of an artist are sometimes outweighed by the greater public and artistic good. B. the destruction of an artist's work is never warranted. C. once a work of art is created, its destruction is almost a crime against humanity. D. great artists will always attempt to keep their works from being seen and must be prevented from doing so.

Solution: The correct answer is A. A) This is a Reasoning Within the Text question because it asks you to consider how a particular passage example is used, that is, what the curator intends the example of Max Brod and Kafka's manuscripts to mean. The answer to this question is A because the curator lends support to the passage argument by claiming "the artist is not always in the best position to judge his or her work"; and the author adds that others may need to make these decisions, "to the world's benefit" (paragraph 5). B) B is incorrect because this is too extreme a claim to derive from the example; the curator does not suggest that such destruction is "never warranted," nor that artists are never in a position to judge when such destruction might be warranted. C) C is incorrect because again, this is too extreme a claim to derive from the example in the passage and is not supported. D) D is incorrect because the curator and the author do not imply that all "great artists will always attempt to keep their work from being seen."

One view of war is that it springs from fear of the unknown: People who are confident about their future do not declare war on their neighbors. Does the information in the passage support this view? A. Yes; the Embers' research suggests that fear of unpredictable disasters is at the root of much warfare. B. Yes; studies show that the innate tendency toward aggression leads societies to fear outsiders. C. No; Ferguson's views suggest that a great deal of confidence is required for a society to engage in war. D. No; the theory of consequences of previous conflicts suggests that people's attitudes have little effect on whether their society will wage war.

Solution: The correct answer is A. A) This is a Reasoning Within the Text question because it asks you to evaluate a theory that is briefly discussed in the passage—that is, that war springs from fear of the unknown—and apply it to other information presented in the passage. The answer is A because paragraph 5 quotes the Embers' main finding that societies with the most warfare express fear of food shortages ". . . caused by . . . unpredictable natural disasters. . . ." B) B is incorrect because the passage does not present definitive evidence supporting the claim that aggression is an innate human tendency or that warring societies fear outsiders. C) C is incorrect because Ferguson does not suggest that confidence is required in order to engage in war. D) D is incorrect because the theory of consequences (versus causes) suggests that when societies lose wars, "harsh child-rearing methods diminish sharply" (paragraph 6). This is different than suggesting societal attitudes have little effect on waging war.

The information that the Skellig was once particularly favored as a place of pilgrimage is most plausibly connected to which of the following passage assertions? A. There is a stubborn sufficiency about the place that is irresistibly attractive. B. The monastery was similar to hermitages that originated in the deserts of Egypt. C. The first Christian king of Norway was supposedly baptized by a Skellig monk. D. The eremetical movement was like a frenzy.

Solution: The correct answer is A. A) This is a Reasoning Within the Text question because it asks you to identify relationships or connections between passage claims. A is the answer because it suggests that there was something specific about Skellig Michael that would draw those from the outside, even after the height of the eremitical movement was over. This is most plausibly connected to the claim that "there is a stubborn sufficiency about the place that is irresistibly attractive." B) This does not explain anything about the particular attraction of Skellig Michael; rather it links the place to other hermitages. So it does not explain why penitents would have continued to be attracted to it as a site of pilgrimage into the eighteenth century. C) This historical fact hardly explains why pilgrims "from across Europe" would continue to seek out Skellig Michael. D) The attraction of pilgrims to Skellig Michael is described in the passage as having occurred after the "frenzy" of the movement (which subsided after the fourteenth century), so this reference does not illuminate the pilgrims' attraction to the place.

Which view expressed by the author is most important for readers to share if they are to be convinced of the merits of the passage argument? A. Fantasy violence is an essentially healthy experience. B. The taste for television violence reflects audience inadequacy. C. The majority of people are dishonest about violence. D. Professors abuse their positions as teachers.

Solution: The correct answer is A. A) This is a Reasoning Within the Text question because it asks you to identify the view that most supports the central passage argument—that television violence deserves "much less criticism" (first paragraph). The answer to this question is A because the author's view that fictional violence is healthy is the premise of the argument that television violence deserves "less criticism" (first paragraph). B) B is incorrect because the author does not view the taste for violence as reflecting inadequacy; rather, she views television violence as "an improvement in the human condition . . . [a way] that well-adjusted human beings vent in a civilized society . . ." (paragraph 5). C) C is incorrect because although the author claims that people may be dishonest about what they say they want (as opposed to "what they actually watch" (first paragraph), this premise is not central to her argument that fictional violence is healthy. D) D is incorrect because the author does not claim that professors deliberately or consciously abuse their positions, but rather that they promulgate their views in "their lectures . . . and, indirectly, by not contradicting the conventional antiviolence wisdom" (paragraph 4).

With which of the following statements would the author most likely agree? A. That which delights is always that which is right. B. The more things we can appreciate, the richer our being. C. To be human is to be an artist. D. A tree is just a tree.

Solution: The correct answer is B. A) The author does not address morality. B) This is a Reasoning Within the Text question, because it asks you to identify the claim that is most consistent with passage arguments. B is the answer because it is consistent with the author's position: "The more beauty a person apprehends in the world, so much the more of universal forces such a person brings into unity with the personality that is the hallmark of self" (final paragraph). C) Incorrect. Although plausible, there is nothing in the passage to suggest the author would agree with this, as he makes no claim about the origin of the artistic impulse. D) Incorrect. The author believes just the opposite: "To him who has never opened his eyes to behold the beauty of field and hill and trees and sky, to him whose spirit has not dimly apprehended something of that eternal significance of which these things are the material visible bodying forth. . . ." (paragraph 4).

The author intends to suggest in paragraph 5 [from "Alternatively, insofar" to "their employee's performance"] that large organizations: A. overestimate the quality of their employees' performance. B. underestimate the quality of their employees' performance. C. hire independent consultants to estimate the quality of their employees' performance. D. are willing to pay high fees to obtain an accurate estimate of the quality of their employees' performance.

Solution: The correct answer is A. A) This is a Reasoning Within the Text question because it asks you to understand the use that the authors make of a particular claim and the authors' likely reasoning. The answer to this question is A because the authors suggest that workers in large organizations may not be properly monitored, and therefore may be over compensated: "such organizations may pay higher wages than small organizations because larger ones are less able to accurately measure their employees' performance." The implications is that the inaccurately monitored performance leads to higher wages (compared to those paid at smaller organization), thus the inaccuracy in performance measurement would be an overestimation (rather than the underestimation that could lead to lower wages). B) B is incorrect because the author is seeking to explain higher wages here, and thus the potentially inaccurate measurement that leads to those higher wages is unlikely to be an underestimation. C) C is incorrect because it is not supported; the passage does not mention that large organizations may hire independent consultants. D) D is incorrect because the author suggests the opposite, that "large organizations may want more qualified workers in order to reduce monitoring costs" (paragraph 5).

Why does the author most likely mention "the role of chance" (paragraph 4)? A. As part of an explanation of why clinical impressions may be flawed B. As a response to concerns about the small sample size clinicians may use C. As a feature that makes clinical experience superior to evidence-based medicine D. As part of an argument against the use of intuition in medical practice

Solution: The correct answer is A. A) This is a Reasoning Within the Text question, because it asks you to understand the structure of the author's argument, as well as the function of a particular part of the passage as an element in the argument. The answer to this question is A because paragraph 4 is the second paragraph of three paragraphs (paragraphs 3, 4, and 5) that develop the author's claim in the preceding paragraph that "there are a number of reasons that clinical experience may provide insufficient or misleading data" (paragraph 2). B) The small sample size is given as another reason that the evidence gained from clinical medicine may be misleading (paragraph 4); the discussion of the role of chance is not a "response" to concerns about the small sample size. C) Again, the reference to the small sample size available in clinical experience is part of the author's discussion of the limitations of clinical experience, not its superiority to evidence-based medicine, which can provide a "better information source." D) The author does not argue against intuition so much as argue that it is insufficient by itself (especially clear in paragraph 2).

The passage authors' central concern appears to be to: A. explore various factors that might be responsible for the relationship between organization size and job rewards. B. present evidence supporting the claim that employees in large organizations have higher earnings than those in small organizations. C. advocate greater equity with regard to job rewards for employees of small organizations. D. explain why organization size directly affects job rewards.

Solution: The correct answer is A. A) This is a Social Science passage that falls under the content category "Business." This is a Foundations of Comprehension question because it requires you to identify the authors' primary argument in the passage. The answer to this question is A because the authors state in the first paragraph that there are several factors that may explain the differences between large and small organizations in terms of rewards for employees. B) B is incorrect because the authors do not present specific evidence that employees in larger organizations have higher earnings, nor is this point broad enough to encompass the authors' central concerns. C) C is incorrect because the authors do not advocate for equity for employees of smaller organizations. D) D is incorrect because the authors elaborate on several factors affecting job rewards beyond organization size, including product and labor markets, organizational structure, job characteristics, and unionization.

Assume that the description of teaching behavior in the second paragraph applies to a human context, not just to the animal world. Which one of the following scenarios would qualify as teaching behavior, according to the criteria Franks adopts? A. A toddler complains that the sun hurts his eyes, and the teacher takes off her sunglasses and shows the child how to put them on. B. A toddler says she is cold, and the teacher shows her how to button her sweater. C. A child incorrectly draws a square on the blackboard, and the teacher corrects her by drawing a figure with four equal sides next to it. D. A student of French watches as a teacher points to objects in the classroom that are masculine nouns in that language.

Solution: The correct answer is A. A) This is a Social Science passage that falls under the content category "Education." This is a Reasoning Beyond the Text question because it asks you to apply a concept from the passage to a scenario that is not discussed in the passage. The answer to this question is A because Franks describes an animal as teaching when it "modifies its behavior in the presence of another, at cost to itself, so another individual can learn more quickly" (paragraph 2); the teacher giving her sunglasses to the toddler fits Franks's description. B) B is incorrect because the teacher's behavior is not putting herself at risk. C) C is incorrect because the teacher has not modified her behavior in the presence of another. D) D is incorrect because the teacher's behavior is not putting herself at risk.

By using terms such as subtlety, unexpectedness, and complex in describing Winogrand's work, the author seems to be implying that: A. Winogrand was not interested in fame or monetary rewards. B. Winogrand's talent is not readily apparent to viewers of his work. C. only photography experts can appreciate Winogrand's work. D. Winogrand attempted to do too much with his photography.

Solution: The correct answer is B. A) A is incorrect because these descriptors do not relate to the interest (or lack thereof) in fame or money on the part of the artist. B) This is a Reasoning Within the Text question because it asks you to understand the point that the author means to make through the use of particular descriptive language such as subtlety, unexpectedness, and complex. The answer is B because these descriptors all point to something that requires some kind of careful attention or discernment to recognize. C) C is incorrect because the author does not maintain that only experts can appreciate Winogrand's works. Although the descriptors suggest that Winogrand's talent may not be readily apparent to all viewers, this hardly suggests that only experts can appreciate it. D) D is incorrect because these are all positive descriptors; this criticism is not implied.

The central thesis of the passage is that class-size reductions: A. can improve academic achievement under some conditions but may not be the most effective way to do so. B. improve academic achievement when teachers use techniques well suited to small classes. C. improve academic achievement only in the early elementary grades. D. are less efficient than other means of improving academic achievement.

Solution: The correct answer is A. A) This is a Social Science passage, which falls under the content category "Education." This Comprehension question tests your understanding of the main argument of the passage. The answer is A, as the passage argues that while there is some evidence that reductions in class size do improve academic performance, these reductions are shown to be effective "only under certain conditions" (paragraph 2). The author argues that because reductions in class size are more expensive than are some other alternatives for improving academic performance, researchers should do more to compare the costs and benefits of reductions in class size with those of other interventions "before they can make sensible policy decisions" (final paragraph). B) The passage suggests that this is a "likely explanation" for the "academic benefits of class-size reductions," but this is not the central thesis of the passage. C) The author says that research shows that performance gains from small classes do seem to "occur in the early elementary grades and do not accumulate" beyond these, but this is not the central thesis of the passage. D) The passage does not make this claim, but suggests that the relative efficiency and effectiveness of different means of improving performance must be studied in much more detail to prevent wasted resources.

The author implies which of the following about clinical practice? A. Some physicians tend to give clinical experience undue preeminence over research experience. B. Even at its best, clinical experience is anarchic. C. The role of clinical experience in the practice of medicine is overly complex. D. Physicians embrace clinical experience primarily because it gives priority to their own intuitions.

Solution: The correct answer is A. A) This is a Social Sciences passage that falls under the content category "Population Health." It is a Foundations of Comprehension question because it asks you to identify the author's position about a passage topic—clinical medicine—using language that is not in the original passage. The answer to this question is A because the author writes: "unfortunately, some physicians consider information derived from clinical experience to be sacrosanct and above questioning" (paragraph 2). The author goes on to speak of control groups, statistical analysis, and "rigorous scientific testing"—research experience—that is not taken into account by clinical experience. B) It is not the author but "economists and other health experts" (paragraph 1) who "view . . . clinical experience as . . . uncontrollable, chaotic, and obeying few rules." C) Although the author says the role of clinical experience "is based on a complex interweaving of observations" (paragraph 2), this does not necessarily imply that clinical experience is "overly" complex. D) The author does not say this is why clinical experience is embraced, nor does he say that intuition is given priority (he mentions values and judgment along with intuition).

Which of the following findings would most weaken the author's explanation of the observed effects of class-size reductions? A. Class-size reductions in the early grades are effective regardless of whether the teachers use methods well suited to small classes. B. Raising teacher salaries produces much greater improvements in student performance than does reducing class sizes at all grade levels. C. When teachers emphasize personal interactions with students, academic achievement typically improves regardless of class size. D. Academic achievement levels are often low even when teachers use methods well suited to small classes.

Solution: The correct answer is A. A) This is another Reasoning Beyond the Text question, because it asks which situation, listed in the options and not presented in the passage, would have a particular effect (in this case, a weakening effect) on a passage claim. The answer is A, because the author's explanation for improved performance in small classes rests on the fact that "some teachers, who already use methods well suited to smaller classes, do very well when they are actually given small classes." If, however, early grades showed improvements in student performance in smaller classes regardless of teaching techniques, then this would clearly weaken the author's explanation for the effects of class-size reductions. B) This is irrelevant to the author's explanation for the effects of class-size reductions. C) This does not weaken the author's explanation for the observed effects of class-size reductions; this describes an effect that seems to be independent of class size. D) Again, the author explains an "observed effect." This claim does not weaken that explanation. Further, the author discusses improvements in academic achievement based on class size; if achievement starts at a very low level, it may still be relatively low, even after improvement.

Which of the following statements from the passage most strongly indicates that the passage was writtenprior to the twenty-first century? A. "The regular holding of deliberative assemblies of a thousand members encounters the gravest difficulties in respect of room and distance. . . ." B. "The crowd . . . is . . . readily influenced by the eloquence of popular orators." C. "[T]hese same assemblies, if divided into small sections, . . . would be much more guarded in their assent." D. "The sovereign masses are incapable of undertaking the most necessary resolutions."

Solution: The correct answer is A. A) This is a​ Social Science passage that falls under the content category "Government." This Comprehension question tests your understanding of passage statements as they appear in the content of the passage. The correct answer is A, because that option clearly reflects an earlier set of technological possibilities. In the twenty-first century, the Internet and other forms of electronic communication would certainly ameliorate "the gravest difficulties" in "holding . . . deliberative assemblies of a thousand members." The sentence in paragraph five that follows the sentence in option A ("Even if we imagined the means of communication . . .") clearly signals as well that the passage was written before the twenty-first century. B) There is no reason to think that eloquent orators do not retain the power to influence a crowd in the twenty-first century. C) There is no reason to think that the influence of large groups of people, as opposed to smaller groups, is less powerful in the twenty-first century than it was in previous eras. D) Nothing about this claim signals that it was made before the twenty-first century, or that it could not be made today.

Suppose that given a choice of various paintings, sailors choose seascapes. How would this relate to the opinions expressed in the passage? A. It would support the author's claim about the tendency that is the basis of all art appreciation. B. It would challenge the author's claim about how to best develop our capacity to appreciate art. C. It would support the author's implication about the beauty inherent in all things in nature. D. It would challenge the author's implication about the kind of art that is likely to be popular.

Solution: The correct answer is A. A)This is a Reasoning Beyond the Text question because it asks you to consider how a scenario not presented in the passage would affect passage arguments. The answer to this question is A because the sailor would be intimately familiar with the subject of the seascapes. This is similar to the author's discussion in the passage of the visitor who wonders why an author has not included chickens in his study of sunlight: "The chickens, which she had seen and could recognize, were the element of the familiar she required in order to find any significance in the picture. This tendency is the basis of all appreciation" (paragraphs 3-4). B) The author argues that the capacity to appreciate art "is to be trained by the exercise of itself" (final paragraph). This suggests that we develop the ability to appreciate art by exposing ourselves to a great deal of art. The sailor's preference for seascapes does not challenge this approach, and may in fact support it, by suggesting that had the sailor been exposed to more art, he or she might find more to admire in art that contained other subjects besides the sea. C) The author does imply that beauty is inherent in all things in nature, but the sailor's preference for seascapes does nothing to support that. D) The author writes of "the popularity of those pictures whose subjects are familiar and can be immediately recognized" (paragraph 3), so this supports, rather than challenges, this implication.

Which of the following best describes the kind of artist the Golden Era songwriters were? A. Inexperienced artists B. Artists-for-hire C. Would-be artists D. Former artists

Solution: The correct answer is B. A) The passage indicates that they were professional, which means they made an income from their songwriting. Their work is described in a context that traditionally provides payment. B) This is a Foundations of Comprehension question because it asks you to understand a key component of the passage and uses language that is not necessarily in the passage itself. B is the answer because, according to the passage, the Golden Era songwriters were hired to write specific songs for specific purposes; they "wrote songs to fit the needs of musical comedies" (paragraph 3). They did not write primarily from personal inspiration. C) The passage describes them as successful working artists. D) Paragraph 3 describes them as successful working artists, and especially as they improved as they aged, they would be less likely than the rock songwriters to become "former artists" very quickly.

The discussion of the Parthenon's corner columns (paragraph 4) best supports the passage assertion that: A. Greek architecture was largely concerned with refining the post-and-lintel system. B. Greek architects strove to create a sense of balance among elements. C. the principle behind the Parthenon's design was extremely simple. D. the conception of the Greek temple was basically that of an idealized house.

Solution: The correct answer is B. A) A is incorrect as the discussion of the corner columns describes an architectural intervention that goes well beyond "refining the post-and-lintel system;" the system that involves simply "setting up a series of vertical posts for the support of a horizontal member, called the lintel" (first paragraph). B) This is a Humanities passage, which falls under the content category of "Art History." This is a Reasoning Within the Text question because it asks you to consider how a particular piece of evidence from the passage works to support a passage claim. The answer is B. Paragraph 4 indicates that because of the color of the background against which the corner columns appear, the corner columns would seem to be thinner than those in the middle, if all columns had the same diameter. Therefore, the architect has "corrected this optical distortion . . . thus promoting the appearance of stability at these important points" (paragraph 4). This supports the claim that "Greek architects strove to create a sense of balance among elements." This is precisely what the architect is described as doing with the corner columns: working to create a sense of balance, or equality, among the columns, rather than allowing the optical distortion (creating a sense of difference among the columns) to prevail. C) C is incorrect as the discussion of the corner columns describes the technical approach that the architect used to correct the appearance of an imbalance in the columns; the discussion does not support a claim about the principle behind the Parthenon's design (which is again a reference to the post-and-lintel system). D) D is incorrect because the discussion of the way that the architect created a sense of proportion among the columns and stability in the corner columns does not relate to the point about the Greek temple as an "idealized house conceived of as the dwelling place of the god" (first paragraph).

Which one of the following groups of people is likely to find support for their educational ideas in Hauser's distinction between learning and information? A. Opponents of classroom discussion and debate B. Opponents of rote memorization C. Proponents of team teaching D. Proponents of case study approaches

Solution: The correct answer is B. A) A is incorrect because Hauser would most likely argue that true teaching involves discussion and debate, because the teacher is aware "that the students don't know something" (paragraph 7). B) This is a Reading Beyond the Text question because it asks you to apply a concept from the passage to a scenario that is not discussed in the passage. The answer to this question is B because Hauser argues that teaching is more than a "transfer of information . . . [it] involves[s] a 'theory of mind'" (paragraph 6). C) C is incorrect because the passage does not present any information about team teaching. D) D is incorrect because the passage does not discuss case study approaches.

Someone who agreed with Misrach's defense of his choice to show the color slides would be most likely to also approve of: A. exhibiting works that an artist had donated to a museum for scholarly purposes only. B. examining the rest of Winogrand's unprinted photographs and selecting some for display. C. requiring that artists clearly state their intentions for display and publication when donating works to a museum. D. organizing an exhibition that included all of Winogrand's work whether previously shown and published or not.

Solution: The correct answer is B. A) A is incorrect because Misrach does not argue that works should be displayed if the artist has specified that they should be used for research only. B) This is a Reasoning Beyond the Text question because it asks you to apply passage arguments (by Misrach) to a conclusion not articulated in the passage. The answer to this question is B because Misrach maintains that if Winogrand didn't want his unprinted photographs to be displayed, he could have "simply destroyed them" (paragraph 5). Someone who agreed with this approach would likely also believe that it is permissible to assess any extant photographs and select some for display. C) C is incorrect because Misrach does not claim that artists must clearly state their intentions when donating works to a museum. D) D is incorrect because Misrach does not suggest that all of Winogrand's works should be shown in one exhibition.

The point that Payne makes about the three images of the Iroquois (paragraph 5) is most analogous to: A. the argument that anthropologists often readily adopt the culture of those people whom they study. B. the claim that artists represent the reality that they perceive, rather than an empirically verifiable reality. C. the idea that linguists must understand the values of those who speak a particular language before they can understand the structure of that language. D. the belief that the underlying principles of musical notation are consistent across historical periods and across cultures.

Solution: The correct answer is B. A) A is incorrect because Payne's point does not necessarily imply the adoption of the actual culture of the Iroquois; it is perception that he is concerned with. B) This is a Reasoning Beyond the Text question that asks you to apply a point from the passage to a series of novel scenarios, to assess which one is most analogous to the passage claim. The answer is B because Payne describes 3 distinct images of the Iroquois: "(a) the Iroquois as they actually were in the seventeenth and eighteenth centuries; (b) the Iroquois as the authors of the Articles of Confederation and the framers of the Constitution understood them to be in the late eighteenth century; and (c) the Iroquois as they have been described by the proponents of the Iroquois influence thesis." The differences between the reality of the Iroquois and that the way that the framers and the proponents of the influence thesis see them suggests that there is a difference between perception and reality and that it would be this perception of the Iroquois that would be most relevant if the founders were to be influenced by the Iroquois notion of government. "In Payne's view, only the image Anglo-American leaders had of the Iroquois could have influenced the formation of the union." C) C is incorrect because Payne's point is about the distinction between perception and reality, not about understanding the Iroquois as they understood themselves (understanding their values). D) D is incorrect because Payne's point is about the different versions of the Iroquois that exist, not about the consistency among these versions.

It is reasonable to conclude from the information presented that poets were constrained in writing about their experiences in the Great War because: A. few had been able to complete their education due to their military obligations. B. the poets who might have served as their models wrote in a style inappropriate to wartime subjects. C. they knew very little about the poetic traditions that had preceded them. D. they were intimidated by the style of the previous generation of poets.

Solution: The correct answer is B. A) A is incorrect because although plausible, the passage does not discuss English poets' ability to complete their education during the Great War. B) This is a Reasoning Beyond the Text question because it asks you to apply passage information to draw a conclusion that is not discussed in the passage. The answer to this question is B because the author states in paragraph 4 that the Great War generation "was ill-equipped poetically to express itself, having neither tradition to draw upon nor worthwhile models to imitate." The author explains that before the Great War, poets "who dealt with war . . . render[ed] it exotically or historically removed from immediate experience" (paragraph 3). The author implies that such a style was inappropriate to the Great War which demanded the representation of the "indescribable and . . . unthinkable" (final paragraph). And according to the passage, the Georgian poets, who wrote just before the Great War "also found inspiration in the exotic, counterbalanced by romantic rhapsodizing about the English countryside" (paragraph 4). Again, the author clearly implies that such an approach to poetry, taken by those poets who might have served as models for the poets of the Great War, was hardly appropriate to influence those who would represent the "catastrophe of 1914." C) C is incorrect because the author does not indicate that poets of this generation knew little about poetic traditions. D) D is incorrect as the passage does not suggest that the Great War generation was intimidated by the style of previous poets; rather that, absent any "worthwhile models," they initially "relied on well-established forms" (paragraph 4).

Which of the following most accurately summarizes the passage author's views on television violence in the context of public policy? A. The current advocates of television violence tend to be highly biased. B. Public stances on television violence are affected by ulterior motives. C. We have good reason to believe that television violence benefits society. D. The presentation of imaginary violence on television is not a public policy issue.

Solution: The correct answer is B. A) A is incorrect because the author does not view the advocates as biased; rather, he claims that those politicians who oppose television violence link their fame to "an immaculate concern—the purification of American family life" (paragraph 3). B) This is a Foundations of Comprehension question because it asks you to understand the author's claims about public policy using different language than found in the passage. The answer to this question is option B because the author claims that "astute politicians . . . are depicting [themselves] . . . as savior[s], purer than pure" (paragraph 3), which would indicate ulterior motives. C) C is incorrect because although the author does promote the idea that television violence may benefit society, his main argument regarding public policy is that politicians have ulterior motives (paragraph 3). D) D is incorrect because the author claims the opposite—that television violence should be a public policy issue: "What can be asked for here is that the public and the scholarly community apply a high level of intellect to the issue of television violence" (paragraph 6).

Applying the organization-employee autonomy discussion in the passage to a family-child situation, which of the following children is most likely to have the LEAST autonomy? A. One whose parents do not monitor after-school activities B. One whose parents establish formal rules of behavior C. One who receives a high weekly allowance D. One who lives in a small community

Solution: The correct answer is B. A) A is incorrect because the author states in paragraph 5 that difficulty with monitoring may increase autonomy. B) This is a Reasoning Beyond the Text question because it asks you to apply a concept from the passage to a scenario that is not discussed in the passage. The answer to this question is B because the author states in paragraph 4 that formal rules tend to decrease autonomy. C) C is incorrect because the author indicates that higher salaried jobs in large organizations may result in more autonomy (paragraph 5). D) D is incorrect because the author explains that small organizations tend to be in smaller communities, and that, overall, workers in small versus large organizations may enjoy more autonomy (first paragraph and paragraph 3).

If ungrooved columns appeared round in direct sunlight, this would most strongly challenge the passage assertion that the use of grooves: A. forms concave channels in a column's shaft. B. corrects an optical illusion. C. provides graceful curves. D. is aesthetically pleasing.

Solution: The correct answer is B. A) A is incorrect because the author writes that the twenty grooves in the round columns "formed concave channels from the bottom to the top of the shaft" of the column (paragraph 3). If the ungrooved columns appeared round in direct sunlight, this point would be irrelevant to any grooves made in the columns. B) This is a Reasoning Beyond the Text question because it describes a hypothetical scenario that is not discussed in the passage (in this case, it is actually the opposite of a passage claim) and asks you to assess how this scenario would affect the existing passage. The answer is B because the author explains that, without grooves, round columns appear flat when seen in direct sunlight. The grooves, the author explains, "correct [this] optical illusion." If, in fact, ungrooved round columns appeared round in direct sunlight, there would be no optical illusion to correct, and the claim in B would be challenged—or nonsensical. C) C is incorrect because even if ungrooved columns appeared round in direct sunlight, this point would not pose any challenge to the claim that the grooves (or fluting) create the appearance of graceful curves. D) D is incorrect because the presence of round-appearing columns would not do anything to challenge the claim that grooves help to create an aesthetically pleasing set of curves.

The information in the passage most strongly supports which of the following general statements? A. War is inevitable because aggression is an inherent human trait. B. War is a response to the need for food or other resources. C. The more internally aggressive a society, the more it is likely to wage war on other societies. D. The study of warfare among nonindustrial societies makes it easier to understand war in industrial societies.

Solution: The correct answer is B. A) A is incorrect because the passage suggests that aggression is not an inherent human trait (paragraph 7). B) This is a Reasoning Beyond the Text question because it asks you to consider passage arguments in order to draw a conclusion that is beyond the scope of the passage. The answer to this question is B, because scarcity of food and resources is mentioned in paragraph 2 as one of the ultimate causes of war; scarcity is mentioned as well as in paragraph 5 as the major finding of the Embers' study: "societies that engage in the most warfare express considerably more fear of food shortages. . . ." C) C is incorrect because the passage indicates that "warfare apparently causes [aggression], rather than vice versa" (paragraph 6). D) D is incorrect because there is nothing in the passage that specifically or directly compares warfare in industrial to nonindustrial societies.

Suppose that there were no trade barriers in existence anywhere for the import and export of electronics. Information in the passage suggests that which of the following would most likely be the case? A. Fairtrade electronics would develop. B. Electronics would be a profitable industry. C. "Designer" equivalents would flood the electronics market. D. The electronics industry would develop more efficient technologies.

Solution: The correct answer is B. A) A is incorrect because there is no indication in the passage that a Fairtrade variant of a particular product will develop in response to the existence of trade barriers. B) This is a Reasoning Beyond the Text question because it asks you to consider the impact of passage content on a hypothetical scenario not discussed in the passage. The answer to this question is B because the author agrees with Lindsey in paragraph 4 that "rich nations' trade barriers and subsidies" are "obstacles" to profitability. Thus, based on the passage, the barrier-free scenario described in the question would "most likely" produce a profitable industry. C) C is incorrect because it is not supported. Trade barriers are mentioned in relation to profitability, not in relation to the emergence of "designer" products. D) D is incorrect because trade barriers are not mentioned in relation to efficiency. New, more efficient, technologies are mentioned in paragraph 3 only in relation to creating less expensive products.

If Brink Lindsey's information is correct, how reasonable is his claim that Fairtrade is a "dead end"(paragraph 3)? A. Not very reasonable, because his information concerns Brazil and Vietnam, not Britain B. Not very reasonable, because it indicates a misunderstanding of the purpose of Fairtrade C. Very reasonable, because without the support of multinationals, Fairtrade is ineffective D. Very reasonable, because his information deals specifically with coffee

Solution: The correct answer is B. A) A is incorrect because this option confuses where the coffee is made with where it is exported to; further, Lindsey's claim is about Fairtrade in general, not just Fairtrade coffee. B) This is a Reasoning Within the Text question because it asks you to evaluate the basis or the justification for a passage claim. The answer to this question is B because Lindsey's concern is relatively narrow: it focuses on the reasons for the fall in the price of coffee, while the passage demonstrates that the purpose of Fairtrade is much broader than that, encompassing the participation of workers in cooperatives, environmental standards, and other priorities. Thus, Lindsey's point about price alone hardly demonstrates that Fairtrade is a "dead end." C) C is incorrect because neither Lindsey nor the author suggests that Fairtrade requires the support of multinationals, so whether or not it garners that support is irrelevant. D) D is incorrect because Lindsey's claim is about Fairtrade in general, not just about coffee.

The views of R. Brian Ferguson (final paragraph) are probably most similar to those of researchers who: A. link warfare with high rates of murder and theft. B. believe that warfare causes parents to encourage aggression in their children. C. contend that there is a genetic tendency toward physical violence. D. disagree with the Embers' findings.

Solution: The correct answer is B. A) A is incorrect because those researchers who link war with a high rate of murder or theft see warfare as more likely to emerge among societies with a high rate of interpersonal aggression. That is, they see aggression as in part causing war. Ferguson's view—that warfare is not "natural"— is more in line with that of the Embers, who maintain that it is war that can cause or promote aggression, not the other way around. B) This is a Reasoning Within the Text question because it asks you to evaluate the claims of Ferguson and apply them to a related conclusion. The answer is B because in paragraph 7 Ferguson argues that, in wartime, leaders must often "paint the enemy as inhuman . . . to motivate people to kill . . . many soldiers come out . . . with severe psychological aftereffects." This in turn could lead parents to encourage aggression (a pattern described in paragraph 6). C) C is wrong because Ferguson states in paragraph 7 that "we need to dispense with the idea that people love violence and are doomed to fight." D) D is incorrect because the Embers' theory of war and aggression are similar to Ferguson's; for example, in paragraph 6 the Embers' study found that "when these societies lose wars...harsh child-rearing methods diminish sharply."

Of the following arguments for higher wages, which one would most likely have been proposed in the late 1920s? A. One based on the importance of equality and fairness B. One based on the importance of consumers' ability to pay for products C. One based on the importance of a minimum standard of living D. One based on the importance of providing food and shelter to the needy

Solution: The correct answer is B. A) Although "ethical" calls for higher wages were made prior to World War I, this was not the predominant mode of argument used in the time referred to in the question. B) This is a Foundations of Comprehension question because it tests your understanding of passage content. B is the answer because, as described in the first paragraph, "as the 1920s progressed, union leaders and their liberal allies offered a different justification [for higher wages]: rising industrial productivity would require increasing working-class 'purchasing power' to provide an expanded market for the abundance of goods" now being produced. C) The focus on a minimum standard of living was a predominant emphasis before the late 1920s. D) This might be allied with ethical demands (which is incorrect, as explained in A), and it might be allied with arguments based on standards of living (which is incorrect, as explained in C), but in either case, it would be an argument made prior to the late 1920s.

The passage suggests that biologists were influential in persuading the United States government to: I. destroy stockpiles of biological weapons. II. abandon programs of biological weapons development. III. better utilize existing biological weapons rather than rely on new ones. A. I only B. I and II only C. I and III only D. II and III only

Solution: The correct answer is B. A) Although option I is directly supported by the last paragraph of the passage, option II is also correct, so this option is incomplete. B) This Comprehension question tests understanding of information that is explicitly stated in the passage. In the last paragraph, the author describes the "unilateral abandonment of all development of biological weapons, the destruction of our weapon stockpiles, and the conversion of our biological warfare laboratories to open programs of medical research." Options I and II are clearly supported here. The passage describes "converting" weapons laboratories to support different kinds of research, not "better utiliz[ing]" the weapons themselves, so option III is not supported. C) Option I is correct and supported by the passage; option III is not. D) Option II is supported by the passage; option III is not.

Which of the following, if assumed to be true, would provide the best evidence to support the author's conclusion about how government should promote health? A. Making the presentation of nutritional information on food packaging mandatory was a step in the right direction. B. America's obesity rate was far lower back when nutrition was largely a parental responsibility. C. Most public health officials support some government involvement in nutrition policy. D. Government efforts to reduce smoking rates in the U.S. have been quite effective.

Solution: The correct answer is B. A) Although the author might agree with aspects of this claim, as food labelling arguably helps individuals make informed dietary decisions, it does not directly support the author's stance—and the passage also implies that the author might well disapprove of the "mandatory" component of the labeling. B) This Reasoning Beyond the Text question asks you to determine the effects on the author's argument of a piece of evidence that is not discussed in the passage. The author says, "The best thing government can encourage Americans to do on the health front may well be to develop their own diet and exercise programs" (paragraph 7), advocating an individualistic approach over one in which the government provides guidance, recommendations, or regulations. So if America's obesity rate was indeed lower when nutrition was seen as a parental responsibility (not a government responsibility), that would support the author's conclusion. C) The author leans away from government involvement in nutrition policy in paragraphs 6 and 7. So this option, which describes expert support for government involvement in nutrition policy, does not support the author's stance. D) In paragraph 6, the author labels government anti-smoking efforts as "paternalistic," and the passage takes an anti-government stance generally in its approach to nutrition policy. So claiming the efficacy of government efforts to reduce smoking would contradict, rather than support, the author's stance.

Based on passage information, one can infer that implementing the author's program for conservation (final paragraph) would most likely include suggestions for: A. limiting the use of computer images in architectural models. B. discouraging the use of photography in magazine advertisements. C. increasing police use of cameras to monitor suspected criminals. D. decreasing government support of art that might offend the public.

Solution: The correct answer is B. A) Architectural models are plans or blueprints for buildings rather than pictures of actual buildings. Therefore, the author would not be likely to single out architectural design as an area in which to implement a conservation program. B) This is a Reasoning Beyond the Text question, because it asks you to apply passage information to a situation outside of the passage. B is the answer, because it suggests "discouraging the use of photography," and not only that, discouraging its use in advertising, which, unlike architectural models, is emphasized in the passage. That said, it's not the use of photographic images per se that is the problem, according to the passage, but the production of new images; if advertising used the same images, if it didn't constantly replace images (paragraph 5), the problem described by the author would be lessened. C) This option describes an increase in images and that would only increase the problem, as the author defines it. D) Decreasing the support of art doesn't necessarily decrease the production of art. Moreover, the issue in the passage argument pertains to the problem of too many images, not to those with potentially offensive content.

What finding from other research would be most apt to raise questions about the conclusion reported in the passage? A. Actors report feeling more confident before an enthusiastic audience than before a passive one. B. Musicians usually perform more skillfully when a friend is present than when they are alone. C. People do not normally invite nonsupportive persons to witness their public performances. D. The presence of family members seldom affects the performance of household tasks.

Solution: The correct answer is B. A) Experiment 1 shows that participants were "not aware of the effect of the observer on their performance." The actors' reports of their feelings of confidence in front of an enthusiastic audience, then, would not challenge the passage conclusion, because the actors could well be wrong. B) This is a Reasoning Beyond the Text question because it presents as options hypothetical scenarios not discussed in the passage and asks you to consider how those scenarios would affect the claims made in the passage. The answer to this question is B because the conclusions in the passage say that an audience of friends impairs performance on "difficult, skill-based task[s]" (paragraph 5). The finding described in B suggests that musicians perform better on such tasks in the presence of a friend and thus raise questions about the research conclusions. C) This option is irrelevant to the research conclusions. D) The research focuses on "difficult, skill-based task[s];" household tasks may be unpleasant, but they do not usually involve the degree of difficulty described here, so this option does not raise questions about research conclusions.

Which of the following entities are LEAST likely to be seen at a voodoo session? A. Houngans B. Loas C. Hounsis D. Mambos

Solution: The correct answer is B. A) Houngans are priests, who would be seen at a voodoo session, as described in paragraph B) This is a Humanities passage that falls under the content category of Religion and addresses Studies of Diverse Cultures. This is a Comprehension question, because it depends on your understanding of passage information about the meaning of several different terms. The answer is B, because loas are "divine beings who inhabit an intermediate realm between humans and a higher supreme life force" (paragraph 2). They are unlikely to be visible at a voodoo session, although their presence is very important to participants. B) Hounsis are the aspirants, who would be present and visible at a voodoo session, as described in paragraph 2. D) Mambos are priestesses, who would be seen at a voodoo session, like the Houngans.

The artificial language Esperanto was constructed from European languages and uses Western European alphabets. Based on the passage, this language would be most likely classified as: A. ideographic. B. phonetic. C. morphemic. D. a combination of ideographic and phonetic.

Solution: The correct answer is B. A) Ideographic languages do not have alphabets and express ideas without "letters, words, or sentences" (paragraph 5). B) This is a Reasoning Beyond the Text question because it presents a hypothetical scenario not discussed in the passage and asks you to apply passage information to that scenario. B is the answer because according to the author, the broad category of alphabetic scripts, like those in the European languages that are the basis for Esperanto, are called "phonetic," (although they are composed of both phonemes and morphemes, based on the passage definition of these). C) Although the European languages use morphemes (as defined in paragraph 2), the broad categorization for such languages is defined as "phonetic" (last paragraph). D) Ideographic languages would not have the alphabets that are included in Esperanto.

Suppose that a psychologist is interested in the performance of trial lawyers. On the basis of Experiment 1, the psychologist should predict that a legal argument will be more effectively presented if: A. the lawyer is serving without compensation than if the case involves a large financial settlement. B. the judge is unknown to the lawyer than if the two have a cordial relationship. C. the courtroom is empty than if it is filled with spectators whose sympathies are unknown. D. jurors watch the trial through a one-way mirror than if they are present in the courtroom.

Solution: The correct answer is B. A) Nothing in the results of Experiment 1 speak to the effects on performance of financial motivation or incentive. B) This is a Social Science passage that falls under the content category of "Psychology." This Reasoning Beyond the Text question asks you to use passage arguments to analyze a hypothetical situation not discussed in the passage. The answer to this question is B because the results of Experiment 1 show that the presence of a friend (or a "supportive audience") has a negative effect on one's performance on a stressful task when compared with the effect on one's performance by a stranger. Making a legal argument before a judge would likely qualify as a stressful task, so results would predict that a lawyer would present his or her argument more effectively to an unknown judge than to a judge with whom the lawyer has a cordial relationship. C) Experiment 1 does not evaluate the performance effects of the lack of an audience or of an audience with an unknown relationship to the participant. D) The experiment does not evaluate the difference in the effects on performance depending whether the observer is watching through a mirror or present in the room with the performer.

When using the term "the sovereignty of the masses" (paragraph 4), the author seems to mean the ability of the masses to: A. vote in elections that decide who their leaders will be. B. make every decision concerning their governance. C. discuss all the potential consequences of specific actions. D. understand the difference between popular orators and natural leaders.

Solution: The correct answer is B. A) Option A refers to what the passage calls "indirect" democracy, and "the sovereignty of the masses" is another way of describing "direct" democracy. B) This is a Comprehension question, as it asks you to determine the meaning of a phrase as it is used in the context of the passage. The answer is B, because the author first refers to the "mechanical and technical impossibility of [the] realization" of the "sovereignty of the masses" (paragraph 4). He later states that some "technical and administrative" factors "render impossible the direct self-government of large groups" (final paragraph), and it is clear that this is another way of stating "the sovereignty of the masses." C) The author claims that very large groups do a poor job of "serious discussions or thoughtful deliberations," and there is no indication that "sovereignty of the masses" suggests that large groups are indeed able to engage in this kind of deliberation. D) The author suggests that large groups tend to be susceptible to popular orators, so it is unlikely that he implies that "the sovereignty of the masses" suggests that the masses can see popular orators for what they are.

Suppose that the ruins of a sixteenth-century monastery are discovered on a remote island off the Swedish coast. Why, according to passage information, is this discovery surprising? A. Few European pilgrims would have been able to reach the site. B. An eremitical motive would have been unlikely at that period. C. The influence of Olav Trygvasson did not spread to Sweden. D. Monks would have been unlikely to learn of such an island.

Solution: The correct answer is B. A) Remote, inaccessible locations are described throughout the passage as attractive to religious pilgrims. These communities, by definition, were not meant to be hospitable to large numbers of pilgrims. B) This is a Social Science passage that falls within the content category "History." This is a Reasoning Beyond the Text question, as it describes a scenario not discussed in the passage and asks you to apply passage information to that scenario. The answer is B because the passage says that the eremitical movement flourished "until the fourteenth century." After that, "all across Europe, the . . . oratories of hermits were superseded by . . . urban cathedrals" (final paragraph). So the scenario described in the question would be surprising because it would be anachronistic. C) The passage does not suggest that Trygvasson's influence was necessary to establish a monastery or the impulse towards a remote religious settlement. The passage simply says that his conversion ended the attacks on Skellig Michael by Vikings. D) Again, the passage describes remote locations as deeply attractive to monks, mentioning the "many monastic communities on the wilder coasts of Europe" (paragraph 3).

In order to defend poets from the charge that they were liars, Sidney noted in paragraph 6 that "a maker must imagine things that are not." Sidney's point is that: A. a true poet must possess a powerful imagination. B. in order to create something, one must first imagine it. C. poets are the most creative people in our society. D. imagination is not a gift unique to poets, but is possessed by all creative people.

Solution: The correct answer is B. A) Sidney's concern here is not to define what makes a "true poet." B) This Comprehension question tests your understanding of passage information in context. The passage explains that Sidney is defending poets from the charges that they are liars. He calls them "makers," meaning creators of something new, and suggesting that makers "must imagine things that are not" implies that they must imagine things that do not yet exist in order to "make" or "create" something. C) Sidney does not say anything about the creativity of poets relative to other people. D) Sidney does not focus on the gifts that all creative people share.

Based on the passage, during the 1919 strike, which aspect of acting were actors able to make use of that was NOT available to most other industrial workers? A. Their role as "weavers of dreams" B. Their role as commodities C. Their role in producing an object in the culture industry D. Their role in the "obliteration of the factory"

Solution: The correct answer is B. A) The actors' role as "weaver of dreams" would tend to focus only on the aspect of their profession that is consumed—not on their role as producers. B) This is a Humanities passage that falls under the content category Theater. This is a Comprehension question, which tests your understanding of a paraphrase of passage material. The answer is B, because the author writes that, in the 1919 strike, actors (or "the denizens of the U.S. stage") were "able to exploit their commodity status" (paragraph 2). The author explains that consumer culture functioned to obscure the labor that goes into making a commodity available for purchase or consumption. Actors were different from other industrial workers because their labor is what the audience also consumes, and the author maintains that in the strike of 1919, the actors were able to take advantage of their unusual position as both the producers of theater and as what is consumed by the audience. C) This is not what differentiates actors from other workers; other producers in the culture industry might not themselves be what is also consumed. D) "The obliteration of the factory" is another way of talking about the divide between production and consumption created by the "culture of consumption" In the strike of 1919, Ewen argues, actors resisted that very divide, instead highlighting their position on the production side of the equation.

According to the passage, a fan of William Faulkner who buys a pipe that allegedly belonged to Faulkner should: A. accept the authenticity of the pipe without question. B. realize that the purchase has nothing to do with appreciating Faulkner's work. C. investigate the authenticity of the pipe thoroughly before agreeing to purchase it. D. preserve the pipe as carefully as a saint's relics

Solution: The correct answer is B. A) The authenticity of the pipe is not really relevant here, but as the question suggests that the pipe "allegedly belonged to Faulkner," and the passage contains the story of the fraudulent locks of Stevenson's, the passage certainly does not support this option. B) This is another Reasoning Beyond the Text question, as it asks you to apply passage logic to a hypothetical scenario not discussed in the passage. The answer is B, because the passage author argues explicitly that relics of an author's life have nothing to do with appreciating or understanding that author's work ("only the work can really explain the work," she says in the final paragraph). So the fan of Faulkner who has purchased a pipe that is supposed to have once belonged to Faulkner should recognize that this souvenir will offer no insight into Faulkner's work. C) Again, the authenticity of the pipe is not the author's concern; she argues that souvenirs (such as the pipe) offer no help in appreciating an artist's work. That comes only from immersing oneself in the work, according to the author. D) The author rejects the equation of artists and saints, so this option is not supported.

According to the passage, an image is a versatile tool that: A. is always visual, never abstract. B. can be either abstract or visual. C. is always abstract, never visual. D. is neither visual nor abstract.

Solution: The correct answer is B. A) The author explicitly states that "image" in this passage does not refer only to a visual image. B) This Comprehension question assesses your ability to understand the definition of a word in the specific context of the passage. Although the word "image" usually has a visual connotation, in this case, the passage author makes clear that he or she is using the word in a broader sense. Writing in paragraph 3, the author says that the passage uses the word "in a wide meaning, which does not restrict it to the mind's eye as a visual organ. An image in my usage is what Charles Peirce called a sign, without regard for its sensory quality." The passage goes on to talk about words as images as well; it is clear, then, that an image can be either visual or abstract in the context of the passage. C) The author does not restrict his or her definition of "image" to the non-visual. D) The passage suggests that it can be either of these, rather than neither.

What evidence does the author use to support the contention that Renaissance scholars misunderstood Egyptian writing? A. Egyptian writing was ideographic. B. Kircher incorrectly translated an inscription about Psamtjik. C. Missionaries and explorers discovered Chinese and Mayan scripts that were ideographic. D. Kircher was the first scholar to correctly understand Egyptian writing.

Solution: The correct answer is B. A) The author makes clear that Egyptian writing was misunderstood, first by the Greeks and then by Kircher and the Renaissance, as ideographic, but was eventually recognized "as both phonetic and morphemic" (paragraph 6). B) This is a Foundations of Comprehension question because it asks you to establish the author's purpose in providing a specific piece of evidence. B is correct, because after calling Egyptian writing "allegedly ideographic" (paragraph 4), the author goes to say that Kircher was a "leading scholar and scientist of the Renaissance," who "popularized" this misunderstanding of Egyptian writing. The author then notes that Kircher translated one inscription inaccurately (he translated an inscription that "actually spelled out the titles of Pharaoh Psamtjik, as instructions on securing divine protection from the god Osiris" (paragraph 5)). This clearly supports the notion that Renaissance scholars, such as the influential Kircher, misunderstood Egyptian writing. C) The author says that missionaries and explorers "reported" that Chinese and Mayan scripts were ideographic, not that the scripts actually were ideographic. The author goes on to make clear that although these understandings of Chinese and Mayan writing were persistent, they were incorrect, and that "eventually, both Egyptian and Chinese scripts were recognized as both phonetic and morphemic" (paragraph 6). D) The author explicitly describes Kircher's mistranslation and positions Kircher's view as one that needed to be overcome by the eventual recognition that Egyptian writing was both "phonetic and morphemic."

Suppose the author were asked to advise a newly independent nation about the form of government it should establish. Based on the passage, the author would probably recommend: A. a monarchy, in which decisions are made by one supreme leader. B. a representative democracy, in which decisions are made by a limited number of elected representatives. C. a direct democracy, in which decisions are made by all the adults in the country. D. an oligarchy, in which decisions are made by a small group of capable individuals.

Solution: The correct answer is B. A) The author wishes to avoid "oligarchy" and the sway of a powerful "orator" (first paragraph), so it is clear that he prefers democratic values over the reign of a supreme leader. B) This is a Reasoning Beyond the Text question, because it asks you to apply reasoning from the passage to a scenario that is not addressed in the passage. The answer is B because the passage focuses on the problems inherent in a direct democracy, in which "the masses" would govern themselves entirely. The author worries about the susceptibility of crowds to passionate enthusiasms and the tendency of very large groups to avoid "serious discussion or thoughtful deliberations" (first paragraph). Based on the author's preference for smaller assemblies rather than larger ones (paragraph 3) and his concern for the impracticality of huge assemblies—coupled with a dislike of "oligarchy" or domination in any form, the author would certainly recommend option B. C) Much of the passage is devoted to discussions of the intellectual and practical disadvantages of the political efforts of very large groups, so the author would not recommend a direct democracy. D) In the first paragraph, the author mentions the problem of failing to "guarantee against the formation of" an oligarchy, so this is not the form of government that he seeks.

The passage would be most appropriate in a collection of articles for: A. historians of sixteenth- or seventeenth-century literature. B. students of contemporary communication theory. C. scholars of the history of the English language. D. fans of the fiction of Ernest Hemingway.

Solution: The correct answer is B. A) The discussion of sixteenth- and seventeenth-century readers is fleeting and functions primarily to illuminate Hemingway's authorial strategies. There is little in the text for historians of sixteenth- and seventeenth-century literature except for the comment that readers from this historical period would be confused and disoriented by the lack of detail in Hemingway's texts. B) This is a Reasoning Within the Text question because it looks at the purpose of the passage, its appeal, and, perhaps ironically, its implied audience. The answer is B, because the passage is really about the interaction between the author and an implied reader, and the "game" in which authors engage readers. The passage is a relatively abstract and theoretical demonstration of the notion that there are "rules" that structure the relation between the author and the "mock reader," who is constructed through the author's language use (such as the omission of indefinite articles) and is not a real person. The concluding paragraph of the passage talks about the possibility of writing a history of literature that focused on the ways audiences are imagined and inscribed into a text. All of this suggests that the passage would be of interest to scholars of contemporary communications theory, as the terms used to talk about the Hemingway novels could be applied to other communicative situations. C) The passage is focused on a very specific aspect of the English language—the ways literary authors imagine and address their audiences. The final paragraph suggests that a "history of literature" could be written from this vantage point; this is not the same as saying that a history of the language could be written this way. D) Hemingway's fiction is not the primary focus of the passage; the primary focus is the importance of an imagined or fictional audience in modern literature and the ways a reader learns the terms of the "game" he or she plays with the author of a text. Hemingway's work is used as an example to illustrate this process; in itself it is not the focus of the passage.

In the context of the passage, the statement in paragraph 7, "if thereby we die a thousand deaths, that is the price we pay for living a thousand lives," is most likely meant to suggest that: A. we must guard against using our imaginations toward destructive ends. B. although imagination sometimes causes pain, its positive aspects outweigh its negative ones. C. it is possible to be too imaginative for one's own good. D. without imagination, the uniquely human awareness of death would not exist.

Solution: The correct answer is B. A) The passage is largely a celebration of imagination, and the quotation does not support a reading that is primarily about the threats posed by the imagination. B) This Comprehension question asks you to draw on passage context in order to determine what a particular statement means in that context. Here, the passage is a celebration of imagination and claims that imagination is what makes us human. Thus, in this context, the statement suggests that the price we pay for imagination is worth the toll that it may exact. C) The passage does not present this quotation in the context of the risks or dangers of imagination. D) While the quotation talks about "dying a thousand deaths," it is not referring to literal death; further, the passage does not link imagination to the human awareness of death.

According to information in the passage, the best way to conduct an argument with someone from a culture that views an argument as a dance would be to: A. methodically attack the person's weaker claims. B. calmly respond to the person's points. C. explain that arguments are not like dances. D. irrationally defend your position.

Solution: The correct answer is B. A) The discussion of the culture that views an argument as a dance contrasts that view with one that sees an argument as a "war." The approach described in this option ("attacking" the other viewpoint, as if the other person was an "opponent") is based on the military metaphor, not the dance metaphor. The passage discussion of the way that metaphors shape our conceptual schemas makes clear that this approach would be inappropriate in this culture. B) This Reasoning Beyond the Text question asks you to take a passage claim and apply that to a situation or scenario that is not discussed in the passage. The passage uses the example of a culture that conceives of an argument as a "dance" to demonstrate how differently people might behave in an argument if they saw arguments this way (rather than in a culture that saw arguments as a "war," as people in the U.S. tend to). A calm, non-oppositional response to the points made by the other person would be most consistent with the passage discussion of the way argument might operate in this culture. C) The point of this example is that arguments are not one way or another; we understand them differently in part based on the metaphors we use to talk about them. So the passage does not provide support for this approach. D) Again, to "defend" one's position (especially without reason) is to operate in the conceptual schema of argument as "war," which is not appropriate to this culture.

The author claims that on the question of developing biological weapons, biologists were: A. generally supportive. B. generally opposed. C. split nearly evenly. D. unanimously opposed.

Solution: The correct answer is B. A) The first paragraph clearly counters this claim. B) This question tests comprehension of explicit passage material. The first paragraph says that "biologists, with rare exceptions, never pushed the development of biological weapons." Thus, biologists are "generally opposed" to developing these weapons. C) The first paragraph makes clear that a majority of biologists were opposed to these weapons (thus the "rare exceptions" to those opposed). D) Because the first paragraph mentions "rare exceptions" to this position, biologists can't be seen as "unanimously opposed" to developing biological weapons.

The author's analysis of image-making rests on the assumption that a capitalist society separates: A. advertising from entertainment. B. private perception from social order. C. diseases that afflict society from their cures. D. free political choice from free economic consumption.

Solution: The correct answer is B. A) The fourth paragraph says a capitalist society "needs to generate images of new commodities" [which refers to advertising] and "forms of entertainment in order to stimulate buying." Both are used in order to stimulate buying; it's not that the first, advertising, is used to stimulate buying, and the second is used to amuse; thus, the capitalist society does not separate the two, according to the passage author. B) This is a Comprehension question, because it asks you to recognize an implicit assumption on which a passage argument is based. The answer is B. In paragraph 3, the author distinguishes the aesthetic approach from the instrumental approach to image-making. The first is matched up with private perception ("Cameras capture and 'fix' impressions from that mythical space known as private perception"); and the second linked to the social order ("cameras also arm vision in the service of power - of the state. . . ."). In the next paragraph, this dichotomy is described as a capacity to "subjectify" reality and to "objectify it." C) "Cameras are both the antidote and the disease" (paragraph 5) is the only reference to illness in the passage and in this metaphor the camera paradoxically acts as both. But it's not the capitalist society that separates them, and the analysis doesn't depend on the separation. D) "The freedom to consume a variety of images and goods is equated with freedom itself" (paragraph 4); therefore, these two are NOT separated.

According to the passage, if a speaker says, "I've never won an argument with him," he or she is most likely thinking that: A. arguments are violent. B. arguments are like contests. C. conceptual systems are metaphorical. D. competition is unpleasant.

Solution: The correct answer is B. A) The notion of "winning" an argument does not necessarily imply that the contest was a violent one. B) This Reasoning Beyond the Text question asks you to apply the passage discussion of the significance of the "argument is war" metaphor to a situation that is not specifically described in the passage. The passage argues that the "argument is war" metaphor does not only shape how people talk about arguments, but how they understand (and conduct) arguments. To describe an argument as something that one "wins" or cannot seem to "win," suggests that one is thinking of an argument as a kind of contest. C) While this is a point that the passage author makes, nothing about the example here suggests that this is what the speaker is thinking. D) The example does not imply anything about the pleasantness or unpleasantness of the competition, nor does the passage.

Based on the passage, it is reasonable to conclude that a nation's war effort is significantly affected by the: A. development of war-related technology. B. attitudes and perceptions of its leaders. C. ability to quickly mobilize troops and supplies. D. establishment of a morally correct national position.

Solution: The correct answer is B. A) The passage does not focus on technology of any kind as an important element in a war effort. B) This is a Reasoning Within the Text question. From the beginning of the passage, the author suggests that the leaders (and decision-makers) of the British military played a decisive role in the British defeat. Their failures included a lack of a strategic plan, complacency, and a preoccupation with internal politics rather than an effort to achieve unity in the military and in the nation. All of these faults can be subsumed under the heading "attitudes and perceptions," and the passage focuses on failures of leadership specifically, making this the correct option. C) Although the passage does discuss the inefficient, protracted decision-making among the British leadership, the inability to quickly mobilize forces and supplies is not specifically discussed in the passage. D) The morality of a war effort is not addressed here as a factor in the success of that effort.

The assertion that reducing class sizes requires more teachers (first paragraph) plays which of the following roles in the passage? A. It establishes that smaller classes are the most expensive way to improve education. B. It provides an explanation for the substantial costs of class-size reductions. C. It is a conclusion supported by the observation that class-size reductions are quite expensive. D. It establishes the need for more controlled studies on the cost-effectiveness of smaller classes.

Solution: The correct answer is B. A) The passage does not say that class-size reductions are the "most expensive" way to improve academic performance. It does say that they are expensive and that their cost "can dwarf" the cost of other interventions, but it does not support this option. B) This is a Reasoning Within the Text question, because it asks you to evaluate the role played by a particular passage claim. The answer is B, because the passage reference to the greater number of teachers required in order to reduce class size comes right after the point that class-size reductions cost "plenty," and implicitly explains what makes them cost so much. C) The author does not conclude that reducing class size requires more teachers as a result of observing that reducing class size is expensive; rather, the cost of additional teachers helps to explain the cost of the intervention. D) It is not the number of teachers required to reduce class size that establishes the need for more data on the cost-effectiveness of smaller classes. It is the cost of the interventions, relative to their effectiveness, as discussed in the passage.

The passage suggests that the presence of human remains, tools, and animal bones at a single location means that: A. bison and other animals migrated from one place to another. B. communal tasks were performed at the site. C. erosion has not yet occurred at the site. D. extensive interactions occurred among bands of Paleoindians.

Solution: The correct answer is B. A) The passage does not suggest that the game animals migrated from site to site. B) This Reasoning Within the Text question asks you to consider the author's logic or reasoning. In describing the San Luis site, the author says that the large number of artifacts at this site "has enabled archeologists to study a wide variety of pointed and other tools, as well as skeletons of the hunters and their prey." There are, then, human remains, tools, and animal bones at this site. In paragraph 5 the author writes of the site that "there are clusters of tools and bones, indicating the remains of a hearth around which domestic activities took place" (paragraph 5). The author clearly takes these "clusters of tools and bones" as evidence of a site where communal tasks were performed. C) As the passage describes significant erosion at the San Luis site, which fits the description in the question, it is clear this is not the case. D) Again, this is explicitly contradicted in paragraph 3.

Based on the third paragraph of the passage, with which of the following statements would the author be most likely to agree? A. Change is good, but only when it is accompanied by knowledge. B. Change and variety are required for history to exist. C. Those who do not learn from history are condemned to repeat it. D. Randomness in the cultural fabric gives life deeper meaning.

Solution: The correct answer is B. A) The passage does not support this claim. The passage focuses on the question of what makes history interesting—not what makes history a positive social force. B) This is a Reasoning Within the Text question because it asks you to consider what ideas are consistent with the author's reasoning in a specific part of the passage. B is the answer because the third paragraph describes an environment in which nothing changes: "all the magnets would be pointing in the same direction and would rarely if ever depart from it . . . your world would have fixed laws and unending peace and uniformity." As a result, concludes the author, "there would be no history at all, since a record of unbroken sameness is not history but the lack of it." If sameness means that there is no history, in the author's view, then the author would believe that history requires a dynamic or changing state. C) Again, this is not supported in the passage. The passage is not about the social utility of the study of history; it is about what makes history compelling to study. D) The passage suggests that a truly random world, with "no shred of continuity," (paragraph 4) would be boring. It does not suggest that this randomness would provide "deeper meaning."

According to the passage, it is true of King George III that he: I. had trouble making up his mind. II. wanted conquest but had no plan. III. had a detailed plan of how to win the war. A. I only B. II only C. III only D. I and II only

Solution: The correct answer is B. A) The passage explicitly contradicts this in paragraph 3, which says that George III "had no trouble making up his mind." B) This Comprehension question tests understanding of material that is clearly explained in the passage. It requires you to review the description of George III to see that his mind "contained only one idea: to conquer, but not how" (paragraph 3). This is essentially a restatement of option II. If option II is true, and the King had "no plan," then option III cannot be correct. C) Option III is not supported by the passage. D) Although II is supported by the passage, I is not.

The author's explanation of adaptation difficulties among immigrants would be most challenged by the finding that: A. many report no increase in their level of stress. B. those with the lowest status report the least stress. C. the only stress reported occurs during social encounters. D. most skilled workers report finding work in their field.

Solution: The correct answer is B. A) The passage focuses on the relative stress experienced by specific groups of immigrants, especially those with the lowest status; thus, a report that many immigrants do not report an increase in stress would not conflict with the basic argument. B) This Reasoning Beyond the Text question asks you to consider a piece of evidence that is not discussed in the passage and consider how it would affect passage arguments. The author describes a consistent finding that stress is inversely related to socioeconomic status; those with lower status report higher stress. The scenario described in the question, then, makes the opposite point and would directly challenge passage arguments. C) The passage does not focus on social encounters. It does talk about the inadequacy of "social-stress" models of immigrant adjustment, but this particular claim is not directly relevant enough to the argument to challenge it directly. B is a much better answer. D) While the passage does describe some skilled workers who are unable to find work in their fields, this is not central to the author's "explanation of adaptation difficulties among immigrants." Thus, this statement does not challenge the author's basic argument that immigrants with the lowest status report the most difficult adjustments.

Suppose that a long-dormant ballet is being revived. Based on the passage discussion of how operas were edited for Callas, one can most reasonably infer that the passage author would prefer changes to the ballet that: A. make it easier to perform. B. retain its original style. C. modify its form. D. help to "keep the action moving."

Solution: The correct answer is B. A) There is no implication in the passage that the author values ease of performance; in fact, the author praises Callas for the "musical complexity" that is part of her approach. B) This Reasoning Beyond the Text question asks you to apply the passage author's ideas to a hypothetical situation that is not discussed in the passage. In this case, the passage author critiques the "cuts" and other changes made to operas revived for Maria Callas, charging that these works "were eviscerated, rearranged, and even recomposed to a point that the hand of the composer was sometimes scarcely perceptible." He or she also remarks that the changes to the music "ruined the phrase structure and obscured the original character of the music." It is clear (when the author calls Callas's approach to such cuts "ill-informed, for instance") that the author values that "original character." Thus, you can conclude that the author would prefer that a ballet revival similarly "retain its original style." C) Based on the passage author's critique of the changes to the operas that "radically compromise the composers' styles and forms," it is clear that modifying the original form of a piece is not the author's highest value or goal. D) Although the author complains that the cuts to the operas revived for Callas do not actually keep the action moving as they are intended to, the author does not indicate that this is a particular priority for him or her.

What does the passage suggest about the use of blue paint for a house in the sixteenth century? A. It would have been impossible. B. It may have been a statement of wealth. C. The paint would have faded quickly. D. The original paint may have been titanium-white.

Solution: The correct answer is B. A) There is no suggestion that it would have been impossible to paint a house blue in the sixteenth century—it just would have been expensive. B) This is a Humanities passage that falls under the content category of Art History. This is a Reasoning Beyond the Text question, as it asks you to use passage information to answer a question about a scenario that is not discussed in the passage. The answer is B, because the passage refers to a development in the seventeenth century, when a particular kind of blue pigment was developed "as a cheaper and more workable alternative to the expensive pigments azurite and smalt" (paragraph 2). Thus, you can infer that, in the sixteenth century, it was still expensive to produce blue paint; in these circumstances, using blue paint might well have been a sign of wealth. C) The passage says that brightly colored pigments are "stable"; it does not suggest that the sixteenth century paints, containing more expensive pigments, were likely to be less stable (or more likely to fade). D) The passage does not support this suggestion.

The author's reasoning implies that it would be folly to do which of the following with a budding conceptual artist? A. Connect her with an agent who can take care of the business end of her career B. Subject him to a thorough academic education in his art C. Discourage those who advise her to "pace herself" D. Discourage him from keeping a diary

Solution: The correct answer is B. A) There is nothing in the passage to suggest that this would be folly. B) This is a Reasoning Beyond the Text question because it asks you to apply passage arguments to a scenario not discussed in the passage. The answer is B because, as described in the passage, conceptual artists find success by "creating new art forms with little regard for, or knowledge of, traditional rules and constraints" (paragraph 4), so exposing these artists to such a body of knowledge would indeed be folly. C) Based on the passage, conceptual artists seem to flourish when young, then "deteriorate with experience" (final paragraph). So, if these artists were encouraged to pace themselves, they would miss their critical period of productivity. Thus, it would not be folly to discourage those who advise such artists to pace themselves. D) Although the author describes the Beatles as turning rock and roll into "music fit for communicating autobiographical intimacies," there is nothing that suggests that it is necessary for conceptual artists to write down their personal experiences in diary form in order to create music with an introspective dimension. So there is no clear indication that this would be folly. B is the better response.

Of the following statements, which one best describes the central focus of the passage? A. Materialistic desire is both the engine of and the universal virtue for capitalism. B. In the 1920s and 1930s, new forms of economic research were used to understand the role of consumers in the economy. C. Materialistic desire is the basis for addressing perceived economic inequalities. D. Expenditure surveys helped justify arguments in favor of minimum standards of living.

Solution: The correct answer is B. A) This does not specifically emphasize the primary focus of the passage—the economists' research interest in consumer behavior and its motivations. B) This is a Foundations of Comprehension question because it asks you to identify the central concerns of the passage. B is the answer because after establishing that "materialistic desire" can be used to support liberal—or redistributive—economic objectives, paragraphs 2, 3, and 4 all focus on the movement of economists beginning in the 1920s to use empirical research to study consumer behavior. The passage describes the shift in the focus in economists' research objectives and the critiques of previous economists out of which this new focus on consumer behavior developed. The passage culminates by describing the most sophisticated such study to date—a study that was ultimately intended to support economic policies designed to increase mass purchasing power. C) This is not specific enough to focus on the economists' program or strategies. D) This is far too narrow to encompass passage arguments; it focuses only on the research that economists did before the shift in priorities described in the passage.

Given the passage discussion of Dali and the Surrealists, which of the following works of art would be most reasonably considered to be Surrealist? A. An indistinct painting of water lilies that only becomes clear from across the room B. A model of a telephone that substitutes a lobster for the receiver C. A large, moving sculpture with circles spinning within circles D. A lithograph consisting of photos of a celebrity, each of which is a different shade

Solution: The correct answer is B. A) This option does not describe a work of art that is "fantastic" or reminiscent of "unconscious" states. B) This Reasoning Beyond the Text question asks you to apply the discussion of Surrealist work in the passage to novel scenarios not in the passage. The passage refers to Dali's "fantastic" imagination and to the Surrealists' "interest in the revelations of the unconscious and psychopathological states." All of this best applies to the hallucinatory image described in this option. C) This option describes an abstract image, but not one that is "fantastic" or evokes the "unconscious." D) This option does not describe a work of art that is "fantastic" or reminiscent of "unconscious" states.

Which of the following situations would be the best example of the "[u]nprecedented objects, processes, and possibilities" alluded to in the passage? A. The transformation of wild geese into tame domestic stock B. The flurry of technological innovations in the late twentieth and early twenty-first centuries C. The brutal civil wars that have historically wracked certain countries D. The wide variety of shows offered on cable television

Solution: The correct answer is B. A) This is an example of a common process; although it describes a transformation, it is one that occurs with regularity. B) This is a Reasoning Beyond the Text question because it asks you to take an idea from the passage and apply it to a series of novel options. B is the correct answer because the "flurry of technological innovations" in the contemporary period have indeed produced objects and processes that have never before existed—and that in many cases, had never been imagined. Examples abound, but tiny cell phones with internal cameras that allow you to transmit a photograph you have just taken to another tiny cell phone are one example. C) Civil wars occur in countries all over the globe, with dismaying regularity. There is nothing novel about them. D) Even if each show is in some sense distinct, the shows are variations on a theme and share generic conventions with other shows; some shows are even designed explicitly to revive or evoke an earlier show.

According to passage information, which of the following reasons was probably determinative in the selection of Skellig Michael as the site for a monastery? A. Its proximity to the shrine at Needle's Eye B. Its isolation from worldly distractions C. The protection it promised from raiders D. The opportunities it provided for suffering

Solution: The correct answer is B. A) This is described as an important attraction of Skellig Michael in the eighteenth century, centuries after the height of the eremitical movement. It is not identified or suggested as an important element in the original identification of Skellig Michael as a site for a monastery. B) This is a Foundations of Comprehension question, as it asks you to identify a central theme of the passage. B is the answer, because the passage describes Skellig Michael as most importantly remote and difficult to reach; the island "lies at the end of a seven-mile voyage across open water—a place of sheer precipices and terrifying landings" (paragraph 1). The author continues, "It is difficult to understand the monks' choice of this site (or their ability to live here); the place is fit only for birds. Access to the island by wicker boat would have been infrequent and dangerous" (paragraph 2). It goes on to say, though, that despite the unwelcoming character of the island's geography, "for nearly a thousand years[,] hermits sat on the Skellig rock and searched the sea for some sign of the Absolute." The passage identifies as a movement this search for peace and "solitude." The passage suggests, then, that those in search of the opportunity for religious contemplation seek solitude, and this is precisely what Skellig Michael offers. C) The passage indicates that Skellig Michael was actually the site of many Viking raids. It was not naturally protected from raiders. D) The passage does not indicate that the devout were attracted to "suffering," per se. Although the location is described as unwelcoming and lonely, this does not support a claim that the hermits sought suffering.

Why does the passage author assert that sixteenth- or seventeenth-century readers would have "laughed"at a Hemingway short story? Because they would have found the writing style quaint Because the plots of short stories have changed radically since then Because basic, essential details were missing A. II only B. III only C. I and II only D. I and III only

Solution: The correct answer is B. A) This is incorrect because the passage does not indicate anything about a historical change in plots. B) This is a Reasoning Within the Text question because it requires you to identify the explanation or support for a particular claim—in this case, you also have to bring together distinct parts of the passage. The answer is B because the passage explains that Hemingway treats his imagined or fictional audience as "insiders" who already know essential background details about the fiction, even though those details are not provided in the work. In the first paragraph, the author maintains that a sixteenth- or seventeenth-century audience (as opposed to Hemingway's twentieth-century audience) would have "laughed" at Hemingway's work because they "would have been totally unable to adapt to [the] demands" the work imposed on readers. Later in the passage, the author says that Hemingway's omission of certain details "implies a degree of shared information that would likely have baffled a sixteenth-century reader." So option III is supported in the passage. C) There is no support in the passage for the idea that sixteenth-century readers would have found modern fiction "quaint" (and this idea is somewhat illogical as well). D) Although option III is supported by the passage, option I is not.

A visitor to Skellig Michael who kissed its stone cross probably did so for which of the following reasons? A. To fulfill a qualification for sainthood B. To atone for wrongs committed C. To be spared by Viking marauders D. To be accepted into the monastery

Solution: The correct answer is B. A) This is not supported by the passage. B) This is a Foundations of Comprehension question because it asks you to understand the implied meaning in a passage claim. B is the answer because the passage describes "penitents . . . [who] made the grueling . . . climb to the Needle's Eye" (final paragraph), a climb which culminated in kissing the stone cross. Because the passage refers to "penitents," it is most likely that these people sought to kiss the cross in an act of atonement. C) The Viking marauders were described as active in the eighth and ninth centuries, while the pilgrims who kissed the stone cross are described as traveling to Skellig Michael in the eighteenth century. D) The monastery was no longer active in the eighteenth century; the first paragraph says that the cliff was inhabited into medieval times.

Which of the following best describes the author's attitude toward the fact that the original paint of an architectural element has typically been painted over not just once but several times in the course of history? A. It makes much of architectural paint research guesswork. B. It is what makes architectural paint research such a robust source of information. C. It indicates a difference in status between easel/wall painters and house painters. D. It is something that education about architectural paint research might prevent in future.

Solution: The correct answer is B. A) This is not supported by the passage; the repeated layers offer specific historical evidence and do not make the process "guesswork." B) This is a Reasoning Within the Text question, because it asks you to evaluate the author's position about a piece of information presented in the passage. The answer is B, because the author says that "the accumulation of successive paint layers on architectural elements is now regarded as an important archaeological resource" (paragraph 3). The author contends that the repeated application of paint finishes allows researchers to study a progressive history of the use of the "development and use of the building long after the last structural alteration has been completed" (first paragraph). C) The passage distinguishes between easel/wall painters, whose works is preserved in its original state, and house painters, whose work is repeatedly painted over, but this distinction is not presented as a matter of "status" and this is not the significance of the point the question asks about. D) There is no implication that research should "prevent" successive layers of paint from being used; the repeated layers are part of the historical evolution of the building and house paint is not designed to be preserved or conserved as artworks are.

Which of the following points in the passage is supported by a concrete example? A. Most professors find their positions through national searches. B. Hiring committees are prejudiced against hiring local candidates to teach at the same university they had attended. C. the world outside the classroom into a laboratory will blur the line between "student" and "citizen." D. Most U.S. college students attend institutions in their home states.

Solution: The correct answer is B. A) This option is not supported by a concrete example. B) This is a Reasoning Within the Text question, because it asks you to assess the nature of the evidence provided in the passage. The answer is B, because the claim that university hiring committees are prejudiced against "local candidates" is supported by an anecdote in paragraph 4, in which a job candidate was told that the department (in his home state) from which he had received his degrees would not hire him permanently because they considered him "too much a native." This is certainly a concrete example to support the claim in option B. C) This option is not supported by a concrete example. D) This option is not supported by a concrete example.

Which of the following facts most strongly supports the authors' image of John VI as resistant to social change in his realm? A. When Napoleon invaded Portugal, he fled to exile in Brazil. B. On returning from exile, he agreed to reforms but quickly revoked them. C. As soon as he returned to power, Brazil declared its independence from Portugal. D. After his return, he reigned as monarch of Portugal for only five years.

Solution: The correct answer is B. A) To assert that this option was correct, we would need to know more about conditions in Brazil - for example, whether it was like France and Spain, in culture and law, or like Portugal. B) This is a Reasoning Within the Text question, as it asks you to evaluate the strength of evidence for passage claims. The answer is B. "Reforms," especially in the context of this passage, represent clear efforts at making progressive social changes. Retraction of such measures by a highly repressive King (possibly initially consented to as the price for being allowed to return from exile) would strongly illustrate this king's resistance to change. C) Without a link being established between John and Portugal, on the one hand, and Brazil on the other, this option would be irrelevant. D) To assert this option, we would need to know why he stopped reigning as monarch after five years. This information is not included in the passage.

According to the passage, Meselson asked some generals about the details of the possible use of biological weapons in order to show that: A. biological weapons do little damage. B. biological weapons serve no reasonable purpose. C. biological weapons should only be used in retaliation. D. the U.S. needed to develop more effective biological weapons.

Solution: The correct answer is B. A) To the contrary, the passage explicitly states that Meselson was determined to highlight the risks posed by these weapons. B) This Reasoning Within the Text question asks you to assess the way a particular piece of evidence is being used in the passage. The passage says that Meselson "had to show that their belief was based on an illusion"; it then describes the questions he asked a group of generals about the circumstances under which they would use biological weapons. The generals were never able to give him a clear answer; according to the passage, there was "no answer" to these questions. Members of Congress then concluded that the biological weapons program "made no sense." Considering Meselson's intention here and the result of his questioning, it is clear that this was the purpose of the questioning. C) The author describes the main tenets of Meselson's "arguments," just before detailing his questioning of the generals. One of these is that: "biological weapons are uniquely unreliable and therefore inappropriate to any rational military mission for which the United States might intend to use them, even including the mission of retaliation in kind for a biological attack on our own people." It is clear that this option does not describe Meselson's intent. D) To the contrary, the point of the questions, as the other options show, was to illuminate the irrationality of using the weapons at all; Meselson's aim was diametrically opposed to what this option describes.

If one assumes that the ants are exhibiting teaching behavior, as the Bristol researchers maintained, the pace at which the leader ant moves would likely be influenced by: A. its sense of urgency. B. its capacity for movement. C. its estimate of the time its follower needs to learn something new. D. the relative speed of other ants performing the same task.

Solution: The correct answer is C. A) A is incorrect because a teacher's behavior is not based on its own needs, but on the needs of the follower ants (first paragraph, paragraph 3). B) B is incorrect because a teacher's behavior is not based on its own capacity, but on the capacity of the follower ants (paragraph 4). C) This is a Foundations of Comprehension question because it requires you to understand passage claims and think through the relationship of these claims to passage options. The answer to this question is C because the lesson "proceeded at a pace set by the followers" (paragraph 4). This is consistent with Franks's definition of "teaching behavior" in paragraph 2, as the leader would be modifying its behavior (at a potential cost to itself, because it could likely go faster), so that another could learn more quickly. D) D is incorrect because the passage does not indicate that a teacher looks to others to determine an appropriate speed for a task.

The principles common to the Iroquois League and the American Constitution are present in the proclamations of almost all nations. What is the relevance of this to the information provided in the passage? A. It strengthens Levy's objection. B. It reinforces the three-part distinction made by Payne. C. It challenges Grinde and Johansen's thesis. D. It extends the reach of Grinde and Johansen's thesis.

Solution: The correct answer is C. A) A is incorrect because neither Levy's objection that the parallels (between the founding documents of the Iroquois League and of the U.S. government) are inexact and nor his objection that only Barry was used for authority is strengthened by the fact that the principles are so common. B) B is incorrect because Payne's distinction among the three images of the Iroquois is not affected by the scenario described in the question. C) This is a Reasoning Beyond the Text question because it asks you to consider the relevance to passage claims of information that is not in the passage. The answer to this question is C because if the principles are as common as described in the question, this would weaken the claim that the presence of the principles among the Iroquois likely influenced their adoption by the framers of the Constitution. D) D is incorrect because some of the mentioned national proclamations would have occurred before the Native American governments existed, so it is impossible to say that this necessarily broadens the argument about the influence of the Iroquois on the founding of other governments.

According to the passage, Fairtrade certification procedures take into account all of the following EXCEPT which one? A. Social justice B. Environmental responsibility C. Market value D. Ethical behavior

Solution: The correct answer is C. A) A is incorrect because social justice is a factor in certification: "Small farmers must be organized in ... groups that allow democratic participation" (paragraph 2). B) B is incorrect because environmental responsibility is an element in certification: Fairtrade traders "must pay producers a price that covers the cost of sustainable production ... Plantations and factories can use the Fairtrade label if they . . . comply with . . . environmental standards . . ." (paragraph 2). C) This is a Foundations of Comprehension question because it tests your understanding of information presented in the passage, in this case, the elements required for Fairtrade certification. The correct answer to this question is C because market value is not an element in Fairtrade certification. Instead, the author explains, that the minimum price of Fairtrade products is set "no matter how low the market price may fall" (paragraph 2). D) D is incorrect because ethical behavior on the part of companies is required for certification: "Plantations and factories can use the Fairtrade label if they . . . do not use child labor or forced labor" (paragraph 2).

The main point of the passage is that: A. poetry dealing with war should not be considered in the same class as other poetry. B. English poetry prior to 1914 depicted war as exotic. C. Yeats was wrong to dismiss the works of the Great War poets. D. it is the duty of all poets to portray the sufferings of war.

Solution: The correct answer is C. A) A is incorrect because the author argues the opposite point, that wartime poetry should be considered as equal to other poetry (paragraphs 2, 3, 5). B) B is incorrect because although the author does describe war poetry from before 1914 as preoccupied with the exotic (paragraph 3), this is not the main point of the passage. C) This is a Humanities passage that falls within the "Literature" content category. This is a Foundations of Comprehension question because it asks you to identify the central argument of the passage. The answer is C because the author states in paragraph 5 that "Yeats could not have been more mistaken" in his assessment of certain poets who wrote about the Great War (or World War I, as we now know it). Much of the passage is concerned with making the argument for the value of this group of "really original poets" and of the importance of writing to testify about the "indescribable . . . and unthinkable" (final paragraph) war they experienced. D) D is incorrect because the passage does not imply that all poetry should be wartime poetry.

For thousands of years, theorists have proposed alternate accounts of the functions or effects of fantasized violence. Which of the following theories would be most consistent with the central point of the passage? A. The normative: violent retributions symbolically affirm important social values. B. The mimetic: fantasy violence represents what actually occurs in the real world. C. The cathartic: impulses toward real violence are dissipated through vicarious fantasies. D. The social learning: images of fictional violence teach the use of force to solve real conflicts.

Solution: The correct answer is C. A) A is incorrect because the author does not view real-life violent retribution as reflecting normative social values; the passage argues for the normalcy of the desire to interact with "turbulent content" on an imaginary level. B) B is incorrect because the author claims the opposite: that fantasy violence can be a "harmless way of sublimating . . . those normal feelings" (paragraph 5). C) This is a Reasoning Beyond the Text question because it asks you to apply theories not discussed in the passage to a central passage argument. The answer to this question is C because the author reinforces a cathartic perspective throughout the passage (paragraphs 2, 5 and 6); for example, "to give television violence its due, we need first to acknowledge the complex, sometimes violent content of our inner lives" (paragraph 2). D) D is incorrect because the author claims the opposite: that images of fictional violence may help to obviate the use of violence in real life (paragraphs 2, 5).

The hope expressed by the author in the last paragraph depends on an increase in: A. political lobbying. B. trade barriers. C. consumers' prioritization of ethics. D. consumers' disdain for high-status brands.

Solution: The correct answer is C. A) A is incorrect because the author says explicitly that the success of Fairtrade (which is his hope) does not depend on political lobbying. B) B is incorrect, as it is not supported in the passage—and the author earlier echoes Lindsey's critique of trade barriers. C) This is a Foundations of Comprehension question because it tests your understanding of the meaning of the author's words. The correct answer to this question is C. The author advocates for Fairtrade and says that "it reflects better ethical priorities" (final paragraph) when consumers are willing to purchase Fairtrade products (because they value what Fairtrade signifies enough to pay potentially higher prices for an item). The "hope" expressed by the author in the last paragraph is that the market demand for Fairtrade in Europe will soon be matched "around the world"; for this to happen, consumers would have to prioritize ethics enough to be willing to pay Fairtrade prices. D) D is incorrect, as there is no implication that consumers must devalue high-status brands per se to value Fairtrade brands. The author suggests that both consumers who buy high-status brands and those who buy Fairtrade must be willing to pay more for products that embody what they value (in one case, designer brands, in the other ethically-made products).

What does the passage imply about worker autonomy? A. Autonomy is the main job reward in large companies. B. Autonomy tends to parallel wages. C. Most workers desire autonomy. D. Few workers think autonomy is important.

Solution: The correct answer is C. A) A is incorrect because the first paragraph indicates that autonomy is a job reward of small, rather than large, organizations. B) B is incorrect because the passage is inconsistent about the correlation between higher wages and autonomy (the first paragraph versus paragraph 5). C) This is a Foundations of Comprehension question because it requires you to understand central passage claims and think through the relationship of these claims to passage options. The answer to this question is C because the first paragraph lists autonomy as a job reward. D) D is incorrect because the passage does not suggest that workers believe autonomy is unimportant; instead, it is plausible that many employees would value autonomy.

In the context of the passage, the comparison between how fast the leader could go and how fast it does go when paired with a follower ant is LEAST reliable as an index of how much: A. restraint is exercised by the leader. B. help the follower ant appears to need. C. effort the task being performed should take. D. effort is made to orient and instruct the follower.

Solution: The correct answer is C. A) A is incorrect because the leader's effort is based on the restraint they exercise (for example, they slow down or speed up) in order that the followers can learn the tasks. B) B is incorrect because the leader ants do tailor their speed, dependent on how much help the follower ants need: "the lesson was highly interactive . . . if the gap between leader and follower increased too much, the leader slowed down. If it was too close, the leader accelerated" (paragraph 4). C) This is a Reasoning Within the Text question because it asks you to consider one piece of evidence from the passage (that of the rate of speed of leader ants when teaching) and consider what kind of point this evidence could support. The answer is C because the leader ants did not perform the task as fast as it "should" be performed; rather, they "proceeded at a pace set by the followers" (paragraph 4). D) D is incorrect because in the first paragraph the author asserts that "once a follower got its bearings, it tapped the leader with its antennae, prompting the lesson to literally proceed to the next step."

The central thesis of the passage is that: A. we are basically unaware of our conceptual system. B. a culture can view argument as an aesthetically pleasing dance or as war. C. metaphors control our perceptions, thoughts, and actions. D. metaphor is a poetic as well as a rhetorical device.

Solution: The correct answer is C. A) Although the passage does say that we are not "normally aware" of our conceptual system, this is not a pervasive or consistent enough point in the passage to be called its central thesis. B) The passage describes these two alternative viewpoints on argument, but as an important example or illustration of its central thesis, not as the thesis itself. C) This Comprehension question asks you to identify the primary argument of the passage, the purpose towards which the passage's claims and examples are building. The last sentence of the first paragraph (where one often, but certainly not always, finds a "thesis statement") articulates this main argument. The rest of the passage demonstrates this point, through argument and especially example, particularly in the extended example about what it means to view an argument as a "war." D) The passage acknowledges these functions of metaphor, but claims that these are the functions that people typically attribute to metaphor (rather than the ones the author is concerned with); demonstrating their existence is not the central thesis of the passage.

Which of the following statements is most strongly suggested by Butterfield's quotation in the passage? A. Cultural transformation is what makes history so interesting. B. Major global upheavals are what make history thrilling. C. Unexpected and significant events are what make history absorbing. D. Talented people change the direction of history.

Solution: The correct answer is C. A) Butterfield talks about historical events, not necessarily cultural transformation (and such transformation would not necessarily take place in an unexpected or unpredictable fashion). B) Butterfield's quotation does not necessarily refer to a "major global upheaval," but just a shift in direction. C) This is a Reasoning Within the Text question because it requires you to understand for what purpose the author cites a particular source and the viewpoint expressed by that source. C is correct because after the author explains that historians study history because they find it fascinating, he quotes Butterfield saying that "there is something in the course of historical events which twists the course of history in a direction that no man ever intended" (paragraph 1). The author, then, uses Butterfield to explain what makes history fascinating to historians. Option C refers to the fascination of "unexpected and significant events," which best captures the idea that these events were not anticipated or calculated outcomes. D) Butterfield describes the events taking a course that "no man ever intended," so Butterfield does not suggest the idea that a "talented person" would consciously change the course of history.

Which of the following relationships is most like the suggested relationship of the birds on Skellig Michael to the monks there? A. Butterflies to entomologists in Newfoundland B. Helicopters to mountain climbers in Nepal C. Penguins to explorers in Antarctica D. Cattle to wheat farmers in Montana

Solution: The correct answer is C. A) Butterflies are studied by entomologists; as described in the passage, monks do not study birds. B) Helicopters may be used to benefit mountain climbers, perhaps to rescue the climbers or to provide supplies for them. This is not the relationship between the birds and the monks on the island that the passage describes. C) This is a Reasoning Beyond the Text question because it asks you to apply what you have read in the passage to the unfamiliar scenarios presented in the answer choices. C is the answer because the passage describes the challenge of living in the remote and inhospitable location where the Skellig Michael monastery is located: "The monastery is reached from the sea by a ladderlike stairway hewn into the rock . . . the place is fit only for birds" (paragraph 2). In other words, birds, rather than intrepid monks, are the natural inhabitants of the steep cliffs of the island. This is similar to the relationship between penguins and Antarctic explorers. D) Cattle most likely eat the grain grown by wheat farmers in Montana; the passage does not describe the birds benefitting in any way from the presence of the monks.

Which of the following assertions, if true, would most support the author's statement that "Egyptian culture dwindled under the Classical world's onslaught" (paragraph 4)? A. Greek scholars used discoveries by Egyptian astronomers to further their own studies. B. Egyptian scribes invented a flowing script for use in keeping economic records. C. Invaders from Alexander the Great to the Romans tried to integrate Egypt into their empires. D. Renaissance scholars incorrectly identified Greek script as ideographic.

Solution: The correct answer is C. A) Even if Greek scholars had used or built on the discoveries of Egyptian astronomers, this would not necessarily in any way have diminished Egyptian culture. B) This would be a reference to the flourishing of Egyptian culture, not its dwindling. C) This is a Reasoning Beyond the Text question that asks you to consider the effect on passage claims of a set of hypothetical scenarios that are not discussed in the passage. C is correct because it represents an attack (or "onslaught") on Egypt by the Classical world (including the Romans) and describes an effort to appropriate or incorporate Egypt and its culture into other empires, which would by definition diminish Egyptian culture. D) This refers to Greek script, not to Egyptian culture.

Which of the following groups played a role in conserving the older social, religious, and political order in Portugal? I. Peasants II. Freemasons III. The royal family A. II only B. III only C. I and III only D. II and III only

Solution: The correct answer is C. A) In paragraph 3, the author characterizes the "small minority" of the population "irked by the despotism of the past" as a group of students and Freemasons. Thus, proposition II is incorrect. B) Proposition III is correct, but so is Proposition I, so this option is incorrect. C) This is a Social Science passage, which falls under the content category, "History." This is a Comprehension question because it tests your understanding of passage language and argument. C is the answer because Commoners and peasants are referred to as "other guardians of the past" (paragraph 2); therefore, proposition I is correct. The opening paragraph of the passage makes clear that "Queen Maria Francisca and her son [were] dependably protective of the Church as the indispensable support of private morals, social order." Proposition III is, therefore, also correct. Option C is correct. D) Proposition II is incorrect, so this option is incorrect.

Assume that a film adaptation of a novel has authenticity, as the author claims the term is used in historical performance circles. Of the following statements about the film, which one is most likely to be true? A. The film's plot diverges markedly from the plot in the novel. B. The film uses images to reproduce the feelings expressed by the novel's prose. C. The film's re-creations of scenes from the novel lack emotion. D. The film manages to evoke a sense of humanity in the viewer.

Solution: The correct answer is C. A) In the final paragraph, the author describes the idea of "authenticity" as it circulates in historical performance circles as requiring faithfulness to the original text or score (sometimes excessive faithfulness, in the eyes of some in these circles). So this option is incorrect. B) As the author describes the way "authenticity" is used in historical performance circles, it connotes excessive faithfulness to the original, accompanied by a lack of "vitality." This option describes a situation in which the adaptation preserves the emotional content of the original, so it is incorrect. C) This Reasoning Beyond the Text question asks you to apply information from the passage to a scenario that is not related to or discussed in the passage. The passage author explains that "authenticity" is defined in historical performance circles as a performance that preserves the text (or score) of an original, but that lacks "vitality," so this description would be the most appropriate. D) A sense of "humanity" is not something that the passage particularly dwells on. Option C is the better choice.

Suppose that mainstream musicians who write songs for musical comedies get better at their art as they age. How does this affect the opinions expressed in the passage? A. It extends James Miller's observations. B. It challenges the passage author's claims about conceptual artists. C. It supports the passage author's claims about traditional artists. D. It does not affect the opinions expressed in the passage.

Solution: The correct answer is C. A) Miller merely describes the change in the nature of the Beatles' music; this information about mainstream musicians does nothing to extend that description. B) These mainstream musicians are not the kind of conceptual artists described in the passage. C) This is a Reasoning Beyond the Text question, because it describes a scenario not discussed in the passage and asks how this scenario would affect passage claims. C is the correct answer because the author writes that "traditional artists' work tends to improve with age" (final paragraph), so the circumstance of mainstream musicians described here would support the author's claims. D) It is directly relevant to passage claims about traditional artists.

The author uses the term "culture as commodity" to characterize which aspect of Haitian voodoo? I. The charging of admission at voodoo sessions II. The molding of voodoo to fit audience expectations III. The sale of voodoo trinkets and other artifacts A. I only B. III only C. I and II only D. II and III only

Solution: The correct answer is C. A) Option A is incorrect, as I is not the only correct option. B) There is no support for option III, as the passage makes no reference to the sale of voodoo-related artifacts, so B is incorrect. C) This is a Comprehension question, as it tests your understanding of (the meaning of) a passage phrase, as that phrase is used in the context of the passage. The answer is C, because both options I and II are correct, while option III is not. The author explains that the "outsider" to Haiti expects voodoo to be "almost synonymous" with the country itself. According to the author, this powerful "identification" of Haiti with voodoo leads to "culture as commodity"—or the outsiders essentially being able to get what they want from voodoo. In paragraph 3, the author describes a form of voodoo that is staged for outsiders and which the audience pays admission to watch; this supports option I. Also, in paragraph 3, the author contends that the form of voodoo witnessed at Mariani "confirms the popular expectation of 'scary voodoo'"; this supports option II. D) There is no support for option III, as the passage makes no reference to the sale of voodoo-related artifacts, so D is incorrect.

According to the passage, which of the following situations during the Renaissance supported the view that Egyptian script was ideographic? I. Kircher's claim that the Psamtjik inscription listed his titles II. Kircher's reputation as a scholar and scientist III. Missionaries' reports on Chinese and Mayan scripts A. I and II only B. I and III only C. II and III only D. I, II, and III

Solution: The correct answer is C. A) Option I is incorrect. In fact, this is not what Kircher claimed; he misread the inscription as "instructions on securing the divine protection of the god Osiris." B) Although Option III is correct, Option I is incorrect. C) This is a Foundations of Comprehension question, because it asks you to identify the author's purpose in making particular claims. C is the answer, because both Options II and III are correct. Option II is correct because the passage says explicitly that "Kircher's reputation gave credibility to claims that Egyptian script was ideographic" (paragraph 5). Kircher has earlier been referred to as a leading scholar and scientist. Option III is also correct, because right after discussing Kircher's claim, the author says: "Further, missionaries and explorers reported that Chinese and Mayan scripts were ideographic . . . provid[ing] independent proof that nonalphabetic scripts used signs conveying metaphysical ideas." So the author clearly states that ideas about Chinese and Mayan scripts buttressed claims about Egyptian script. D) Option I is incorrect, although Options II and III are correct.

Why does the author most likely mention Norway? A. To provide an exemplar of clinical practice B. To provide an exemplar of evidence-based medicine C. To provide an example of accounting for economics D. To provide an example of one drug being less effective than it seems

Solution: The correct answer is C. A) The Norwegian example does not mention clinical practice. B) The author invokes the example of Norway as an example of an aspect of patient care that evidence-based medicine "does not account for," not as an instance of evidence-based medicine. C) This is a Reasoning Within the Text question because it asks you to understand how the author uses a particular piece of evidence—to identify the argumentative point that the evidence serves. The answer to this question is C because after discussing the advantages of evidence-based medicine, the author writes of evidence based medicine: "This approach does not, however, account for other aspects of patient care, such as patient preferences, economics, and ethical issues. Norway, for instance, is discouraging the use of the osteoporotic agent alendronate; although the drug decreases hip fractures, ninety high-risk women would have to be treated with the drug for three years to prevent one hip fracture at a cost that could bankrupt the country's medical plan" (final paragraph). The emphasis on the cost of a particular treatment is, of course, a reference to economic factors in health-care decision-making. D) The Norwegian example does not involve a comparison of two different drugs. It only discusses one drug—and it is not the effectiveness of this drug that is at issue but its cost.

The statement that "if you preserve the trivial, then you must truly value the serious" (paragraph 2) functions in the passage to: A. demonstrate that the motives of souvenir hunters are superior to those of biographers. B. convey the author's belief about the motives of souvenir hunters. C. explain the way souvenir hunters justify their enterprise to themselves. D. suggest that souvenir hunting is more respectable than it might seem.

Solution: The correct answer is C. A) The author actually considers all preoccupation with the artist herself—with her biography, and with souvenirs of the author's body or life—to be equivalent. She does not suggest that souvenir hunters are superior to biographers (or vice versa). B) This statement does not provide evidence of the author's views of the souvenir hunters' motives; the author gives perhaps the clearest hint of her theories about their motives when she says, 'You may feel 'close' to a writer when you . . . examine locks of his or her hair" (paragraph 6). C) This is a Humanities passage that falls under the "Literature" content category. It is a Reasoning Within the Text question because it asks you to evaluate the function of a statement within the passage. The answer is C. The statement quoted in the question is introduced this way: "It seems like proof of proper intent: If you preserve the trivial, then you must truly value the serious." The author argues throughout that readers should value an author's words over tangible objects or "souvenirs," and clearly does not endorse the statement quoted in the question. Instead, she says that valuing souvenirs "seems like proof of proper intent." This implies that those "who love art" believe or want to believe that their preoccupation with tangible remnants of artists is evidence of their own seriousness of purpose. D) The author does not use this statement to suggest anything about the "respectability" of the souvenir hunters' motives.

The author's main purpose in exploring the aesthetic and the instrumental approaches to photography is to suggest that a new form of conservation must: A. allow ordinary people to make judgments about beauty. B. increase constantly the production of entertaining images. C. solve the problem of the continual proliferation of images. D. develop a coherent ideology to facilitate the task of governing.

Solution: The correct answer is C. A) The author asserts that "The camera empowers everyone to make artistic judgments about importance, interest, or beauty" (paragraph 2). This happens, according to the passage, each time someone decides "that would make a good picture." In other words, the author asserts that this is already happening; it is not the goal of a "new form of conservation." B) The problem, for which "a new form of conservation" is the solution, is due to "the increasing supply of photographic images" (paragraph 6). Therefore, it wouldn't make sense, for a new form of conservation to increase the production of images; instead a solution would involve a decrease in the production of images. C) This is a Social Science passage, belonging to the content category "Sociology." It is a Comprehension question, as it asks you to identify an aspect of the author's argument. The answer is C. The author postulates that the increasing supply of photographic images is a problem because it "can overwhelm and threaten to obliterate reality" (final paragraph). It threatens reality because these images do not actually depict "reality"; they are framed and positioned versions made by the picture-taker and reproduced by some form of media. According to the passage author, these second-hand, mediated images confront individuals in modern technological society, outnumbering and drowning out direct perceptions. "The remedy lies in a new form of conservation" (paragraph 6) that would reduce the number of artificial images and restore balance between what is actually seen with the naked eye and what is staged or manufactured by means of technology. Option C is the best answer. D) The only references to, or examples of, the use of images as an instrument for governing relate to surveillance, verification, or documentation of identity: for example, the photographic image is referred to as "an object of scrutiny to assist officials responsible for governing." In terms of the "instrumental approach" the images are part of a "sphere of surveillance and social utility." Conserving or limiting the number of these images would therefore result in less rather than more control by the government. It would not, moreover, further any "coherent ideology."

The passage discussion of local content (paragraph 5) assumes which of the following? A. Students lack the ability to criticize "concrete realizations about observable communities." B. Materials that deal with social sciences should not be a part of a rooted education. C. There is an "artificial boundary between the roles of student and citizen." D. Professors have traveled to distant communities more often than have the students they teach.

Solution: The correct answer is C. A) The author does not assume this; rather, she suggests that current curricular models have implicitly assumed this. B) The author does not rule out social science itself; she objects to the way it has been presented to students, as something to be "taken on faith." C) The answer is C. In paragraph 5, the author advocates a locally-based curriculum that is meant to allow students to do more than study "abstract theories [and] . . . airy generalizations. The author writes that this "rooted education . . . will tend to erase the artificial boundary between the roles of student and citizen." Because the author seeks to erase this boundary, the author must believe that such a boundary has been erected and enforced to begin with. This is a Comprehension question, because it asks you to understand a passage claim and how it functions as part of the author's argument. D) The passage does say that "the majority of U.S. college students attend institutions in their home states" (paragraph 2), but this does not mean that they may not have traveled extensively. The passage does not support this as an assumption of the author's.

The passage argument implies that socially responsible drama: A. does not generate negative attitudes and feelings. B. uses people's feelings to stimulate their thought. C. evokes people's culture-specific emotions. D. purges people of unhealthy emotions.

The solution is B. A) The author does not make this claim; in fact, the author acknowledges that art may "excite the passions," but does not see this as contrary to social responsibility in art. B) The author writes that "the reactive component of emotion may serve the reflective component by directing attention to information of importance to us" (paragraph 5). C) The author recognizes that "emotions are [not] impervious to culture" (paragraph 6), but the cultural specificity of emotions is not relevant to the author's discussion of social responsibility in art. D) There is nothing in the passage that supports this option.

Which of the following underlying assumptions about professors is implied by the passage argument? A. Most of them are generalists, not specialists. B. Most of them lack a preference for particular geographic areas. C. Most of them care more about ideas than they do about actual places. D. Most of them in the U.S. attended college or graduate school outside their home states.

Solution: The correct answer is C. A) The author talks about the abstract and ideal in contrast to the specific and particular, but this is not the same as identifying the professors as generalists rather than specialists on the basis of the overall scope and content of their professional endeavors. B) The author claims just the opposite: ". . . although they [most professors] may have geographical preferences" (paragraph1). C) This is a Humanities passage, which belongs to the content category "Philosophy." This is a Comprehension question, because it asks you to recognize a fundamental assumption of the passage argument. The author says that rootless professors (deemed to be most professors) "are ignorant of the values of connectedness to place" (paragraph 2), that they should include local content rather than "abstract theories about distant peoples" in their courses (paragraph 4), and that "what is needed is . . . educators willing to take root and cultivate a sense of place" (paragraph 6). All of these suggest that professors care more about ideas than they do about actual places. C is correct. D) The author says that "the majority of U.S. college students attend institutions in their home states" (paragraph 2) and that professors often obtain their jobs (not their educations) outside their home states.

The passage offers no information about mambos' and houngans': A. genders. B. roles. C. costumes. D. beliefs.

Solution: The correct answer is C. A) The distinction between mambos and houngans is a gendered distinction, so information about their genders is available. B) The mambos and houngans are described as priests and priestesses, respectively, so information about their roles is available. C) This is a Reasoning Within the Text question, as it asks you to evaluate the kind of information that is available (either implicitly or explicitly) in the passage. C is the correct answer, because passage information is available about the other options, but there is no information about or even reference to the specific costumes worn by the mambos and houngans. There is a reference to "costume changes" at the end of paragraph 3, but there is no way to know who is wearing these costumes, and there is certainly no information about any costumes specifically worn by mambos or houngans. D) The mambos and houngans are described in the context of the passage depiction of a "polytheistic belief system" centered on loas (paragraph 2), so information about their beliefs is available.

Which one of the following groups would constitute part of a fictional audience, as the term is used by the passage author? A. People gathered in a concert hall to listen to a violinist B. Passersby who listen to someone on a park bench who is strumming a banjo C. People who belong to the target group of 18- to 49-year-old viewers of a sitcom D. People who vote by telephone for a contestant in a television talent show

Solution: The correct answer is C. A) The fictional audience is usually "not actually present" (paragraph 2), and the audience described in option A is, indeed, physically present. B) This is a physically present audience, not an imagined one. C) This is a Reasoning Beyond the Text question because it asks you to apply a concept from the passage to a set of novel scenarios not discussed in the passage. The correct answer is C because this option most clearly describes the kind of imagined audience the passage author discusses. The author describes the fictional audience as "some individual or group [Hemingway, like any other writer] conjures up himself like one of the characters in his novels or short stories" (paragraph 2). This is similar to the target audience described in option C. This audience is not physically present and is the imaginative creation of the writers of the television show. D) This is incorrect because although this group is not physically present, they are the actual viewing audience; there is no imaginary or implied reader (or viewer) present in this case. Further, this audience is actively engaged in constructing their own role as audience, rather than simply being bound by the rules designed by the author.

Why does the author quote the letter which states "'Today my chambers painted a fine blue'" (paragraph 5)? A. To return to a point made earlier in the passage about blue paint B. To show that interior decoration has been as important to occupants as exterior decoration C. To illustrate the value of consulting documentary evidence as part of architectural paint research D. To suggest the danger of using one type of evidence without corroboration by another type of evidence

Solution: The correct answer is C. A) The letter is unrelated to the earlier point about the blue pigment developed in the seventeenth century. B) This is not the point of the letter. C) This is a Reasoning Within the Text question, as it asks you to determine the function in the passage of a particular piece of evidence. The answer is C, because the author quotes the letter as part of the argument that good research is holistic, drawing on all available sources. The author uses the example of the letter as an example of a situation in which a document may be more useful that "sophisticated pigment analysis" (when one wishes to date "a significant paint layer"). D) The author does not suggest that it is "dangerous" not to corroborate finds with another type of evidence, just that doing so leads to more accurate and efficient work.

Based on the passage, which of the following best describes how research objectives changed beginning in the 1920s and 1930s? A. A shift toward providing an empirical basis for economic policies B. A shift toward documenting trends in industrial production C. A shift toward examining consumer behavior D. A shift toward defining an appropriate standard of living

Solution: The correct answer is C. A) The passage describes a shift in "empirical research objectives," but both the research on low-income families and that on consumer behavior more broadly was empirical. B) The passage says instead that the new trend in empirical research (a focus on consumer behavior) in part stemmed from a concern that economists were overly focused on production (paragraph 3), so this is the opposite of what the passage says. C) This is a Social Sciences passage that falls under the category "Economics." This is a Foundations of Comprehension question, because it asks you to identify a trend that the author describes in the passage. C is the answer because the passage says explicitly that before the 1920s and 1930s, expenditure surveys generally focused on the "standards of living" of people with lower incomes (or "social groups near the bottom of an industrial economy") (paragraph 2). Beginning in the 1920s, the author explains in paragraph 3, empirical data on expenditures began to focus on consumer behavior rather than primarily on low-income families. D) This was an emphasis of the empirical research on expenditures that preceded the change described in the passage.

Which of the following criticisms of a proposed space shuttle flight is most like that of the "major scientific dissenters" (paragraph 6), as the author presents their views? A. The composition of the shuttle crew violates equal rights legislation. B. The shuttle flight is motivated mostly by political objectives. C. The shuttle assembly has serious design flaws. D. The shuttle flight costs too many taxpayer dollars.

Solution: The correct answer is C. A) The passage description of the "major scientific dissenters" does not suggest that they have any concern about violations of legislation. B) The passage description of the "major scientific dissenters" does not imply that they claim the guidelines are motivated by political objectives. C) This Reasoning Beyond the Text question asks you to find similarities between passage information and a scenario that does not appear in the passage. According to the passage, the "major scientific dissenters from government dietary policy are not especially concerned with governmental paternalism, though that is a legitimate issue. They dissent because they find the government's evidence inadequate and its recommendations potentially harmful" (paragraph 6). A concern about "serious design flaws" is, similarly, a concern about potential harm. D) The passage does not represent the "major scientific dissenters" as particularly concerned about expense.

Which of the following statements best explains Ewen's use of the phrase "obliteration of the factory," as it is used in the passage? A. Industrial jobs were declining in number. B. Industrial jobs produced consumer goods. C. Industrial jobs were hidden from public view. D. Industrial jobs were being replaced by new models of production.

Solution: The correct answer is C. A) The passage does not suggest this. B) This is not what the phrase "obliteration of the factory" implies. C) This is another Comprehension question, as it asks you to understand what a particular phrase means in the context of the passage. The answer is C, because the author explains that Ewen talks about the "obliteration of the factory" to describe the division of the economic world into "an unpleasant sphere where goods were produced and a gratifying sphere where they were consumed" (paragraph 2). This suggests that the "obliteration" is meant to make the production side of the consumer economy invisible, so that it the gratification of consumption seems to come without the messy realities of industrial labor. The author also explains that theatrical performance is unusual because the labor side of the equation cannot actually be hidden, as it is "embodied by the performer." As the author opposes theatrical labor to other kinds of labor, it is clear that he suggests that industrial jobs can be hidden. D) The passage does not suggest this.

The author's apparent attitude toward Dali's strange behavior in public is that it was: A. the result of his lifelong struggles against madness. B. the sort of action in which most Surrealists indulged. C. necessary for artistic and publicity reasons. D. needed to divert attention from his unskilled paintings.

Solution: The correct answer is C. A) The passage refers to Dali's "feigned madness," and also quotes him directly disavowing madness. The author suggests that Dali used the appearance of madness to the benefit of his art, not that he was truly mentally ill. B) The passage provides evidence against this claim, when the author describes the Surrealists who, after first welcoming Dali, eventually sought to disassociate themselves from him. If "most Surrealists indulged" in this kind of behavior, then it follows that they would not have been so bothered by it. C) This Reasoning Within the Text question asks you to assess the author's attitude toward Dali's bizarre public behavior, based on evidence in the passage. The passage provides evidence that the author does see Dali's behavior as "necessary for artistic and publicity reasons," particularly in paragraphs 2 and 3, as the author discusses the ways in which Dali's performances served "useful" purposes. D) The author refers to Dali as an "admirable creator" and as a very significant contributor to modern art; the author in no way implies that Dali was "unskilled."

The passage suggests that the outcome of the Revolutionary War could have been altered if: A. Britain's navy had been allowed to fight in force. B. Britain had committed itself to a war of longer duration. C. Britain had developed a comprehensive strategic plan. D. King George III had been in charge of British plans.

Solution: The correct answer is C. A) The passage suggests that the problems of complacency, insufficient guiding strategy, and internecine politics were of greater import than the amount of naval power brought to bear. B) The length of the war that the British were willing to fight is not presented in the passage as a matter of particular strategic importance. C) This is a Comprehension question. The author refers repeatedly to the lack of a coherent strategy on the part of the British (in paragraphs 2, 3, and 4, as well as in paragraph 6, when she writes that the "British assumed that no clear plan was needed"). The implication is that a "clear plan"—a guiding strategy—was clearly necessary. D) To the contrary, the passage suggests that King George had a limited and ineffective view of the situation—he knew only that he wanted "to conquer—but not how."

What conclusion about the nature of audience support is justified by the results of Experiment 2 alone? A. Supportive friends can disrupt a performance if they would benefit from its success. B. Supportive strangers can enhance a performance if they would not benefit from its success. C. Strangers can disrupt a performance if they would benefit from its success. D. Strangers can disrupt a performance if its success would benefit the performer.

Solution: The correct answer is C. A) The question asks what can be concluded from Experiment 2 alone, and only strangers were included in Experiment 2. B) The only result reported involved the negative impact of an involved audience, not a potential positive impact of strangers who do not stand to benefit from the performer's success. C) This is a Reasoning Beyond the Text question because it asks you to apply passage information to scenarios not presented in the text in order to judge which scenario would be most consistent with passage arguments. The answer to this question is C because all of the observers in Experiment 2 were strangers, and participants in the "supportive-audience" condition were told that they and the observer would receive a cash prize if the participant performed well. The results of Experiment 2 confirmed that a "skilled performance would suffer with a supportive audience." You can therefore conclude that strangers can have a negative effect on a performance if they are in a position to benefit from it. D) In both conditions, the participant was told that he or she would benefit from success; it was only the potential to benefit of the observer that was varied.

Suppose that Experiment 1 is repeated with the addition of a "hostile-audience" condition and that this condition produces data equivalent to those of the "supportive-audience" condition. Which of the following hypotheses would best accommodate this outcome? A. A hostile audience does not affect performance. B. A supportive audience impairs performance. C. An involved audience impairs performance. D. A nonhostile audience enhances performance.

Solution: The correct answer is C. A) The question specifies that the effect of a hostile audience was equivalent to that of the supportive audience, which had a negative effect on performance. B) This does not account for the additional information about the hostile audience provided in the question. C) This is a Reasoning Beyond the Text question because it presents a hypothetical scenario not described in the passage and asks you to incorporate passage information to interpret that scenario. The answer to this question is C because Experiment 1 showed that "a supportive audience" (friends of the participant, in the case of Experiment 1) "has a detrimental effect" on performance, when compared with the effect of strangers on performance. If the scenario described in the question was also true, and a hostile audience produces an effect similar to the supportive one (thus, a negative effect), then it is correct to conclude that an "involved" audience—whether involved positively or negatively with the participant—hinders performance. D) This is not supported by the discussion of Experiment 1, which concerns only the negative effect of a supportive audience, not any audience that improves performance. So if you consider the strangers "nonhostile," there is no support that they improve performance. If you consider the friends "nonhostile," the evidence shows that they impair performance.

Suppose an artist created an exhibition consisting entirely of tastefully arranged collages of mug shots and fingerprints. This exemplifies the passage assertion that: A. pictures of exploding bombs can be used to advertise a safe. B. capitalist societies use photographs to help officials enforce laws. C. any photograph, regardless of intention, may be viewed as art. D. capitalist societies must provide abundant entertainment.

Solution: The correct answer is C. A) The scenario in the question does not exemplify this example of a use of photographs. B) This option does not account for the fact that the photographs described in the question are being displayed as art. C) This is a Reasoning Beyond the Text question, because it presents a scenario not discussed in the passage and asks you how it would relate to passage arguments. The answer is C because the scenario in the question talks about items that are intended to identify people as part of the criminal justice system, and these photographs are now being used as part of an art exhibit. Thus, this exemplifies the passage claim that "any photograph, regardless of intention, may be viewed as art." D) This example does not specifically relate to the ways in which the passage discusses capitalist societies.

The number of monks who resided on Skellig Michael was probably due to: A. insufficient food and water in the monastery. B. extreme difficulty in reaching the monastery. C. limited available living space on the site. D. buildings that were precariously situated.

Solution: The correct answer is C. A) There is nothing in the passage that indicates this. B) Although the monastery is difficult to access, this would not necessarily limit the number of people who were able to reach it. C) This is a Foundations of Comprehension question because it tests your understanding of basic elements of the passage. C is the answer because the passage makes clear that the monastery had little available living space. After describing the elevated "monastic enclosure," the author writes: "Most of the rock is too steep for human habitation. At no time could the little cluster of cells have sheltered more than a dozen monks" (paragraph 3). D) It is specifically the climb to the Needle's Eye that is described as precarious. The buildings are described as built on a narrow shelf of sandstone, but it is not the way the buildings are situated that itself limits the number of inhabitants; it is the available amount of living space.

The story of Stevenson's Scottish nanny (paragraph 2) is probably included in order to: A. demonstrate that those who preserve the trivial also value the serious. B. expose the nanny as a charlatan. C. establish the pervasiveness of the desire to collect an author's mementos. D. demonstrate the popularity of Stevenson's books.

Solution: The correct answer is C. A) This anecdote does not testify to the "seriousness" of those who value the trivial; as is clear in item -02, the author sees that contention as a rationalization on the part of the collectors. B) While the anecdote does humorously expose the nanny as a fraud, this story is not included in order to demonstrate something about the nanny; rather, it is intended to demonstrate something about the souvenir hunters. C) This is a Reasoning Within the Text question because it asks you to establish the likely purpose or function of an anecdote that the author includes in the passage. The answer is C because the story of Stevenson's nanny, who purported to have a "single" lock of his hair, but actually sold countless locks of dubious origin, shows how common is the desire to own something that apparently belonged to an author. The author writes archly that "by the time" the nanny was finished with her sales, "there was enough Stevenson hair around to open a wig shop." D) The story really does not say anything about Stevenson's books, which is part of the reason that the author dislikes souvenir hunting.

Based on the passage, the attitude of the United States military as reflected in Field Manual 3-10 was that the military: A. disapproved of the development and/or use of biological weapons by any nation. B. approved of the development of biological weapons, but did not see the use of biological weapons as logical. C. saw the development and use of biological weapons as logical. D. thought biological warfare was ineffective.

Solution: The correct answer is C. A) This clearly contradicts the passage discussion of the point of view of U.S. Army, as reflected in the field manual, which the passage describes as "propaganda" that suggested "that every country which wanted to keep up with the Joneses must have its own biological agents and bomblets too." B) The description suggests that the field manual took a data-driven approach to the topic and represented the use of the weapons as logical. C) This Reasoning Within the Text question asks you to draw conclusions from the way that a piece of evidence is presented in the text. The passage discussion of the field manual suggests that it contains a dispassionate, graphic assessment of biological weapons capacities: "A series of graphs is presented that tell how many biological-agent bomblets an aircraft should drop to cover a given area under given conditions, daytime or nighttime, for various types of terrain and various types of human target." The passage also indicates that the manual treats biological weapons as a military advantage: "It said that the United States was equipped and prepared for biological warfare, that this was the way a modern army should be trained." From the description of the manual, you can infer the position of the U.S. military, which authored the manual. Thus, C is the best option. D) The passage description of the field manual suggests, to the contrary, that the manual represents biological weapons as a military advantage to the nation that possesses them.

In the passage, the author explains that his focus is different from that of most historians because he: A. concentrates on the early twentieth-century theater. B. accepts that actors are "weavers of dreams." C. deals with the economic realities of the theater. D. emphasizes the unequal relationship between businesspeople and their employees.

Solution: The correct answer is C. A) This is not what the author suggests makes his focus different from that of other authors. B) The author suggests that other historians also accept the "weavers of dreams" metaphor; he puts his focus on a different part of the metaphor than do other authors. C) This is a Comprehension question, because it tests your understanding of a central passage theme. The answer is C because the author writes in the first paragraph that "Most often, historians have allowed the dreams that actors weave to take precedence over the process of weaving." He then goes on to distinguish his own perspective from that of "most" historians, writing, "However, actors' dealings with their employers, like those of many other laborers, were fraught with tension." The rest of the paragraph talks about the economic circumstances confronted by actors and puts that in the context of the economic conditions in the culture industry more broadly. The author's concern with the "economic realities" of actors' lives is part of what makes his work different from that of other historians, according to the passage. D) The author implies that most scholars recognize that these unequal relationships exist in business; what distinguishes the author is that he applies these economic insights to the business of acting.

The author would most likely favor a college course that: A. emphasized hands-on experience in a laboratory over textbook learning. B. crossed disciplines and combined creative arts with science. C. applied classroom concepts to a community garden project. D. required a semester of studying and living in another country.

Solution: The correct answer is C. A) This option does not include the local element that the author advocates; the laboratory could be anywhere. B) The author does not argue for this kind of an interdisciplinary approach. C) This is a Reasoning Beyond the Text question, because it asks you to apply passage arguments to unfamiliar scenarios and determine what the passage author would most likely say about them. The answer is C, because the passage is an argument for grounding university education in a more "particular," place-based ethos. The author contends that professors "should include local content in their courses." Option C emphasizes participation in a local setting, exemplifies the environmental awareness the author advocates in the last paragraph, and fulfills the author's prescription of "transforming the world immediately outside the classroom into a laboratory" (paragraph 4). D) The author advocates an immersion in a local environment, rather than the more "cosmopolitan" ethos that she believes characterizes academia in general.

Given the claims made in the passage, the expressions "She's brimming with vim and vigor," "She's overflowing with vitality," "He's devoid of energy," and "I don't have any energy left at the end of the day" would suggest that: A. some people have more energy than other people. B. most people wish that they had more energy. C. many people think of vitality as a substance. D. some people think that vitality affects our ability to argue.

Solution: The correct answer is C. A) This would be a literal rather than a metaphorical reading of these expressions. B) These expressions do not support this reading, as they do not articulate any wish or desire. C) This Reasoning Beyond the Text question asks you to apply ideas from the passage to examples that are not discussed in the passage. All of these expressions describe vitality as if it were a substance (something one can lack or run out of, something that can "overflow" or reach to the brim of a container). As the author claims that we do not simply speak in terms of metaphor, but actually think in metaphorical terms, you can conclude that based on the passage, these expressions suggest that many people conceive of vitality as a substance, as it is figured in the expressions. D) This option brings together two different examples of metaphor—the one about argument in the passage, and the expressions in the question. There is no basis in the passage for combining these two examples in this way.

Why was the identity of the audience unproblematic for the oral storyteller who rode from town to town reciting tales? A. The composition of the audience kept changing. B. The plot of the tale was already familiar to the listeners. C. The members of the audience were literally on the scene. D. The novelty of the performance superseded any need for information.

Solution: The correct answer is C. A) This, if anything, would make the situation of the storyteller more difficult (rather than unproblematic), as he or she might have felt compelled to adapt the story for different listeners. B) This is not supported in the passage. C) This is a Foundations of Comprehension question because it tests your understanding of a basic component of the passage. The answer is C because the author explains that the fictional or imagined audience has been important "from the time when writing broke away from oral performance" (final paragraph). In an oral performance, of course, there were listeners present; the "fictional audience" comes into play when, as the author puts it, the "writer's audience is not actually present during the writing process" (paragraph 2). D) This is not supported by any evidence in the passage.

Suppose that rigorous scientific research demonstrated that physicians who use a clinically-based model provide better care and have better patient outcomes than those who use an evidence-based model. How would this finding affect passage claims? A. It would undermine the views of those clinicians who see clinically-based experience as sacrosanct. B. It would support the suggestion that the approach taken in Norway is very unlikely to be effective. C. It would challenge the views of the economists and health experts, as represented in the passage. D. It would support the idea that music is an apt metaphor for medical practice.

Solution: The correct answer is C. A) To the contrary, this finding would strengthen the views of those who value clinically based medicine. B) The finding is irrelevant to the Norwegian example. C) This is a Reasoning Beyond the Text item, which presents a scenario not discussed in the passage and asks you to consider the impact of that scenario on passage claims. The answer to this question is C because economists and health experts are described in the first paragraph as minimizing the value of clinical judgment (viewing it as "uncontrollable, chaotic, and obeying few rules"), and the findings described in the question challenge this. D) The finding actually weighs in favor of clinically based models over evidence based ones, so it does not particularly support the collaborative, harmonious paradigm that the musical metaphor describes.

Suppose that various expenditure surveys find that when wages increase, people generally save their money rather than spend it on new products. How would this finding affect the arguments of 1920s-era labor activists? A. Their arguments would be supported because savings increase the long-term well-being of the working class. B. Their arguments would not be affected because they do not address the issue of savings. C. Their arguments would be weakened because an increase in wages may not lead to increased consumption. D. Their arguments would be weakened because an increase in wages may not lead to an increase in productivity.

Solution: The correct answer is C. A) While it may be true that savings increase the well-being of the working class, this is not an element of the arguments made by the labor activists in support of higher wages. If people were inclined to save their money when they received higher wages, this would undermine the contentions of the labor activists that higher wages would stimulate economic demand. B) Even if the activists do not address the issue of savings, the question of the effect of higher wages on spending is central to their claims. C) This is a Reasoning Beyond the Text question, because it presents a hypothetical scenario not discussed in the passage and asks you to consider how this scenario would affect passage claims. C is the answer because 1920s-era labor activists justified their arguments for increased wages for workers with the claim that workers needed to be paid enough to ensure that they had adequate "purchasing power" to create demand for the goods that the U.S. economy produced. If, however, people save rather than spend their money when their wages are increased, then this would weaken the arguments made by the labor activists, beause wage increases would not necessarily lead to increases in consumer demand. D) Based on the passage, the labor activists did not link wages and productivity.

It would be consistent with Hauser's views in the passage to regard threats to survival as catalysts for animal responses that are: A. aggressive. B. unique. C. acquired. D. automatic.

Solution: The correct answer is D. A) A is incorrect because Hauser indicates that animal responses to predator threats are for protection versus aggression purposes (paragraph 5). B) B is incorrect because Hauser claims these responses are "commonplace" (paragraphs 5 and 6). C) C is incorrect because in paragraph 7 the author suggests that Hauser sees animal leader responses as more instinctual than acquired. D) This is a Reasoning Within the Text question because it asks you to understand a passage argument (Hauser's distinction between a transfer of information and learning a new skill, or a theory of mind) and consider what would be consistent with this argument. The answer to this question is D because Hauser is cited in paragraphs 5 and 6 as saying that animals often send out alarm calls to warn of predators, but that this is more of an automatic response, as opposed to teaching and learning a new skill.

The author of the passage quotes Yeats's introduction a second time (final paragraph) in order to: A. illustrate Yeats's evocative use of imagery. B. show that they are in agreement on some minor points. C. acknowledge that Yeats's attitude was understandable in the context of the period. D. contradict Yeats's characterization of the relationship between poetry and war.

Solution: The correct answer is D. A) A is incorrect because although Yeats's quotation includes imagery, the author does not present the quotation in the last paragraph in order to highlight that imagery. B) B is incorrect because the author does not indicate that he and Yeats share minor points of agreement about wartime poetry. C) C is incorrect because although the author acknowledges that Yeats's attitude was somewhat understandable (paragraph 2), the author ultimately disagrees with Yeats's perspective. D) This is a Reasoning Within the Text question because it asks you to understand how, or for what purpose, the author cites a particular piece of evidence. The answer to this question is D because the author again quotes Yeats at the end of the passage (after doing so in the first paragraph) in order to challenge Yeats's view of the role of poets in relation to war. Rather than exhorting poets to forget the suffering of war (as Yeats recommends in the first paragraph), the author uses Yeats's words to argue that poets should help readers remember that suffering.

The answer to which of the following questions would be most useful in determining the validity of the Embers' study? A. Who determined the length of the period that would be covered in the study? B. Were there any similarities between child-rearing practices in warlike and non-warlike societies? C. Which Native American tribes were included in the study? D. Did the societies that worried most about food shortages actually attempt to avoid them through war?

Solution: The correct answer is D. A) A is incorrect because answering who determined the time period covered in the study is not clearly relevant to the study's validity. B) B is incorrect because even though the Embers' theory did discuss child-rearing practices, child-rearing was a secondary, rather than primary, concern of the study (paragraph 5). C) C is incorrect because knowing which Native American tribes were included in the study is not clearly relevant to the study's validity. D) This is a Social Sciences passage that falls within the Anthropology content category. This is a Reasoning Within the Text question, because it asks you to evaluate the validity of the Embers' theory presented in paragraph 5. The answer to this question is option D, because, according to the Embers' theory of war, "societies that engage in the most warfare express considerably more fear of food shortages . . ." (paragraph 5).

If Bill Jay's arguments against the presentation of Winogrand's slides and negatives (paragraph 4) were accepted, which of the following would NOT be a logical outcome? A. Less of Winogrand's work would be seen by the general public. B. Critical opinion of Winogrand's abilities would be lacking in some areas. C. Winogrand's color photographs would be forgotten by all but specialist scholars. D. Winogrand's recognition as one of the U.S.'s great photographers would be lessened.

Solution: The correct answer is D. A) A is incorrect because it is probable that less of Winogrand's photos would be exhibited if his color slides were not allowed to be shown publicly. B) B is incorrect because critical opinion of the color slides would most likely be lacking, as only researchers and not public reviewers would be allowed to view them (paragraph 4). C) C is incorrect because the color slides would likely only be viewed by scholars (paragraph 4). D) This is a Reasoning Beyond the Text question because it asks you to understand a claim in the passage, Bill Jay's argument, and apply it to a conclusion not articulated in the passage. The answer to this question is D because the passage implies that Winogrand would still be considered a great photographer even if his color slides were not exhibited (first paragraph).

Which of the following, if assumed to be true, most supports the author's position on Fairtrade? A. It is difficult to monitor producers' compliance with labor standards. B. People will hesitate to pay $12 for a pound of coffee. C. Many non-Fairtrade traders provide a decent wage to their workers. D. Much of the money given to aid agencies does not reach its intended recipients.

Solution: The correct answer is D. A) A is incorrect because it would actually weaken, rather than strengthen, the author's stance on Fairtrade. B) B is incorrect because if consumers are hesitant to purchase expensive products, this would challenge, rather than support, the author's advocacy of Fairtrade products and the passage suggestion that some consumers will choose a more expensive product "that they know has been grown without toxic chemicals . . . by farmers who can now afford to feed and educate their children" (paragraph 5). C) C is incorrect because it does not support (nor does it weaken) the author's position that Fairtrade is valuable. D) This is a Reasoning Beyond the Text question because it asks you to consider the impact on the passage argument of a series of hypothetical scenarios given in the options. The correct answer to this question is D because D helps the author refute the suggestion of economists who would suggest that it is better to give money "to an aid agency that provides food and education to poor children" (paragraph 6) than to purchase expensive Fairtrade products. If, as option D would have it, a significant portion of the money given to aid agencies "does not reach its intended recipients," this only strengthens the author's argument for the advantages of empowering growers through Fairtrade.

Assume that societies with a high degree of conflict between internal factions tend to wage war more frequently than do more cohesive societies. This information most strongly correlates with which of the following factors mentioned in the passage? A. The relationship between natural disasters and warfare B. The consequences of warfare C. The proximate causes of warfare D. The ultimate causes of warfare

Solution: The correct answer is D. A) A is incorrect because natural disasters, although discussed in paragraph 5, are not clearly linked to the concept of conflict between internal factions. B) B is incorrect because this question's assumption is addressing the causes versus the consequences of war. C) C is incorrect because the proximate causes of warfare address factors such as "military preparedness and the goals of its leaders" (paragraph 2, second bullet). D) This is a Reasoning Beyond the Text question because it asks you to consider a hypothetical scenario not directly discussed in the passage and link it to passage claims. The answer is D because in paragraph 2 under the first bullet, it is mentioned that ultimate causes of war can include "intense divisions between groups of related individuals."

The passage suggests that the poetry of Kipling and Hardy was unusual at the time they wrote because it: A. recounted individual acts of heroism. B. contained moving elegies to the dead. C. celebrated the glories of contemporary battles. D. depicted the squalor and futility of war.

Solution: The correct answer is D. A) A is incorrect because paragraph 3 does not indicate that Kipling and Hardy wrote about individual heroism. B) B is incorrect because the passage does not state that Kipling and Hardy wrote elegies to the dead. C) C is incorrect as the passage does not suggest that Kipling and Hardy celebrated contemporary battles. To the contrary, the author writes that they worked to "get away from . . . commentaries on the glories of the nation's victories." D) This is a Foundations of Comprehension question because it tests your understanding of passage claims using language that is not in the original passage. The answer to this question is D because the author states in paragraph three that Hardy and Kipling were unusual poets for their time in that they wrote realistically about war: they "made a serious attempt to reproduce the voice of the ordinary soldier and to get away from the bardic commentaries on the glories of the nation's victories."

The discussion of job characteristics (paragraph 5) seems to contradict the earlier claim that: A. large organizations have more differentiation than do smaller organizations. B. large organizations have more difficulty filling jobs than do smaller organizations. C. workers in large organizations have more autonomy than those in smaller organizations. D. workers in large organizations have less autonomy than those in smaller organizations.

Solution: The correct answer is D. A) A is incorrect because the discussion of job characteristics in paragraph 5 is not in conflict with the concept of differentiation discussed in paragraph 4. B) B is incorrect because the discussion of job characteristics in paragraph 5 (including the higher demand for highly skilled workers in large organizations) is not inconsistent with the claims in paragraph 3 about large organizations having more positions to fill and potentially facing more competition for workers. C) C is incorrect because the earlier claim (in the first paragraph) says that workers in large organizations have less (not more) autonomy than those in smaller organizations. D) This is a Reasoning Within the Text question because it requires you to understand the logical relationships between passage claims. The answer to this question is D because the authors state in the first paragraph that employees in large organizations have less autonomy than those in smaller organizations; the authors then state in paragraph 5 that highly skilled employees, as well as less monitoring—both characteristic of large organizations, according to the authors—may translate to higher autonomy.

Which of the author's assertions is most susceptible to empirical verification or refutation? A. Six-year-olds feel essentially powerless in the world of human affairs. B. Congressional members seek fame and reputations as crusaders for nonviolence. C. Professors lack wisdom on the issues of television violence. D. Scholars provide the information central to debates about television violence.

Solution: The correct answer is D. A) A is incorrect because the feelings of six-year-old children (like the feelings of any human beings) are difficult to verify or refute empirically. B) B is incorrect because it is difficult to verify or refute people's motivations. Even if someone does gain fame as a crusader for nonviolence, it is difficult to prove that the fame was the person's motivation in doing so. C) C is incorrect because the presence of wisdom is particularly difficult to quantify, prove, or refute. D) This is a Social Sciences passage that falls within the "Sociology" content category. It is a Reasoning Within the Text question because it asks you to consider the nature of a claim and whether it is empirically verifiable. The answer to this question is D because scholars' studies on television violence can be examined and quantified and the presence of scholars' work in debates can also be verified.

If it were established with certainty that Winogrand did, as the author suggests, stop shooting in color because of the "difficulty and expense of making color prints and their instability" (paragraph 2), this information would best support which of the following arguments? A. Winogrand would have liked to have his color slides printed once the technology made this feasible. B. Winogrand felt that working in color was stylistically inferior to black and white. C. The color slides should be viewed as finished products and not printed. D. Winogrand would have returned to photographing in color once the technology improved.

Solution: The correct answer is D. A) A is incorrect because the passage does not suggest that Winogrand would have liked his color slides printed, even if the color technology had been supported. B) B is incorrect because the passage does not indicate that Winogrand believed that color photography was stylistically inferior, and if he stopped photographing in color because of the expense and difficulty of doing so, this hardly supports the idea that he was dismissive of color photography. C) C is incorrect because the scenario in the question offers no support for this claim. D) This is a Reasoning Beyond the Text question because it asks you to take a passage argument and consider its implications for a series of hypothetical claims not discussed in the passage. The answer is D because if it was true that the reason Winogrand stopped photographing in color was because of the difficulty and expense of doing so, then it is likely that with improved technology (less difficulty and likely less expense), Winogrand would have been likely to return to color photography.

The author of the passage suggests that if a contemporary literary critic were to agree with Yeats about war poetry, that critic's view would probably be: A. welcomed by most readers of poetry as a voice of reason. B. influential in directing attention to the poetry of later wars. C. taken into account in the compilation of poems for the next edition of the Oxford anthology. D. dismissed by most devotees of contemporary poetry.

Solution: The correct answer is D. A) A is incorrect because the passage indicates that current readers most likely agree with the author that Yeats was not being reasonable in his opinions about war poetry (paragraph 5). B) B is incorrect because Yeats's opinion of wartime poetry is no longer considered influential (paragraph 5). C) C is incorrect because Yeats's opinion of wartime poetry (or the poetry of the Great War, specifically) would most likely not be included in the next edition of the Oxford anthology, as his perspective is no longer accepted (paragraph 5). D) This is a Reasoning Beyond the Text question because it asks you to apply a passage argument to a scenario that is not discussed in the passage. The answer to this question is D because in the final paragraph the author states that "we can now see this [Yeats's opinion about the poetry of war] for the pernicious nonsense that it is. . . ." So any contemporary critic who embraced this view would likely be considered far outside the mainstream of critical thought.

According to the passage, corner columns with the same diameter as the columns they flank most likelyappear thinner because these corner columns: A. seem to give slightly under the extra weight of the lintel. B. are spaced closer to the columns they flank than those columns are to each other. C. are made of lighter material than the columns they flank. D. are viewed against the sky rather than against the cella.

Solution: The correct answer is D. A) A is incorrect because the point about the columns seeming to give slightly under the weight of the lintel concerns the curving outward and then tapering inward of the columns and is not the reason that the corner columns appear thinner than those columns they flank. B) B is incorrect because the spacing of the columns is not the reason that the corner columns appear thinner than those they flank, but instead, the spacing is part of the architect's remedy for the apparent discrepancy in the thickness of the columns. C) C is incorrect because it is not supported; all of the columns are made of the same material—some merely appear lighter or darker due to the background against which they are viewed. D) This is a Foundations of Comprehension question because it tests your understanding of passage content. The answer is D because the author explicitly states that "the corner columns seem dark when viewed against the sky, while those between seem light when viewed against the dark cella. If all [columns] were equal in diameter, those on the ends would appear thinner" (paragraph 4). The passage explains, then, that the corner columns appear thinner because of the lighter background of the sky which gives the corner columns a more distinct and thinner appearance.

The point of some scholars regarding Madison (paragraph 3) is vulnerable to which of the following criticisms? A. They likely reverse cause and effect. B. They fail to appreciate an argument by analogy. C. They invoke an inapplicable principle. D. They unjustifiably assume a mutual exclusivity.

Solution: The correct answer is D. A) A is incorrect because the point that "some scholars" make does not reverse cause and effect. B) B is incorrect because the scholars discussed in paragraph 3 (who argue that Madison took his model of government from a study of governments in Europe, ancient Rome and Greece) do not make an argument by analogy. C) C is incorrect because the scholars discussed in paragraph 3 do not invoke an unjustified principle. D) This is a Social Science passage that falls under the content category "History." This is a Reasoning Within the Text question because it asks you to evaluate the soundness of the author's reasoning and to consider any logical flaws in the author's argument. The answer to this question is D because "Some scholars point out that Madison, in The Federalist Papers, surveyed models of government from Europe, as well as from ancient Rome and Greece" (paragraph 3) and, therefore, doubt the influence thesis. They have, thus, assumed that Madison couldn't be influenced by both the Iroquois model and those other models, which is an unjustified assumption.

If it were confirmed that Greek architects did not deliberately strive for a strict mathematical balance between horizontal and vertical elements in their designs, what effect would this fact have on assertions made in the passage? A. It would challenge the assertion that the Parthenon's design is psychologically correct. B. It would challenge the assertion that the Parthenon was skillfully designed. C. It would support the assertion that the Parthenon's curves are aesthetically pleasing. D. It would support the assertion that the Parthenon was constructed on a subjective basis.

Solution: The correct answer is D. A) A is incorrect because the scenario in the question would potentially support, rather than challenge, this claim. B) B is incorrect because the point that the architects did not intentionally seek strict mathematical balance does not necessarily challenge the point that the Parthenon was skillfully designed. Rather, the passage offers extensive evidence that architects used skillful methods to compensate for various subjective optical illusions—skillful methods that may require a jettisoning of strict mathematical balance. C) C is incorrect because the architects' view of strict mathematical balance is irrelevant to the claim about the aesthetic value of the building's curves. D) This is a Reasoning Beyond the Text question because it presents a hypothetical scenario not discussed in the passage and asks you to consider its effect on passage claims. The answer to this question is D because the passage author writes in the final paragraph that the architects of the Parthenon sought to create the "appearance of a balance between vertical and horizontal elements," but that the building is in fact "psychologically rather than mathematically correct . . . made to compensate for the optical aberrations of the human eye. The construction is thus carried out on a subjective [rather than objective] basis." If, as the question specifies, architects did not seek a "strict mathematical balance between vertical and horizontal elements," this would clearly support the assertion in D, that they sought subjective effects, rather than conforming to an objective, mathematical formula.

The passage suggests that Meselson's opposition to development of biological weapons was based, in part, on the claim that: A. biological weapons do not work. B. biological weapons research is very expensive. C. U.S. laboratories should be converted to medical research laboratories. D. biological weapons in the hands of small and poor countries constitute a particular danger.

Solution: The correct answer is D. A) Meselson believes that these weapons are "unreliable" and "inappropriate" (paragraph 5), not that they do not work. B) The passage does not discuss the costs of biological weapons research. C) This option presents one of the results of Meselson's work against biological weapons, but it is not one of the claims on which his opposition to the weapons actually rests. D) This Comprehension question tests your understanding of passage information. The author introduces Meselson's opposition to biological warfare, then writes, "his arguments rested on three main points. First, biological weapons are uniquely dangerous in providing opportunities for a small and poor country, or even for a group of terrorists, to do grave and widespread damage." Thus, the passage presents this particular risk of biological weapons as one of the assumptions behind Meselson's opposition to the weapons.

Which of the following findings would most weaken the claim that because of greater complexity and differentiation, large organizations offer more opportunities for promotion and less autonomy than small organizations (paragraph 4)? A. Job openings in large organizations are most often filled by people already working within the organization. B. The larger an organization, the more difficult coordination among employees becomes. C. Employee autonomy at small organizations is constrained by informal monitoring. D. Most differentiation occurs horizontally, creating jobs at the same skill and status level.

Solution: The correct answer is D. A) A is incorrect because this finding supports the passage idea of opportunities for promotion from within large organizations. B) B is incorrect because this finding supports the passage idea of less autonomy within large organizations. C) C is incorrect because the passage claims that small versus large organizations have more formal monitoring, which may constrain autonomy (paragraph 5). D) This is a Reasoning Beyond the Text question because it asks you to consider the effects on a passage claim (the statement from paragraph 4) of a series of hypothetical findings. The answer to this question is D because the passage claim that "greater differentiation . . . is associated with more opportunities for career advancement" (paragraph 4) would be challenged if differentiation indeed created more jobs, but they were jobs at the same level, which don't offer opportunity for promotion.

Based on the description of performance in the second paragraph of the passage, which of the following occupations is most analogous to professional acting? A. Weaver B. Doctor C. Sculptor D. Massage therapist

Solution: The correct answer is D. A) A weaver produces a weaving, which the consumer can view or purchase without being aware of the nature of the labor that produced it. B) A doctor does not generally embody his or her labor. C) Like a weaver, a sculptor produces an object for consumption, which can be viewed (or "consumed," in the terms of the passage) independently of the sculptor. Unlike an actor or a massage therapist, the sculptor does not embody his or her commodity. D) This is a Reasoning Beyond the Text question, because it asks you to apply an idea from the passage to a novel scenario not discussed in the passage. The answer is D, because the passage describes performance as "unusual in that it is a labor process both exhibited before and consumed by an audience. The . . . actor both produces the commodity and embodies it." This is most analogous to a massage therapist, who physically and visibly produces the commodity (the massage) for the consumer.

Given the information presented in the third paragraph, which of the following statements could most reasonably be inferred? A. Only obvious cadential trills were left unwritten. B. Cadential trills were often written in unexpected places. C. Written cadential trills were often omitted by singers. D. Unwritten cadential trills were often expected to be sung.

Solution: The correct answer is D. A) Although the author refers to "obvious and expected unwritten cadential trills," this does not mean that all unwritten trills were necessarily obvious. B) Although some unwritten cadential trills are "expected," according to the author, this hardly implies that written cadential trills were written in unexpected places. C) This is not implied in the passage. D) This Reasoning Within the Text question asks you to make logical inferences based on the information presented in the passage. The author critiques instances in Callas's performances when "obvious and expected unwritten cadential trills are omitted." This allows you to infer that although these trills were not written in a score, the standard expectation is that they should still be included in a performance.

Edward Ahrens's criticism, as it is presented in the passage, essentially points to: I. complexity. II. flexibility. III. diversity. A. I only B. I and II only C. II and III only D. I and III only

Solution: The correct answer is D. A) In saying that the guidelines are "simplistic" (paragraph 4), Ahrens is pointing to a lack of complexity, so I is correct, but so is III. B) There is nothing in Ahrens's comments that suggests flexibility (its problematic presence or absence), so II is incorrect. C) There is nothing in Ahrens's comments that suggests flexibility (its problematic presence or absence), so II is incorrect. D) This Comprehension question asks you to describe the essence of a point made in the passage, using language that is not used in the passage itself. In saying that the guidelines are "simplistic" (paragraph 4), Ahrens points to a lack of complexity in the guidelines, so I is correct. In saying that the guidelines "treated the population as 'a homogenous group . . . while ignoring the wide variation' in individual diet and blood chemistry" (paragraph 4), Ahrens points to a diversity in the population that the guidelines do not account for, so III is also correct.

To fulfill the author's view of an exemplary professor, which of the following subject areas would be most useful for economics professors to know outside of their discipline? A. Mathematics B. Sociology C. Political science D. Ecology

Solution: The correct answer is D. A) Mathematics is largely part of the world of "ideals and scholarship" (paragraph 6). It deals with theorems and abstractions rather than descriptive particulars of place. B) Sociology often focuses on abstract theories (criticized by the author in paragraph 5) and is likely to be considered, by the author, as a repository of "social-science hearsay" (paragraph 5). C) Political science, like mathematics, is largely part of the world of "ideals and scholarship" (paragraph 6). It generally deals with theory and abstractions rather than with descriptive particulars of place. D) This is a Reasoning Beyond the Text question, because it asks you to apply the passage author's argument to a situation outside of the text. D is the answer, because Ecology is less abstract than mathematics and the other academic disciplines. As the study of environmental factors, ecology deals with at least some of those concrete "particulars of place" that the passage author believes most professors are sorely out of touch with. A focus on environmental relationships would help redeem the professoriate from what the author argues is a "rootless" existence limited to "ideals and scholarship."

Based on the passage, the author most likely believes that community health centers and the National Health Service Corps may be: A. preferable to the health options created through the free market. B. underutilized in many rural areas. C. controversial, complex, and expensive. D. providing services that recipients are not necessarily entitled to.

The solution is C. A) In paragraph 2, the author demonstrates admiration for free-market solutions. B) That "these two federal programs remain the preeminent safety net programs for rural America" (paragraph 4) challenges this option. C) Paragraph 2 says that "federal or state delivery systems...may be controversial, complex, and expensive," and paragraph 4 describes community health centers and the NHSC as federal programs. D) There is no implication in the passage that rural residents are "undeserving" of adequate health care.

The passage would suggest which of the following explanations for musicians who receive no applause for their performance? A. They are not very skilled performers. B. They perform music that has no beauty. C. There is no truth in the music they perform. D. There is no one who understands the music they perform.

Solution: The correct answer is D. A) Perhaps surprisingly, the author does not talk about skill. B) Although the author mentions beauty throughout the passage, the critical factor in the author's argument is whether people can appreciate beauty. C) Although the author mentions truth throughout the passage, the critical factor in the author's argument is whether people can perceive truth: the "significance" of the work of art to the "appreciator rests upon his or her capacity at the moment to receive it" (paragraph 1). D) This is a Humanities passage that falls under the content category "Art." It is a Reasoning Beyond the Text question because it asks you to apply passage information to explain circumstances that are not discussed in the passage. The answer to this question is D because the author writes: "a jest's prosperity lies in the ear of the person who hears it" (paragraph 1). In other words, the value of a joke depends on the way the person who hears it interprets it. Similarly, the author writes, "To him who has never opened his eyes to behold the beauty . . . to such the work of the master is only so much paint and canvas" (paragraph 4). The latter example makes clear that to the uncomprehending audience, the work is without value. This supports the idea that the musicians' work is unappreciated because it is not understood.

Which of the following best exemplifies the author's assertion that, according to the instrumental approach, cameras can be used to present information that facilitates timely, accurate responses? A. Photographs taken to record family history in an album B. Photojournalism that produces pictures that tell an amusing story C. Photographs used in advertising campaigns to discourage teen-age smoking D. Photographs taken by cameras at stop lights to catch traffic offenders

Solution: The correct answer is D. A) Photographs to record family history don't really facilitate "timely, accurate responses;" they preserve the past. B) Although photojournalism may be timely, an amusing story doesn't necessarily facilitate a response. C) This may or may not elicit a response, but there's nothing to suggest timeliness. D) This is a Reasoning Beyond the Text question, because it requires you to apply passage ideas to a novel set of circumstances. The answer is D. The information presented by the traffic camera is a photo of someone in the act of committing an offense, and the response facilitated by such information is a penalty of some sort. Furthermore, this option is close to the claim in the passage that cameras "implement the instrumental view of reality," an instance of which is "video equipment...used to identify criminal offenders" (paragraph 2).

Given Plato's claim that the spoken word is superior to the written word in the pursuit of truth, which of the following activities would he have been most likely to encourage today's truth seekers to adopt? A. Listening to candidates for political office debate each other on the radio B. Participating in a group that discusses Phaedrus and other written works at a local university C. Posing comments on Internet sites and using online chat rooms to discuss the issues of the day D. Getting involved in an ethics discussion group hosted by a public library

Solution: The correct answer is D. A) Plato advocated debate as a forum for challenging errors and producing truth. It is the debate participants who engage in this process, so simply listening to others debate would not be an activity that Plato would advise truth seekers to pursue. B) Plato argued that a written text was essentially fixed, because the reader could not argue with the writer to correct errors and seek truth. An ancient written text like the Phaedrus would certainly not be susceptible to the kind of active discussion that Plato recommended, since its author, Plato himself, is long dead and could not participate in any discussion. C) Again, the comments described here are written comments and any debate that takes place is in the form of writing and can be "misread, misconstrued, or misused" (paragraph 3), unlike the verbal debating that Plato recommends. D) This is a Humanities question that falls under the content category of "Language." This is a Reasoning Beyond the Text question because it asks you to apply passage information to a series of options not discussed in the passage. D is the answer, because Plato argues that "debating sharpened memories, corrected misconceptions, and produced truth" (paragraph 3). This option describes people actively participating in a public debate about ethics and would be the kind of activity that Plato would advise for those seeking truth.

The purpose of Experiment 2 was to: A. be certain that the supportive observer was truly supportive. B. isolate the reason for adopting a cautious performance style. C. test the effect of an incentive to improve one's performance. D. determine the relevance of friendship to the audience effect.

Solution: The correct answer is D. A) The "true" supportiveness of the observer is never at issue in the experiment; rather, the participants believe that the supportiveness of the observers varies, based on whether or not the observers stand to benefit from the participants' performance. B) There is nothing in the description of Experiment 2 to support this. C) In both conditions, the performer had an incentive to perform well; what differed was whether the participant believed that the observer would benefit from a strong performance. D) This is a Foundations of Comprehension item because it tests your ability to understand the connections between the author's points; the author first describes a problem with Experiment 1 and then a design for Experiment 2 that addresses this problem. The answer to this question is D because in Experiment 1, all of the supportive audience members were friends. The author writes, "the reason for this effect [of the supportive audience] was unclear . . . Friends and strangers differ in many ways" (paragraph 5). In Experiment 2, all of the audience members were strangers, and some of them were in the supportive condition. This experimental design makes it possible to test the effects of a supportive audience, independent of the role of friends.

Assume that a new Folsom hunter site has just been discovered in northern Texas. On the basis of the information contained in the passage, this site would most likely contain all of the following EXCEPT: A. clusters of bones and tools. B. human bones. C. remains of hearths. D. tools made of Colorado flint.

Solution: The correct answer is D. A) The passage suggests that animal bones and tools are present at all Folsom hunter sites, so one can infer they would be present at this newly discovered site in Texas. B) Similarly, the passage suggests these bones are common at Folsom hunter sites and would be present at this site in Texas. C) The passage indicates that these remains would be found in Folsom Hunter sites. D) This Reasoning Beyond the Text question asks you to take information contained in the passage and apply it to a new scenario not discussed in the passage. As the passage states that the Folsom Indians did not travel long distances (paragraph 3), and that there was apparently no trading between geographically distant bands of Folsom Indians, you can infer that Folsom Indians from Northern Texas would not have had access to flint from Colorado.

Which one of the following describes a modification in the role of an intended reader that might best serve the purposes of a mass media advertiser? A. The intended reader is addressed in very personal terms, as indicated by specific terms of endearment that are assumed to be pleasing to him or her. B. The intended reader is addressed as though he or she were an "insider" already in possession of enough information to construe an otherwise cryptic message. C. The intended reader is addressed as though he or she were a scholar seeking a detailed, objective account of product features. D. The intended reader is addressed as if he or she were experiencing a problem that purchasing the product would solve.

Solution: The correct answer is D. A) The advertiser is a "mass media" advertiser and must reach a large audience. Specific, tailored terms of endearment would be ineffective and impractical. B) If the message in the advertisement was cryptic and the intended reader could not understand it, he or she would not know that he or she was intended to purchase the product. This would not serve the advertiser's purposes. C) The advertiser presumably hopes to make the intended reader receptive to, and even eager for, the product. A detached, objective presentation might not position the reader in the frame of mind in which the advertiser would like to place him or her. D) This is a Reasoning Beyond the Text item because it presents a scenario that is not discussed in the passage and asks you to use passage information and context to understand and interpret that scenario. The answer is D, because an advertiser is, by definition, interested in selling a product. Thus, if the intended reader is addressed as if he or she is experiencing a problem that the advertiser's product would solve, the reader may see himself or herself in this same way and thus be more amenable to purchasing that product. As the passage explains, the "reader must play along [with the role the author (or advertiser) casts him in] or put down the book" (paragraph 2).

The author suggests that concluding that diet is responsible for heart disease would be: A. mistaking cause for effect. B. mistaking correlation for causation. C. failing to consider a common cause. D. failing to consider additional causes.

Solution: The correct answer is D. A) The author does not suggest that heart disease in any way determines one's diet. B) The author does not establish a correlation between heart disease and any particular diet (which would be necessary in order to mistake correlation for causation). C) The author does not suggest that there is a "common cause" of heart disease that those who focus on diet are overlooking. D) This Reasoning Within the Text question asks you to identify the logical principle underlying the author's argument here. The author writes that "multiple factors were potential contributors [to heart disease], including genetics and personal habits such as smoking" (paragraph 2)—so concluding that diet alone is responsible for heart disease would indeed be failing to consider additional causes.

Which of the following approaches to research does the author most support? A. Experimental B. Theoretical C. Statistical D. Holistic

Solution: The correct answer is D. A) The author does not talk about performing experiments. B) The type of research that the author advocates involves science and collecting documentary evidence; it is not entirely, or primarily, theoretical. C) The author does not advocate a statistical approach. D) This is a Comprehension question, as it asks you to characterize the author's position or viewpoint. The answer is D, because the author argues for the incorporation of multiple sources of evidence in order to plan and execute good architectural paint research; the author points to "the importance of collating and synthesizing all existing evidence" (paragraph 5). This supports the choice of D, "holistic."

According to the passage, if a reader who thinks highly of Virginia Woolf's fiction came upon a biography of Woolf, that reader should: A. use the biography as a way of ascertaining the meaning of Woolf's fiction. B. read the biography as a replacement for reading Woolf's fiction. C. approach the biography as faithfully as people in earlier times revered biographies of the saints. D. hesitate to read the biography, because to do so would be to betray Woolf.

Solution: The correct answer is D. A) The author explicitly challenges the idea that biography offers insight into a writer's work. B) This option is precisely the opposite of what the passage would recommend. C) The author suggests that in contemporary times, literary biography has perhaps come to replace the study of the lives of the saints, but she certainly does not advocate this approach to literary biography. D) This is a Reasoning Beyond the Text question because it asks you to apply passage arguments to a hypothetical scenario that is not discussed in the passage. The answer is D because the passage author argues explicitly that "the only time [a reader is] truly close [to a writer] is when [the reader is] reading words on the page . . . the rest is dilution, marginality, betrayal." In the next sentence, the author makes clear that biography is included among other forms of souvenir collecting. Thus, based on the passage, a reader should not read the biography of Woolf she has found; in the passage author's mind, doing so would constitute a betrayal of Woolf.

Which pair of contradictory statements is implicit in the passage? A. Poetry and visual art are similar; poetry and visual art are different. B. Only artists can truly appreciate art; art appreciation is what makes us human. C. There is but a single standard by which to judge art; there are many ways to appreciate art. D. We can appreciate only that which we recognize; appreciating art is all about being open to the new.

Solution: The correct answer is D. A) The author mentions poetry (paragraph 2) to make a general point about art (that we gravitate toward that in which we recognize ourselves in an elevated form); the author does not make any comparison between poetry and visual art. B) The author does not imply that only artists can truly appreciate art. In fact, much of the passage is devoted to the belief that the "appreciator" (not necessarily an artist at all) can "develop the personal capacity to receive and enjoy" art. C) The author does say, "There is no single invariable standard by which to try a work of art" (paragraph 1), and never contradicts that. D) This is a Reasoning Within the Text question because it asks you to identify logical relationships between passage statements. The answer to this question is D because paragraphs 2-4 establish that familiarity is a prerequisite for appreciating art, but the first paragraph indicates that the appreciator must look for the new ("What has this work to reveal to me of beauty that I have not perceived for myself? What of new significance has this work to express to me?").

Which of the following consequences would have been most likely had the Church ceased to support divine-right monarchy? A. The common people would have fallen away from the Church. B. Freemasonry would have picked up the former political role of the Church. C. Revolution could have been prevented without John's stringent measures. D. John would have included priests in his list of political suspects.

Solution: The correct answer is D. A) The common people "were fondly habituated to their transmitted faith, comforted by its legends, awed by its miracles, thrilled by its ritual" (paragraph 2), and there is no indication that this allegiance is in any way tied specifically to Church support of divine-right monarchy. There is nothing to suggest that a change in the Church's support would weaken its appeal to the common people. B) Freemasons "dreamed of representative government" (paragraph 3), it is unlikely they would have taken over the role of supporting divine-right monarchy had the Church ceased to do so. C) It is unclear, given the information provided in the passage, whether the driving purpose of revolution in Portugal would have been, or would have been only, to do away with divine-right monarchy. The attempt of the Marquis de Pombal was to bring Portugal "abreast, in culture and law," (paragraph 1) with France and Spain. Culture and law at the time seems to have been defined as much as (if not more so) by the Church as it was by the monarchy, including the queen's son. D) This is a Reasoning Beyond the Text question because it asks you to use passage information to assess the potential effects of a scenario not described in the passage. The answer is D; paragraph 5 supports this choice with its mention of John's "fury" against reformers and his instructions to police to "to arrest, or expel, or keep under unremitting surveillance" every dissident writer including nobles.

Recent technological developments like high-resolution satellite imagery and diagnostic positron emission tomography (PET scans) have refined and extended the camera's capacity to provide information. Which passage assertion does this information support most strongly? A. Cameras can illuminate the private, personal lives of individuals. B. Capitalist consumption requires the unlimited production of images. C. Cameras are a means of appropriating reality and making it obsolete. D. Photography can be used to both control and benefit society.

Solution: The correct answer is D. A) The examples in the question provide macro- or micro-level scientific information, which is highly impersonal and quite different from "private life," so this option is incorrect. B) The information provided in the question doesn't support this option because satellite images and PET scans are not really relevant to capitalist consumption. Moreover, they are unlikely to be characterized as "unlimited." C) The information provided in the question does not relate to making reality obsolete or out-of-date. D) This is a Reasoning Beyond the Text question, because it asks you to identify information from outside the passage that works to support passage claims. Satellite images of offenders such as speeding drivers could, for example, be used to both control (by penalizing offenders) and benefit society (by protecting them from offenders); PET scans could be used to benefit society by revealing information about the human anatomy. Option D is the correct answer.

The observation that some teachers use techniques that work best in small classes (paragraph 4) is used to support which of the following assertions from the passage? A. Substantial performance gains from small classes occur in the early elementary grades and do not accumulate beyond first or second grade. B. Educators rarely change their instructional styles to match the size of their classes. C. Small classes free teachers to bestow individual attention and use creative approaches, such as letting students work in small groups. D. When class sizes are reduced, the improved performance of teachers who already use methods well suited to smaller classes pulls up the average achievement level.

Solution: The correct answer is D. A) The fact that early elementary grades are the grades that show evidence of improvement from small class size is a building block in the paragraph's larger argument, and not the point that the assertion quoted in the question is intended to make, so A is incorrect. B) The author does make this claim, but it is not the point supported by the claim that early grade teachers often use techniques that are well suited to small class size. C) This is a claim about the benefits of class size, which is neither supported nor challenged by the point about the teachers who use these techniques. D) This is a Reasoning Within the Text question because it asks you to understand the way that a particular piece of evidence is used in the passage. The assertion cited in the question is that early elementary grade teachers are most likely to use techniques that are most successful in smaller classes. The paragraph in which the assertion appears begins by arguing that teachers who already use techniques "well suited to smaller classes" do especially well when they actually have smaller classes, thus improving the average performance in smaller classes. The rest of the paragraph, including the assertion cited in the question, is meant to explain and support this argument, so the answer is D.

Of the following passage assertions, which one is LEAST supported by evidence or an example in the passage? A. Callas's performances are characterized by their authenticity. B. Callas's editions of some operas scarcely resembled the composers' originals. C. Callas's understanding of performance practice was quite unstudied. D. Callas was the putative champion of dormant traditions in opera.

Solution: The correct answer is D. A) The final paragraph of the passage discusses the nature of Callas's "authenticity" in some detail. B) The passage details the nature of the cuts that made some of Callas's versions move too far from the originals, in the eyes of the author. C) Paragraph 3 provides specific evidence to support this claim. D) This Reasoning Within the Text question asks you to assess or evaluate the evidence provided in the passage to see whether it supports particular claims in the passage. The passage refers to Callas as the "putative champion of dormant traditions," but does not elaborate or provide any examples of what traditions she purportedly championed.

When discussing a magnetic world, the author apparently wants readers to imagine that they are: A. able to control the critical temperature. B. possessors of magnets. C. neutral observers. D. magnets.

Solution: The correct answer is D. A) This may be an attractive choice, because the author asks the reader to imagine the operations of the magnetic world at three different temperatures. However, the author asks the reader to imagine the temperature being raised or lowered; the author does not suggest that the reader is in control of the temperature. B) Again, the author does not suggest that readers have or control magnets. C) The author explicitly describes the reader as living in a magnetic world, alone with one's friends. The reader is not depicted as outside that world. D) This is a Foundations of Comprehension question because it asks you to understand the way the author is using the idea of magnets in a rhetorical sense. The answer to this question is D because when the author describes the operations of the magnetic world, he or she treats its inhabitants as magnets; in paragraph 4, for example, the author writes, "now all the magnets would be flipping back and forth wildly . . . and what one magnet did at any moment would have no correlation whatsoever with what its neighbors were doing. In such a world, none of your friends could be counted on." Humans here are treated as magnets, governed by the laws of the magnetic world.

Consider that some groups are defined as "cults"; members of these extremist sects generally live communally and obey an authoritarian, charismatic leader. Assume that all cults have fewer than 100 members. This information would tend to weaken the passage's assertions about the: A. ability of great orators to sway large groups. B. loss of personality and sense of responsibility. C. difficulties involved in convening large assemblies. D. reasonableness of small groups compared to large groups.

Solution: The correct answer is D. A) The information in the question only applies to the ability of orators to sway smaller groups; it has no impact on claims about the effect of orators on larger groups. B) The reference to "loss of personality and sense of responsibility" refers to the behavior of the individual in a large group, so the information about people's behavior in a small group is irrelevant. C) The information in the question is irrelevant to passage claims about the difficulty involved in convening large assemblies. D) This is a Reasoning Beyond the Text question because it presents information that is beyond the scope of the passage and then asks you to consider how this information would affect passage claims. The answer is D, because the question describes a small group in which people "obey an authoritarian, charismatic leader." In the passage, however, the author claims that a "great multitude" is more likely to succumb to "unreflective enthusiasm . . . than is a small meeting" (paragraph 2). So if the cults that the question describes are small groups of fewer than 100 members, this would weaken passage claims about the more considered behavior of small groups when compared to larger ones.

The passage describes an irony of tourist-oriented voodoo, namely that: A. mambos and houngans, although they are human, exert some influence on the loas. B. loas, although they are supreme beings, deign to inhabit the bodies of humans. C. audiences who make the most effort to find authentic voodoo are least likely to actually find it. D. audiences are kept at a safe distance, which reinforces their lack of belief in the powers of voodoo.

Solution: The correct answer is D. A) The passage actually says that the mambos and houngans guide the aspirants, "as they are possessed by the loas." It does not say that the mambos and houngans necessarily "influence" the divine loas. B) This is not an irony described in the passage, implicitly or explicitly. C) The passage actually challenges the notion that there is one kind of 'authentic; voodoo, arguing instead that "synthetic" versions of voodoo are also authentic, so this is not a claim the passage makes. D) This is a Comprehension question that tests your understanding of passage information and of the presence of irony. The answer is D, because this is the only option that accurately characterizes a passage claim. The tourist-oriented voodoo that the author describes at Mariani does enforce a distance between the audience and the ceremony, which the author suggests makes "inadvertent possession" impossible. Thus, the way that the tourists experience voodoo (designed to meet their expectations) reinforces their own pre-existing expectations about the limited possibilities of voodoo, which is ironic.

Which of the following, if true, would most challenge the passage author's views as presented in the passage? A. Most of the songs featured in the Great American Songbook are about democracy. B. Most of the songs featured in the Great American Songbook were popularized not by the songwriters, but by other singers. C. Most of the songwriters featured in the Great American Songbook had more than one song in the book. D. Most of the songwriters featured in the Great American Songbook were under the age of thirty when they wrote the songs included in the book.

Solution: The correct answer is D. A) The passage describes the songwriters as "democratic populists," but this does not mean that the songs they wrote were about democracy. Rather, their songs "were intended to advance the plots of" the musical productions they wrote for. B) This does not challenge anything that the author says about "Golden Age" (or conceptual) artists. C) This does not challenge anything that the author says about the "Golden Age" (or conceptual) artists. D) This is a Reasoning Beyond the Text question, as it asks you to identify how a hypothetical scenario not discussed in the passage would affect passage arguments. D is the correct answer because the author maintains that "traditional artists' work tends to improve with age, as they gain a greater understanding of both their subject and their art." The Golden Age songwriters in the Great American Songbook fit the author's definition of traditional artists, so if their most successful work (as measured by inclusion in the songbook) was produced before they were thirty, this would challenge the author's claims.

According to the passage, which of the following activities was common to each band of Folsom Indians? A. Cultivating a number of different crops B. Eating a wide variety of wild game C. Interacting with other bands D. Making tools out of nearby rocks

Solution: The correct answer is D. A) The passage does not discuss crop cultivation. B) The passage indicates only that the Indians at the San Luis site attracted "bison and other game animals"; it is not clear that each band of Folsom Indians consumed a wide variety of game. C) The author explicitly contradicts this: "There was apparently little trading and no long-distance movement between the geographically separated bands of Paleoindians" (paragraph 3). D) This question tests comprehension of passage information. The author explains: "An important conclusion from investigations at this site, as well as from comparisons to other Folsom hunter sites, is that each small population of hunters made tools out of material that they gathered themselves" (paragraph 3). The author then describes the kinds of rock used by the hunters at the San Luis site, and indicates that these rocks "were plentiful in the central and northern Rockies." As earlier passage information places the San Luis site in the Rockies, you can infer that these were "nearby rocks" (an implication strengthened by the information that there did not appear to be trading between bands of Paleoindians).

According to the passage, the United States government eventually made the decision to abandon development of biological weapons because: A. they were too expensive to continue to develop, especially during a time of tightened defense budgets. B. other nations would suspend development or use if the U.S. would. C. they would never be used except in retaliation for another country's using them against the U.S. first. D. they could not be shown to serve a logical military purpose.

Solution: The correct answer is D. A) The passage does not discuss the cost of biological weapons as a factor in the decision to abandon them. B) Although the passage describes the U.S. development of and propaganda about these weapons as increasing the risk that other countries would develop and use them, the author does not cite this as the reason for the U.S. decision to abandon biological weapons. C) This is not described as the reason that the U.S. decided to abandon the weapons. D) This Comprehension question tests your understanding of passage content. According to the author, after Meselson's questioning, "the congressmen became convinced that his third point was valid. Even from the narrowest military point of view, our biological weapons policy made no sense." Meselson's "third point," the previous paragraph tells you, is that, "biological weapons are uniquely unreliable and therefore inappropriate to any rational military mission for which the United States might intend to use them." The passage shows this to be the decisive turn in Meselson's convincing the Congressmen, and thus in the decision to abandon the weapons.

Given the information contained in the passage, if a large number of deer bones were discovered at the San Luis Valley site, the most likely explanation for their presence would be that the deer: A. accidentally died at the scene. B. competed with bison for food. C. migrated from another region. D. served as food for the Indians.

Solution: The correct answer is D. A) The passage does not suggest accidental death as an explanation for the presence of bones at a particular site, so this is unlikely. B) The passage does not treat the presence of animal bones at these particular sites as evidence that the animals competed with one another for food; rather, it treats the bones as evidence (in conjunction with the presence of tools) that the animals served as food for the humans who frequented the sites. C) Based on the passage, there is no reason to propose this as an explanation for the hypothetical scenario the question describes. D) This Reasoning Beyond the Text question gives you a hypothetical scenario not contained in the passage and asks you to apply information or reasoning from the passage to the scenario. The passage describes bison bones found at the Folsom sites as evidence that the bison served as food for the Paleoindians. Applying the logic of the passage, then, the presence of deer bones would suggest that the deer served as food.

According to the passage, the British assumed they would be victorious over the Colonists because of: A. the element of surprise. B. a strategic employment of forces. C. the Colonists' unwillingness to fight. D. a vastly superior military force.

Solution: The correct answer is D. A) The passage does not suggest that the British believed that they had—or needed—the advantage of "surprise" over the Americans. B) The passage mentions several times the lack of a clear British strategy, and their belief that they did not need a strategy to win, so this option is incorrect. C) The passage does not suggest that the British believed that the Americans were unwilling to fight. D) This Comprehension question tests understanding of overt passage content. The author writes, "Carelessness followed from the assumption that the superiority of British force was so great that it made taking pains in performance unnecessary" (paragraph 6).

Of the following passage claims, which one is most supported by evidence or explanation in the passage? A. Before World War I, consumption-based calls for higher wages relied heavily on ethical claims. B. The institutionalists charged that the neoclassical economists lacked an empirically grounded theory of consumer demand. C. Before World War I, labor activists were concerned with arguing for an appropriate standard of living. D. The transition in the political economy during the 1920s and 1930s was accompanied by a shift in research objectives for expenditure surveys.

Solution: The correct answer is D. A) The passage offers no discussion of the ethical bases on which activists made their claims; it only describes what the activists called for: an "appropriate 'standard of living' and . . . wages sufficient to reach that level" (paragraph 1). The passage does not explain what ethical imperatives were appealed to in order to justify these policy recommendations. B) This point is asserted at the end of paragraph 3 but not developed or supported. C) This is asserted, but no evidence for the point is offered. (The focus of researchers on the "standard of living" is discussed in detail, but not the actions of labor activists). D) This is a Reasoning Within the Text question because it asks you to assess the nature and extent of the evidence provided for passage claims. The answer is D because the author provides extensive support for this claim. Paragraph 2, in which this claim occurs, details the empirical research objectives for expenditure surveys before the mentioned shift occurred, explaining how research on the "standard of living" for primarily lower-income families were used: "detailed 'standards of living' could then be employed in multiple ways. . . ." Paragraph 3 explains how research objectives changed to focus on an examination of consumer behavior more broadly and an effort to link that behavior to the health of the economy. (Paragraph 4 provides a specific example of the kind of new study economists sought to do and of its objectives.)

Which of the following is an assertion that is supported by strong evidence in the passage? A. Surrealists were all as eclectic and scandalous as Dali. B. Dali will be remembered for his paintings long after other artists are forgotten. C. Dali was capable of total focus when painting. D. Dali's bizarre public actions had rational causes.

Solution: The correct answer is D. A) The passage presents evidence against this claim, when it indicates that the Surrealists ultimately found Dali overly eccentric and "rejected" him (final paragraph). B) The final paragraph implies that "future generations" should remember Dali, but it does not support the claim that he will be remembered long after other artists have been forgotten. C) The passage does provide some evidence for this claim: "There was even fairly convincing proof that this frivolous and exuberant impresario was capable of the strictest asceticism while he was painting a picture." However, asceticism is not the same as focus, and the passage doesn't actually provide any of that "fairly convincing proof"; it just alludes to it. The evidence for option D is much stronger, as it actually explains the rational purposes Dali's actions served. D) This Reasoning Within the Text question asks you to assess the quality of evidence for the statements given in each option. The passage gives several pieces of evidence to support the claim that Dali's outrageous behavior served rational purposes. Evidence includes the passage statements: "his provocative attitudes were not adopted for publicity purposes only, but satisfied a need to keep his mind in a state of excitement that would be congenial to his artistic activity. They were useful, too, in winning immediate acceptance for his most fantastic works, because their extravagances appeared to illustrate an intellectual drama" (paragraph 3).

The author claims that actors were locked into an unequal relationship with the businessmen who controlled the U.S. theater in the early twentieth century. Which of the following statements by the author provides the strongest support for this claim? A. "[T]he economic world was divided into an unpleasant sphere where goods were produced and a gratifying sphere where they were consumed." B. Performance is an unusual commodity "in that it is a labor process exhibited before and consumed by an audience." C. "In the reconfigured theatrical economy of the early twentieth century, actors' labor was highly segmented." D. Combination companies allowed theater managers to "book several weeks of a touring company's business in a single transaction."

Solution: The correct answer is D. A) The quotation in option A does not support the point about the unequal relationships between actors and businessmen. B) The quotation in option B does not support the point about the unequal relationships between actors and businessmen. C) The quotation in option C talks about the different opportunities that were open to different actors, but this does not specifically support a point about the power held by the businessmen who controlled the theater. D) This is a Reasoning Within the Text question, as it asks you to determine which of the choices provides the strongest support for a passage claim. The answer is D, because this option describes a situation in which theater managers controlled a large number of the opportunities for employment, giving the actors much less leverage to negotiate (for higher salaries, better working conditions, etc). As the author writes, once the theater managers were able to book engagements in a whole group of theaters at once, "this strengthened their bargaining position immeasurably." Because they were able to control a concentrated number of employment opportunities at once, the actors had less opportunity to look elsewhere for a better deal. Thus, this quotation supports the passage claim that the actors were "locked into an "unequal relationship with . . . businessmen."

Which of the following assertions in the passage does the author support with a practical example? A. "[D]irect government by the people . . . greatly facilitates sudden insurrection of all kinds by individuals who are exceptionally bold, energetic, and adroit." B. "[A]nd therewith disappears also personality and sense of responsibility." C. "The adhesion of the crowd is tumultuous, summary, and unconditional." D. "[I]t is impossible for the collectivity to undertake the direct settlement of all the controversies that may arise."

Solution: The correct answer is D. A) The quotation in this option is not supported by any kind of practical, real-life example. B) The quotation in this option is not supported by any kind of practical, real-life example. C) The quotation in this option is not supported by any kind of practical, real-life example. D) This is a Reasoning Within the Text question because it asks you to assess the nature of the support that the author provides for various claims. In this case, the question asks which passage assertion is supported with a "practical example." The answer is D because the claim about the impossibility of a "collectivity" settling all of the "controversies that may arise" in a society, is supported by the example (in the final paragraph) of what happens when "Peter wrongs Paul." That example demonstrates why it is absurd to imagine that the entire "collectivity" would be able to examine the circumstances of the controversy and adjudicate it.

One can most reasonably infer from the passage discussion in the second paragraph that before 1920, there were NO: A. economic studies that provided empirical data about expenditures. B. methods available for accurately comparing the standards of living of different social groups. C. studies designed to assess the economic effect of changes in industrial production. D. nationwide studies of the expenditures of all social groups in an industrial economy.

Solution: The correct answer is D. A) This is incorrect, because expenditure surveys focusing on standards of living are described in detail in paragraph 2. B) Although the social groups described in paragraph 2 as the primary focus of pre-1920s economic studies were "near the bottom of an industrial economy," they were nonetheless different social groups. Further, it is not clear that the methodology did not exist to do the comparisons described in this option. Rather, this was not the approach that researchers took at this time. C) The excessive focus on production was one of the criticisms leveled at neo-classical economists by the institutionalists. D) This is a Reasoning Within the Text question because it asks you to make a logical inference based on passage information. D is correct because paragraph 2 indicates that the expenditure data collected in the period before 1920 came from studies that were "all . . . limited to specific regions and focused on social groups near the bottom of an industrial economy."

Which of the following, if true, most weakens the author's argument about the majority of college professors? A. Most would prefer to have a career doing something other than teaching. B. Most did not earn their degrees at the school where they teach. C. Most do not teach in areas concerned with social issues. D. Most teach in the city where they prefer to live.

Solution: The correct answer is D. A) This is irrelevant to the author's argument about professors' cosmopolitan, rather than grounded, ethos. B) This supports, rather than challenges, the author's claims. C) The author does not restrict her argument to those professors who teach in areas specifically concerned with social issues, so this would be irrelevant to the author's argument. D) This is a Reasoning Beyond the Text question; it asks how information not present in the passage, if true, would affect the passage argument about college professors. D is correct, because the passage author maintains that the majority of college professors are essentially indifferent to place, "expected to owe no allegiance to geographic territory" (first paragraph), and to belong instead to the "boundless world of ideas." But if most professors taught in the cities where they preferred to live, they would be guided by the kind of grounded, local feeling that the argument believes they repudiate—and this would weaken the argument.

Assume that in a particular state, teachers at all grade levels emphasize hands-on projects and personal interactions with individual students. The passage suggests that in this state, class-size reductions would probably: A. be more beneficial than alternative educational reforms. B. improve student performance only in the early elementary grades. C. not substantially improve student performance on average. D. improve student performance at all grade levels.

Solution: The correct answer is D. A) This is not supported by the passage; class-size reductions would still be costly, and there is not enough evidence in the passage to make this comparative claim, based on the scenario in the question. B) The improvements in the early elementary grades, according to the passage author, are due to the kinds of teaching techniques that teachers in those grades generally employ. So if teachers in all grades employed similar techniques, similar improvements would be expected in all grades, not only in the early grades. C) The passage links class-size-related improvements in student performance to the presence of teachers who use teaching techniques suited to small classes; the passage also says that the performance of these teachers and classes pulls up the average level of performance. So this option is incorrect, because performance improvements would be expected in this situation. D) This is a Reasoning Beyond the Text question because it presents a hypothetical scenario that is not discussed in the passage, then asks you to apply passage information to this scenario. The answer is D, because the passage author contends that improvements in student performance as a result of reductions in class size are found primarily in the early elementary grades, because teachers in those grades emphasize teaching techniques (such as hands-on projects) most suited to small classes. So, if teachers in all grades emphasized these techniques, then you would expect to see improvements in student performance due to class-size reductions at all grade levels, and not just in the early grades.

Which of the following is most illustrative of "the illusory correlation bias" (paragraph 3), as it is described in the passage? A. A flutist sometimes performs a difficult trill correctly, and sometimes does not, and is unable to determine the cause of her inconsistency. B. An athlete looks like she has mastered the pole vaulting technique, but since she never actually clears the bar, her coach calls her mastery an illusion. C. A teacher measures his students' performance, praises them in hopes their performance will improve, and then measures their performance again, to assess the effect of the praise. D. A dog trainer attributes a dog's correct response to "Lie down" to the dog's training, but the trainer gives the command only at the end of a tiring walk.

Solution: The correct answer is D. A) This option describes a result whose explanation is elusive (not an illusory result) and, in any case, does not involve attribution of the result, so it is not an instance of the "illusory correlation bias" as it is defined in the passage. B) This option describes something illusory, but misses the meaning of "illusory correlation bias." C) This option involves some attribution (the teacher wants to see whether the data allows her to attribute student improvement to her praise), but there is no suggestion here that there is an illusory correlation—she is essentially doing an experiment to see whether a correlation exists. D) This is a Reasoning Beyond the Text question, because it asks you to apply a concept from the passage to a scenario that is not discussed in the passage. The answer is D, because, as defined in the passage, the illusory correlation bias refers to falsely attributing results to therapy when it could have been "the natural history of the disorder [that] explains the results" (paragraph 3). Similarly, the trainer falsely attributes the dog's lying down to her training, when it was more likely due to the dog's fatigue (a natural result of a tiring walk).

It has been said that language does not merely describe reality but actually helps to bring reality into existence. Which of the points made in the passage would best support this claim? A. To imagine means to make images and move them about in one's head. B. The tool that puts the human mind ahead of the animal's is imagery. C. There is no specific center for language in the brain of any animal except the human being. D. Images play out events that are not present, thereby guarding the past and creating the future.

Solution: The correct answer is D. A) This point does not describe the images that are central to language as having an impact on or changing "reality." Instead, it emphasizes introspection. B) This point does not lend support to the author's claim about the way that language can change reality. C) Again, that language is unique to humans does not support the claim that language itself has an effect on reality. D) This Reasoning Within the Text question asks you to assess which points from the passage support a particular claim; in other words, you are asked to assess the value of evidence. This option describes images (or symbolic thinking) as essential to "creating the future." This point supports the claim that language actually has an effect on reality: if the images that are the foundation of language work to "play out events that are not present," then images, or language, serve to change the present reality into something else.

Dali once helped create a dream sequence for the film Spellbound, a psychological thriller directed by Alfred Hitchcock. Which of the following is information from the passage that best explains why Hitchcock would seek Dali's assistance? A. Dali became part of Surrealism to gain wider recognition. B. Dali's madness was greatly appreciated by the public. C. Dali was a dedicated artist. D. Dali created art based on images from the subconscious.

Solution: The correct answer is D. A) This statement is irrelevant to the scenario presented in the question. B) The passage does say that Dali's behavior in public guaranteed a "brilliant" reputation, and also that he came to represent or exemplify Surrealism to the public, but this does not shed light on Hitchcock's motivations for seeking Dali's assistance for this particular film. C) The passage does describe Dali as dedicated to his art, but this does not explain Hitchcock's motivations in seeking Dali's assistance. D) This Reasoning Beyond the Text question gives you a scenario that goes beyond the material presented in the passage, and asks you to use passage information to interpret it. Since Hitchcock's film is a "psychological thriller," it makes sense that Dali's interest in "hallucinations," and the fact that "all his phobias and his obsessions were flaunted in his behavior and his work," would appeal to Hitchcock.

Which of the following facts cited in the passage gives the strongest support for the claim that Portuguese liberals were in the minority politically? A. The queen and her son were opposed to them. B. Several important liberals were jailed by John. C. The Inquisition was still active in Portugal. D. The commoners were politically conservative.

Solution: The correct answer is D. A) This would be correct only if the passage indicated that the queen and her son were opposed to all political minorities, but the passage doesn't do so. B) This does not address whether or not these liberals were in the minority. C) The Inquisition is a conservative counter-force in opposition to liberalism, but it by no means logically follows that its existence necessarily indicates that liberals were in the minority. D) This is a Reasoning Within the Text question, because it asks you to assess the strength of evidence presented in the text. The correct answer is D, because paragraph 2 defines the components of the commoners as a highly inclusive segment of the populace: "peasants, craftspeople, tradespeople, soldiers." The aggregate number of their ranks would, therefore, vastly outnumber the relatively few groups of individuals (including scientists and poets) that comprise the liberal faction alluded to in the passage (paragraph 3).

The single change to the observed ritual at Mariani that would most weaken the author's distinction between Mariani and Nansoucri voodoo would be a situation in which: A. all possessions were outwardly apparent. B. rituals included elements of danger. C. audience members were invited to dance with the hounsis. D. one or more audience members became possessed.

Solution: The correct answer is D. A) This would not weaken the distinction, as the author describes outwardly apparent possessions at Mariani. B) This is consistent with the descriptions of the sessions at Mariani, as "animal sacrifice and hounsis leaping into fire" are included in the sessions. C) This would not weaken the distinction, as it would still preserve the staged, "choreographed" element of the sessions. D) This is a Reasoning Beyond the Text question, which presents a series of hypothetical scenarios not discussed in the passage and asks you how these would affect passage arguments. The answer is D, because the author's distinction between the two voodoo forms turns on the fact that the Mariani session is a "theatrical" session, in which the audience essentially watches the action from a "distance . . . that disallows real transformation (inadvertent possession)" (paragraph 3). Thus, if audience members at Mariani did indeed become possessed, the author's distinction between Mariani voodoo and the more participatory, ritual form of voodoo at Nansoucri would break down.

The author suggests that adjustment is difficult for the poorest immigrants to the U.S. because of: A. an expectation that the status of their children would exceed their own. B. an unexpected discrepancy between the new culture and the familiar one. C. prejudices by employers that prevent their finding employment. D. a tendency for particular problems to affect many aspects of their lives.

Solution: The correct answer is D. A) While the passage says that people who hope to ensure their children's economic mobility may be more likely to emigrate than others, it does not suggest that this is what makes adjustment difficult, nor that this makes adjustment particularly difficult for poorer immigrants. B) While the end of the passage discusses the "profound" changes that immigration can bring, it does not suggest either that these changes are unanticipated or that they are an element of what makes adjustment especially difficult for the poorest immigrants. C) The passage does not imply that the difficult adjustment for the poorest immigrants to the U.S. specifically stems from prejudices among potential employers. D) This Comprehension question tests your understanding of a point that is made explicitly in the passage, but in different language than is used in the question. The author writes, "At the bottom of the stratification heap, any source of stress tends to be pervasively disruptive in its effects" (paragraph 2). This is another way of saying that for the poorest immigrants, any problem may affect many different aspects of their lives.

Which of the following aspects of the mounds described in the passage best explains archaeologists' reluctance to postulate a strictly Native American origin? A. Their apparently sudden appearance B. Their relatively early appearance C. Their size D. Their purpose

The solution is A. A) "Archaeologists remained reluctant...to postulate a strictly Native American origin for mound building. Instead, diffusion of the practice from some area where it was already well established seemed to be the best way to explain its apparently sudden and relatively late appearance in the eastern Woodlands of the U.S." (paragraph 2). B) To the contrary, the author suggests that it was the "relatively late appearance" of mound building (not its early appearance) that contributed to the diffusionist hypothesis. C) The size of the mounds led to speculation about the size and nature of the labor force involved in their construction, but the author does not mention the size of the mounds as an aspect of the reluctance of archaeologists to see the mounds as of strictly Native American origin. D) The purpose of the mounds is not discussed; the author calls them "enigmatic" mounds. In any case, their purpose is not given as a reason that archaeologists were reluctant to posit a strictly Native American origin for the mounds.

Implicit in the author's discussion of Milgram's folders (paragraph 5) is the belief that: A. people will cooperate more fully if they think they are involved in something official. B. the design of the folders confused the participants. C. something more complicated than a simple chain letter does not work well in such experiments. D. going from rural areas to urban areas works best with a "folder-mailing" design.

The solution is A. A) According to the author, Milgram attempted to explain why so few folders made it through to their intended targets by suggesting that people "didn't bother sending the letters on." But the author dismisses this line of argument: "The folder, however, was not a simple chain letter, but an official looking document with heavy blue binding and a gold logo. If the subjects knew how to reach the targets, they probably would have" (paragraph 5). The author clearly implies that a more "official"-seeming situation will provoke more participation. B) The author doesn't imply that there is anything "confusing" about the folders. Instead, he or she indicates that the folders have an "official-looking" appearance that would be likely to promote cooperation. C) The author does not suggest that anything about the letters themselves contributed to the letters not reaching their targets. To the contrary, he suggests that their "official-looking" appearance would have been likely to increase the chances that the letters would reach their targets—more so than if the folders had appeared to be "simple chain letter[s]" (paragraph 6). D) The author does not suggest anything about the type of folder that would be more likely to be sent on from rural to urban areas.

Which of the following best describes the author's attitude toward the subject of the passage? A. Intrigue B. Disbelief C. Confusion D. Approval

The solution is A. A) After an introductory paragraph in which the author explains that contemporary research has begun to debunk earlier and more limited portrayals of the capacities of the Neandertals, the author presents the work of several scholars whose research testifies to the achievements of the Neandertals. The passage then concludes with expressions of intrigue: "Taken together, the studies leave researchers with many further mysteries to consider.... But if the two groups' behaviors were similar, why did Neandertals disappear?"​ B) The author describes the puzzle that the Neandertals' fate presents to scholars, but he or she does not describe any feelings of disbelief about their fate. C) The author presents an intriguing puzzle: "Many investigators now agree that anatomical differences alone cannot explain the Neandertals' fate. But if the two groups behavior was similar, why did Neandertals disappear?" However, the author does not express confusion or lack of understanding, just curiosity. D) The author presents the views of anthropologists and archaeologists about Neandertals as these have evolved and explains their increasing appreciation of the Neandertals' accomplishments. The author, however, does not express "approval" nor "disapproval" about the passage subject.

Suppose that a child persists in fearing a monster despite assurances by adults that the fear is unreasonable. Which of the following reasons for this fear would undermine the passage author's argument? A. The fear is instinctual and beyond the child's control. B. The child assumes that adults cannot see the monster. C. The child believes that the adults are untrustworthy. D. The fear is a displaced expression of separation anxiety.

The solution is A. A) In challenging Plato's view that emotions are irrational and thus dangerous, the author writes, "Moreover, there is evidence that even strong emotions possess a sort of rationality—that is, they are governed by thought, and they are educable. We try to convince a child not to fear an imagined monster underneath the bed by proving that no monster is there" (paragraph 4). If, however, the child's fear persisted despite such assurances, and the fear that persisted was "instinctual and beyond the child's control," this would challenge the author's claim that reason is a component of emotion and that emotion can be taught or corrected. B) If the child who fears the monster does not believe that the adults are able to see the monster, then the child is not able to trust the adults as a source of information. This does not undermine the author's argument that reason is a component of emotion and that emotion can be modified by reason—the child is just not yet persuaded by the adults' argument because he or she does not yet trust its basis. C) If the child does not believe that the adults who are speaking to him or her are trustworthy, this suggests that the child is using reason to discount the information that the adults are giving him or her. This does not mean that if the child comes to trust the adults, that the child's emotion will not ultimately be "educable." D) If the fear that the child experiences is a result of separation anxiety—a form of emotion—this does not undermine or invalidate the author's contention that such fear can be susceptible to modification by education. The fear, regardless of its source, could still be subject to modification.

Which of the following statements best summarizes the fourth paragraph? A. There are many ways that writers reveal themselves in their writing. B. It makes a big difference if a writer's work is subjective and not objective. C. Most writers' lives are chaotic, and they create order through writing. D. Many writers want their lives to be visible in their written work.

The solution is A. A) In paragraph 4, the author enumerates the different ways in which writers may reveal themselves in their work: "Some writers want to expose themselves, some to disguise themselves, some to efface themselves....There is more than one way to impose order on your personal chaos, but since good writers write what's important to them, they are bound to be in there somewhere, as participants or observers or ombudsmen." The point of the paragraph, then, is that writers do, in one way or another, ultimately reveal themselves in their work, and they may do this through a variety of means or strategies or approaches. B) The author writes instead that "the differences between the extremes of 'objectivity' and 'subjectivity' are obvious but not critical." This particular distinction, then, is not the main point of the paragraph. C) The author does write that "there is more than one way to impose order on your personal chaos," but this really suggests that everyone's life or interior self has chaotic elements, not just writers' lives. Further, that writers write to impose "order" on "chaos" is not the primary point of the paragraph. D) This is not the main point of the paragraph, which says explicitly that different writers seek to be "visible" in their own work to different degrees: "Some writers want to expose themselves, some to disguise themselves, some to efface themselves."

d'Errico's argument about the Neandertals and symbolism (paragraph 3) necessarily requires which of the following? A. Proof that Neandertals used jewelry as symbols B. Proof that Neandertals invented jewelry to wear as symbols C. Proof that Neandertals copied jewelry designs from modern humans D. Proof that Neandertals traded with modern humans to obtain jewelry

The solution is A. A) The argument d'Errico makes is that the way that Neandertals used symbols "was not qualitatively different from that of anatomically modern humans" (paragraph 3). It is the use of jewelry as symbols ("their way of using symbols"—paragraph 3) that proves that Neandertals were capable of symbolic thought, which is clearly essential to d'Errico's claim. B) It is merely the use of jewelry as symbols that proves that Neandertals were capable of symbolic thought and is essential to d'Errico's point; invention of the jewelry is not necessary to his claim. C) The author quotes d'Errico as saying that "We do not know whether Neandertals developed the use of symbols independently or as a consequence of cultural contacts with modern humans. What we know is that their way of using symbols was not qualitatively different from that of anatomically modern humans" (paragraph 3). It is in no way essential to d'Errico's argument that Neandertals copied jewelry designs from modern humans: in fact, the author tells us, d'Errico and "his colleagues would even...propos[e] the novel idea that Neandertals may have taught modern humans how to make some objects" (paragraph 3). D) It is merely the use of jewelry as symbols that proves that Neandertals were capable of symbolic thought, which is d'Errico's point. That they traded with modern humans to obtain the jewelry is in no way essential to d'Errico's argument.

Implicit in the argument that emotions are not equivalent to feelings is the idea that emotion: A. concerns a specific object or situation. B. is genuine only if chemically induced. C. involves clear physiological changes. D. is active during the creation of art.

The solution is A. A) The author argues in paragraph 3 that: "Emotions are not simply bodily sensations.... Suppose that the set of physiological changes you experienced the last time you were angry is chemically induced. You are then left alone in exactly the same feeling state you were in at a meeting the previous month, after a colleague spoke to you sarcastically. Are you angry? No, you are only uneasy, restless—unless there is something or someone giving you cause for anger" (paragraph 3). The author implies that "something or someone" is necessary to make a feeling (a sensation) into an emotion, which suggests that an emotion indeed concerns a "specific object or situation." B) The author uses the example of chemically inducing a feeling state (paragraph 3) to differentiate a simple feeling from an emotion. That is, the author indicates that this chemically produced state is insufficient to create an emotion, which needs "something or someone" to attach to it. C) The author describes the contention that "emotion is a physiological condition" (paragraph 2), in order to argue that this is not an adequate definition of an emotion: "Emotions are not simply bodily sensations" (paragraph 3). The difference between emotion and feeling for the author is that an emotion requires not only physiological changes, but also a "belief or thought about some person or situation." D) The author discusses emotion as it is a component of a reaction or response to art—not as it plays a role in the creative process. Further, the role of art in the creative process is not part of the distinction the author makes between emotion and feelings.

The passage implies that relative to art critics, film critics are: A. more forthright. B. more respected. C. less pretentious. D. less qualified.

The solution is A. A) The author describes a "recent Weekend section of The New York Times" in which "all ten [art] gallery reviews were favorable, even though the praise tended to be of the tepid, hedging, irresolute, variety. One can hardly imagine the book, film, theater, music, or even food critics of the Times succumbing to such a degree of chinlessness" (paragraph 3). The clear implication here is that film (and other reviewers) are far more direct than are the art reviewers. B) The author does not imply anything about the relative degree of respect commanded by film critics and art critics. C) Pretentiousness is not a critical characteristic that the author examines. D) There is no discussion about the relative degree of "qualification" of film critics and art critics in the passage, although the author does suggest that he or she has more respect for film critics, who appear not to hesitate to speak their minds.

The author describes the research of Cyrus Thomas most likely because it: A. discredits the hypothesis that mounds were built by a vanished culture. B. supports the hypothesis that people from the area that is now Mexico influenced Native American mound building. C. establishes when mound building occurred in the eastern Woodlands. D. challenges the diffusion hypothesis put forth by archaeologists.

The solution is A. A) The author writes explicitly that Cyrus Thomas had "convincingly refuted" the "long-popular idea that the mounds were the work of a vanished culture" (first paragraph). B) The author indicates that Thomas "proved...that the mounds had been raised by early Native Americans" (first paragraph), not that the Native Americans had been influenced by people from the area that is now Mexico. C) The author indicates that Thomas "proved...that mound building had continued in some areas up until the arrival of the Europeans" (first paragraph), but this is not the same as establishing when the mounds were built. Further, the passage is most concerned with discussing the origin of the mounds and evaluating the case for the Mexican hypothesis, so the author most likely cites Thomas because of his refutation of the argument that the "mounds were the work of a vanished culture." D) Thomas does not address the diffusion hypothesis directly.

A researcher applying Watts's mathematical models as described in the passage to research on the transmission of an airborne contagious disease can most reasonably assume that: A. a small number of individuals could cause a widespread distribution of the disease. B. all infected individuals would contribute equally to the distribution of the disease. C. widespread disease transmission is more likely in a society with many isolated subcultures. D. disease transmission among individuals who do not know each other is highly unlikely.

The solution is A. A) The author writes that Watts and colleagues at Columbia "have created mathematical models that show how a small world could work. Random connectors in a network, such as especially sociable people who have friends across subcultures, can vastly decrease the distance between points in a network" (paragraph 6). These "random connectors," who can be a small number of strategically connected people, can have a disproportionate impact on the possibility of transmitting the folders. This would be analogous to a small number of people helping to transmit a disease more "efficiently." B) To the contrary, Watts's work shows the disproportionate impact of "random connectors, such as especially sociable people who have friends across subcultures" (paragraph 6). This would not imply that everyone contributes equally to disease distribution. C) To the contrary, Watts's work suggests that it is not "isolated" subcultures that contribute to the transmission of information. Rather it is the possibility of creating links to and among those subcultures that increases the chance of information—or disease—transmission. D) Watts's research shows, instead, how a "small world could work," especially when certain "random connectors," who are particularly sociable, are able to form connectors across subcultures. Thus, applying Watts's mathematical models would show that disease transmission among individuals who do not know each other is possible under certain circumstances. The model does not support a suggestion that such transmission is "highly unlikely."

According to Spencer's views, if a wealthy person gives money to the poor, which outcome is to be expected for the giver? A. A heightened sense of virtue B. A heightened sense of social responsibility C. A reduced sense of social responsibility D. A reduced sense of guilt

The solution is A. A) The author writes, "Individual charity might refine the character of the donor...." (paragraph 6). The reference to a "refined," or improved character, suggests a "heightened sense of virtue" might accrue to the wealthy person who gives money to the poor. B) The passage does not suggest that Spencer believes that wealthy people who give money to the poor will gain any sense of "social responsibility" from doing so, nor should they, according to Spencer's view. Rather, he prioritizes "individual liberties" (first paragraph). C) The passage does not suggest that Spencer believes that a wealthy person who gives money to the poor will end up with a reduced sense of "social responsibility." Rather, he or she might end up with a "refine[d]...character" (paragraph 6). D) The passage says nothing to imply that Spencer believed that wealthy people who assisted the poor would reduce any sense of guilt that the wealthy might feel—nor does it suggest that Spencer in any way felt that the wealthy should experience guilt.

Passage context implies that Spencer's strong following in the United States during the nineteenth century was attributable, in part, to the country's: I. relatively substantial number of affluent residents. II. relatively strong sense of social responsibility. III. relatively strong grounding in Darwinist theories. A. I only B. III only C. I and II only D. II and III only

The solution is A. A) The author writes, "Nowhere did Spencer have a larger following than in the U.S., where his works were celebrated as powerful justifications for laissez-faire capitalism" (first paragraph). The author, then, implicitly attributes Spencer's popularity in the U.S. to his support for capitalism, and it follows that those who were prospering under this system would be the ones who found this support most attractive. B) There is no evidence in the passage that those in the U.S. who admired Spencer had particular knowledge of Darwin. Although the passage indicates that Spencer was "well known as an evolutionist," it also says that this was true "long before" Darwin's On the Origin of Species was published" (paragraph 5). Thus, Spencer's reputation as an evolutionist did not hinge on his affiliation with Darwin. C) Although the first paragraph supports the claim that Spencer's following in the U.S. was due in part to the relatively large number of affluent residents in the country, the passage does not support the claim that a relatively strong sense of social responsibility contributes to Spencer's following. To the contrary, the author describes Spencer as promulgating a very limited view of social responsibility: "Governments had no obligation, or even right, to compel vaccination or education, keep small children from sweeping chimneys, mandate the construction of sewers, or relieve poverty" (paragraph 6). If Spencer advocated such views about the limits of social responsibility, there is no reason to attribute Spencer's popularity to a "relatively strong sense of social responsibility" pervasive in the U.S. D) The passage does not support the claim that a relatively strong sense of social responsibility contributed to Spencer's following; to the contrary, paragraph 6 shows that Spencer promoted a very limited view of social responsibility. The passage does not provide evidence supporting the idea that those in the U.S. who admired Spencer had particular knowledge of Darwin. Although the passage indicates that Spencer was "well known as an evolutionist," it also says that this was true "long before" Darwin's On the Origin of Species was published" (paragraph 5).

What is the main topic of the passage? A. Solutions to problems with the geographic distribution of healthcare workers in the U.S. B. Proposed revisions to the current urban and rural healthcare systems in the U.S. C. Health risks experienced disproportionately by those living in rural areas in the U.S. D. Inequities in healthcare quality based on geographic region of the U.S.

The solution is A. A) The geographic maldistribution of health professionals is identified in the first paragraph as one of the characteristics causing limited access to health care by rural Americans; paragraph 2 goes on to suggest one solution ("The medical education system could help to increase rural access to health care by selecting, training, and deploying..."); paragraph 3 recommends another mechanism "to improve the flow of health professionals to rural areas..."); paragraph 4 identifies a third solution ("When educational interventions and economic incentives do not improve the geographic distribution..."); and paragraph 5 suggests a fourth solution, involving the "emerging technology of telemedicine." B) While the passage does propose revisions to the current healthcare system (with a focus on rural, rather than urban areas), the purpose of such revisions is to solve the problem of the geographic maldistribution of health professionals in the U.S. C) Although there is some reference to health care issues experienced in rural areas in the U.S. (namely, limited access to health care—first paragraph), there is no specific discussion of health "risks" that disproportionately affect those living in rural areas—and this is certainly not the main topic of the passage. D) Although there is some oblique reference to the greater access to health care experienced by those in urban areas compared to those in rural areas, inequities are not the main topic of the passage.

Which of the following, if true, would most strengthen Zilhão's hypothesis (paragraph 2) regarding the ornaments found at Neandertal sites? A. They predate modern-human sites. B. There are more ornaments made of teeth than tusk. C. They look identical to ornaments found at modern-human sites. D. They are typically found at what are believed to be Neandertal burial sites.

The solution is A. A) Zilhão's hypothesis is that the Neandertals made the ornaments themselves (they didn't get them by trade or finding them), independent of any modern-human influence (they didn't copy them). If the Neandertal ornaments date to a time before the modern humans, then that would strengthen the hypothesis. B) Zilhão's hypothesis is that the Neandertals made the ornaments themselves, independent of any modern-human influence whether there are more ornaments made of teeth than tusk is irrelevant. C) If the ornaments are identical to ornaments found at modern-human sites, that would weaken rather than strengthen the hypothesis that the Neandertals made the ornaments themselves. It would offer more evidence that the Neandertals might have copied the ornaments from modern humans, or traded for them. D) That the ornaments are found at likely Neandertal burial sites would neither strengthen nor weaken Zilhão's hypothesis about how those ornaments came to be.

The passage suggests that the "national virtue" Spencer had assaulted during his banquet speech was U.S. residents' tendency: A. to give large amounts of money to charities. B. to devote large amounts of time to pursuing a livelihood. C. to believe that competition, rather than hard work, brings wealth. D. to be ostentatious in displaying their wealth.

The solution is B. A) In fact, the author writes that Spencer believed that "[i]ndividual charity might refine the character of the donor" (paragraph 6), so this was not the "national virtue" he would have critiqued. B) The author describes Spencer's speaking appearance at a banquet full of "distinguished American Spencerians" (paragraph 2), many of whom praised Spencer lavishly: "Instead of graciously accepting these tributes, Spencer told his admirers that they had seriously misunderstood him. He did not approve of the culture of American capitalism.... He felt that for Americans, work was a pathological obsession that endangered mental and physical health" (paragraph 4). The author then refers to Spencer's remarks as a "slap to the face of national virtue" (paragraph 4). C) To the contrary, Spencer assaulted the American tendency to work to excess, and argued that it was time to "'preach the gospel of relaxation'" (paragraph 4). The author also indicates explicitly in the first paragraph that Spencer supported competition, so he would not have believed that Americans over-valued that quality at the expense of hard work. D) The passage clearly indicates that Spencer "admired [the] material achievements of American capitalism" (paragraph 4), and there is no suggestion that he frowned on ostentation.

What is the main point of the first paragraph? A. A writer must be faithful to real life to be successful. B. A work of fiction reveals its creator's strengths and weaknesses. C. A writer is like a camera that perfectly reproduces real life. D. A writer's astigmatisms are what make a work of fiction fascinating.

The solution is B. A) In the first paragraph, the author describes Stendhal's view that fiction should be "faithful to real life," but challenges this view as a prescription for successful writing: Fiction "is more than the mirror in the roadway that Stendhal said it was. Because a good writer is more than a mirror; a good writer is a lens." B) The author emphasizes in the first paragraph that writers do not simply "reflect" reality in a mechanical way. Rather, each writer has a specific "lens" that informs his or her representation of reality. He concludes the paragraph: "there is no escaping the fact that fiction is only as good as its maker. It sees only with the clarity that the writer is capable of, and it perpetuates the writer's astigmatisms." The main point of the paragraph, then, is that fiction demonstrates, or reproduces, both a writer's strengths (or "clarity") and weaknesses (or "astigmatisms"). C) The author disputes the idea that a writer is even capable of being a "camera that perfectly reproduces real life." Instead, he suggests that fiction reflects its author's idiosyncratic perspective: "It sees only with the clarity that the writer is capable of, and it perpetuates the writer's astigmatisms" (first paragraph). D) In the first paragraph, the author describes the specificity of vision that characterizes each writer's work, as he refutes Stendhal's depiction of the writer as a "mirror" of reality. However, the paragraph does not focus on what makes fiction "fascinating," and instead emphasizes the point that fiction reveals both a writer's "clarity" of vision and his or her perceptual flaws.

If a researcher studying rumor transmission among individuals were to use the same methodology as that of Milgram's small-world experiment, which of the following actions would be part of the methodology? A. Making sure starters and targets are located in the same city B. Ignoring cases in which a starter does not repeat the rumor C. Offering a financial incentive for completed rumor transmissions D. Submitting the data to mathematicians for analysis

The solution is B. A) Milgram's model involved asking "starters from places such as Nebraska to mail a folder to a target in major cities, such as Boston" (paragraph 3). B) The author explains that Milgram's explanation for the low rate of folders that made it to their ultimate targets in his published studies was that "people didn't bother sending the letters on" (paragraph 5). The author implies that Milgram attempts to discount such situations, rather than seeing them as presenting a problem with his theory itself. This would be analogous to a situation in which researchers overlooked, or ignored, those people who did not repeat a rumor. There is no indication in the passage that Milgram offered participants any financial incentive as part of the experiment. C) The passage says only that Duncan Watts and colleagues have "created mathematical models that show how a small world could work" (paragraph 6). D) There is no indication that Milgram submitted his own data for such analysis.

In Plato's opinion as the passage author represents it, the relationship of emotion to reason is most like that of: A. paint to painting. B. disease to health. C. body to soul. D. water to ice.

The solution is B. A) Paint is an essential component of painting, the material that allows one to produce a painting. By contrast, in Plato's view, emotion corrupts reason; it does not enable it. B) The author describes Plato's belief that the arts had a corrupting influence on society: "His argument hinged on a conviction that the emotions are irrational, in the sense that they undermine the rule of reason in the individual and, in consequence, in society.... Plato concluded [that] works of art address irrational tendencies and thus undermine our ability to control ourselves through reason" (first paragraph). In the author's description, Plato clearly sees the "controlled" state that reason brings as the desirable (or healthy) state, and the "irrational" role of emotion as the disease that threatens health. C) In Plato's view, emotion is the irrational element that threatens the healthy, socially beneficial operations of reason. The relation between body and soul is different: body might be described as the corporeal element of the self, while the soul is the spiritual or ineffable element of the self. The two elements are both separate and conjoined; the element of body does not threaten the soul. D) Water is the precursor to ice; it can be transformed into ice at a low enough temperature. Plato, as the author describes him, does not believe that emotion can be made into reason; emotion, rather, threatens to undermine or dismantle reason.

Which of the following, if true, would most weaken Shea's hypothesis regarding wood-tipped and stone-tipped spears (paragraph 5)? A. Stone-tipped spears are more durable than wood-tipped spears. B. Wherever both types of spears are found, only one kind of prey animal is found. C. Wherever Neandertal sites are found, there is a variety of crafting tools as well as hunting tools. D. Both wood-tipped and stone-tipped spears are found in modern-human archaeological sites.

The solution is B. A) Shea's hypothesis has nothing to do with durability; it has to do with the "varied arsenal" that the Neandertals created for hunting (paragraph 5). B) The author explains that Shea's "study of stone points from fifty-eight Neandertal sites shows both wood-tipped spears and stone-tipped spears, yielding the hypothesis that each type of tool was for a different kind of prey" (paragraph 5). If there was only one type of prey found, this would clearly weaken the hypothesis that the tools were crafted for different kinds of prey. C) A variety of crafting tools doesn't affect Shea's hypothesis about the use of a variety of hunting tools. D) If modern humans also created more than one type of spear, this would not weaken Shea's claim about Neandertals and their creation of specific tools for specific hunting purposes.

Some theater critics disdain the popularity of the "golden age" Broadway musicals, preferring less accessible theater that they would designate as "high art." How would this view most likely be regarded by the passage author? A. The passage equation of commodity and golden age musicals suggests that the author would agree with the critics. B. The passage discussion of golden age musicals suggests that the author would regard these critics as unnecessarily dismissive. C. The passage discussion of the "common musical language" of the non-commodity musical suggests that the author would be aligned with these critics. D. The passage position on the cost of contemporary Broadway theatrical productions suggests that the author would reject the critics' views.

The solution is B. A) The author champions the "old-fashioned Broadway musical, with its optimism and burgeoning musical energy" (final paragraph), and regrets that it is "gone for good." B) The author very much likes the "old-fashioned Broadway musical, with its optimism and burgeoning musical energy" (final paragraph), and laments its demise. The author also talks about the rooting of the "golden-age' musical" in what he or she calls a "universally popular musical language" (paragraph 4). The author, then, does not dismiss the popular; the author dismisses the inauthentic and clearly derivative. C) The author makes clear in paragraph 4 a regret that the "common musical language" no longer exists. D) Though the author does argue that musicals are so expensive to mount that only commodity musicals can make it to the contemporary Broadway stage, he or she does not use the expense of the productions as grounds for his or her aesthetic preferences and would not dismiss those of the critics on those grounds, either.

The passage author's assertions imply that concern about undesirable social effects would be justified by a: A. tragic play that induces grief and horror. B. novel that promotes belief in demonic possession. C. political play that incites anger at authorities. D. lyrical poem that arouses sensual desires.

The solution is B. A) The author contends that "the only danger of art to society is not that it excites the passions but that it may instill false beliefs" (final paragraph). This option describes a work of art that does "excite the passions." There is no reason to think that the "grief and horror" stemming from tragedy promotes false beliefs—and thus, produces undesirable social effects. B) The author writes that "some emotion-producing representations—those that encourage defective cognitive states—are affronts to reason. That is, the only danger of art to society is not that it excites the passions, but that it may instill false beliefs" (final paragraph). A novel that promotes belief in demonic possession would, of course, be an instance of a work of art "instilling false beliefs" and would justify the author's concern about the undesirable social effects of art. C) A political play that "incites anger at authorities" would, indeed, excite the passions, but would not necessarily "instill false beliefs." Thus, the author would not see this play as likely to produce undesirable social effects. D) A poem that arouses "sensual desires" would stir passions but there is no reason to think it would "instill false beliefs." As the author is concerned about the latter kind of art having undesirable social effects, the poem would not justify concerns about the undesirable social effects of art.

Why does the author most likely mention 9 to 5 (paragraph 2)? A. To make a point about the titles of commodity musicals B. To illustrate the closeness of adaptations to sources in commodity musicals C. To indicate that casting is the most important element of a successful adaptation D. To provide an example of a Broadway musical that was based on a Hollywood movie

The solution is B. A) The author does make the point that adaptations retain the title of the source, but 9 to 5 is not mentioned to support that point. B) The author writes, "The shows may also be cast to evoke their source material: the stars of the stage version of 9 to 5, for example, bore an unmistakable resemblance to their on-screen models" (paragraph 2). C) There is no support for designating casting as the most important element of a successful adaptation. D) The author certainly indicates that many movies have been adapted to the Broadway stage, but does not mention 9 to 5 specifically to support that point.

Based on the passage, it can be reasonably inferred that the intelligentsia accepted Milgram's theory because it: A. was adopted by the media and movies. B. appealed to their imaginations. C. could be modeled mathematically. D. ignored race and class boundaries.

The solution is B. A) The author writes, "the idea of six degrees of separation was adopted by the intelligentsia and it has turned up in the media and in the movies" (paragraph 2). But this doesn't mean that the intelligentsia adopted it because it turned up in the media and movies. Rather, the chronology the author presents suggests that the intelligentsia adopted the idea either before or at the same time as the idea became prevalent in film and other media. B) The author writes that "Milgram's findings slipped away from their scientific moorings and into the world of imagination. The idea of six degrees of separation was adopted by the intelligentsia..." (paragraph 2). The author implies, then, that the imaginative appeal of Milgram's work was the reason it gained a foothold with the intelligentsia. C) The author explains that "Duncan Watts and his colleagues at Columbia University have created mathematical models that show how a small world could work" (paragraph 6). But this was not what Milgram did, nor does the author imply that the "intelligentsia" accepted Milgram's work when he did it because of its susceptibility to mathematical modeling. D) The author says only that in attempts to replicate Milgram's findings across cities, "few folders made it through, especially across class and race boundaries" (paragraph 4). This does not suggest that the theory appealed to the intelligentsia because of a lack of acknowledgement of any such boundaries.

Suppose that psychological studies show that novels arouse emotion only if readers consider the characters and situations described realistic. What is the relevance of this finding for the passage author's conclusions? A. It affirms them by implying an irrationality that serves human survival. B. It affirms them by implying a reasoned engagement with the narrative. C. It violates them by implying a logically unjustified emotional response. D. It violates them by implying that this form of art opposes reason.

The solution is B. A) The described scenario implies readers' rationality, not their irrationality; they demand a sense of realism from the characters and scenarios in a novel before they will react emotionally to a novel. Further, the author says that "we have emotions because...they contribute to the survival of our species" (paragraph 6). The author does not say that these emotions are irrational, or that irrationality is essential to survival. B) The author argues, ultimately, that "the emotions elicited by art do not subvert human reason, since reason—memory, associative conceptualization, judgment—is an inextricable constituent, indeed a determining force, of such responses" (final paragraph). If readers respond emotionally only to those novels that strike them as realistic, this indicates that their judgment plays a strong role in enabling emotional response to art and affirms the author's argument. C) To the contrary, the question describes a logistically justified emotional response to art—readers must find characters and situations credible before they will react emotionally to them. This would then support the author's contention that art does not promote irrationality. D) The scenario described in the question stem does not describe art that opposes reason; rather, it describes readers who will respond to art only if it satisfies their reason.

For which of the following questions does the passage fail to provide any information about Spencer's likely response? A. If Spencer's audience believed him to be a staunch advocate of capitalist culture, did they fully grasp his ideas? B. If the government doesn't build sewer systems, who should build them? C. If a country has many affluent people, why shouldn't they help the less affluent ones? D. If people want to be wealthy, why shouldn't they work tirelessly until they achieve wealth?

The solution is B. A) The passage clearly indicates Spencer's likely response, when the author writes that Spencer told his audience at the banquet that "they had seriously misunderstood him. He did not approve of the culture of American capitalism" (paragraph 4). B) The author describes Spencer's view that "[g]overnments had no obligation, or even right, to compel vaccination or education, keep small children from sweeping chimneys, mandate the construction of sewers, or relieve poverty" (paragraph 6). The passage does not offer any information about or insight into Spencer's view about who would build sewer systems in place of the government. C) The author describes Spencer's likely response to this question in paragraph 6: "...relief from the state, while engendering irresponsibility in recipients, embittered those whose resources funded it." D) The author offers Spencer's response to this question, when it details his objection to the "culture of American capitalism," in spite of his admiration for "its material achievements"; Spencer viewed excessive devotion to work as "a pathological obsession that endangered mental and physical health" (paragraph 4).

If cord-marked Woodland pottery were found in areas between the Bering Strait and the eastern Woodlands, this would support the notion that mound building diffused from: A. Europe. B. Asia. C. Scandinavia. D. Mexico.

The solution is B. A) The presence of cord-marked pottery in areas between the Bering Strait and the Eastern Woodlands is interpreted by the author as a potential sign of mound building having diffused from Asia, not from Europe. B) The author writes that, "A few archaeologists suggested that mound building had diffused, along with cord-marked pottery found in the Woodlands, across the Bering Strait from Asia, but they had to admit that the absence of both traits in the intervening regions was problematic" (paragraph 2). If, however, cord-marked Woodland pottery were found precisely in the intervening regions (between the Bering Strait and the Eastern Woodlands), this would presumably remedy that "problematic" lack of evidence and offer support for the theory that the mound-building had also diffused from Asia. C) The author mentions "another theory" that the mounds might have "been brought across the Atlantic from Scandinavia" (paragraph 2), but the presence of the pottery in the areas referred to in the question stem is not discussed in relation to that claim. D) The presence of the pottery between the Bering Strait and the Eastern Woodlands is discussed as evidence that the mound-building might have diffused from Asia, not from the southern route of Mexico.

Which of the following best describes the relationship of the reference to Talley (paragraph 2) to the passage as a whole? A. It questions the importance of a previously mentioned problem. B. It supports a previously mentioned solution. C. It enables critique of a scenario mentioned later in the passage. D. It provides understanding of a paradox mentioned later in the passage.

The solution is B. A) The reference to Talley supports a solution to a previously mentioned problem (the tendency of healthcare professionals to live and practice in "relatively affluent urban and suburban areas," from the first paragraph). The Talley reference does not challenge the importance of that problem. B) A previously mentioned solution to the problem of "the tendency of healthcare professionals to locate and practice in relatively affluent urban and suburban areas" (first paragraph) is for the medical education program to work at "selecting, training, and deploying more healthcare workers who choose to practice in rural areas" (paragraph 2); the four parts of the Talley reference support that solution, providing evidence suggesting the solution will succeed. C) There is no later-mentioned scenario for which critique is enabled by the Talley reference. D) There is no later-mentioned paradox.

The specific mention by name of Promises, Promises (paragraph 3) is: A. ironic. B. incidental. C. dismissive. D. significant.

The solution is B. A) There is nothing in the passage that would make the mention an ironic one. B) The author is merely making a point about ticket prices, and the point is not dependent at all on that particular play; at most, the point is dependent on the year that play opened. C) There is no dismissal or deprecation made or implied. D) The author is merely making a point about ticket prices, and the point is not dependent at all on Promises, Promises in particular. It is simply used as an example to describe the increase in ticket prices during a particular time period, so the mention of the play by name is not significant.

What idea about art critics does the author mean to convey by the statement: "This mindset begins to make sense when one considers that critics have traditionally missed the point about the best art of their generation" (paragraph 4)? A. They are indifferent to the goals of particular artists today. B. They are too impressed by creativity to appreciate good art. C. They are afraid to commit themselves to definite opinions. D. They are unaware of the standards usually applied to art.

The solution is C. A) Although the author describes the almost reflexive praise that he sees contemporary critics doling out to artists as often of the "tepid, hedging, irresolute variety," there is no support for the claim that the author believes that critics are "indifferent" to the goals of contemporary artists. B) The author maintains that contemporary culture "has been witnessing a depletion that is periodically visited upon a civilization's powers of creativity" (paragraph 2). The author, then, perceives a dearth of creativity, and does not argue that critics over-value creativity, but that they are too accepting of the art that does exist. C) The author maintains that what he characterizes as the "timidity...of art criticism of our day" (paragraph 3) comes out of a fear of misunderstanding or overlooking art that may come to be evaluated by future generations as "great art." He describes what he sees as behind the fearful mindset of contemporary critics: "Never, critics resolve, will they miss the boat as embarrassingly as their predecessors did when confronted with impressionism and cubism. Better a thousand nonentities should be heralded as the new Picasso than that the new Picasso should be met with scorn or neglect!" (paragraph 4). D) This is not supported by the passage.

Suppose it is true that the expression "survival of the fittest" was originally coined by Spencer and later adopted by Darwin. Is this fact relevant to any of the ideas in the passage? A. No; it has no relevance because the expression is not mentioned in the passage. B. Yes; it weakens the claim that Darwin restricted his theories to the biological domain. C. Yes; it supports the characterization of Darwin as a "biological Spencerian." D. Yes; it supports the university president's praise of Spencer's "powerful intellect."

The solution is C. A) Although the phrase "survival of the fittest" is not mentioned in the passage, the passage does characterize Spencer as "well-known as an evolutionist long before Darwin's On the Origin of Specieswas published" (paragraph 5). And the same paragraph goes on to contrast the social relevance or reach of the ideas of Spencer and Darwin. The scenario described in the item stem is indeed relevant to the passage. B) Even if Spencer, rather than Darwin, had coined the expression "survival of the fittest," and then Darwin had adopted the expression, this would not weaken the passage claim that Darwin confined his theories to the biological realm (as paragraph 5 indicates that Darwin does). C) The author writes: "Spencer was well-known as an evolutionist long before Darwin's On the Origin of Species was published, but while Darwin resisted applying the idea to morals, society, or politics, Spencer saw evolution working everywhere. He has been tagged as a social Darwinist, but it is more correct to think of Darwin as a biological Spencerian" (paragraph 5). The author, then, attributes some primacy in terms of the reach of his ideas to Spencer over Darwin (and indicates that his renown preceded that of Darwin's great work). The counter-factual scenario described in the stem, in which Spencer coined the expression "survival of the fittest," would further support the description of Darwin as a follower of Spencer, rather than the other way around. D) The passage describes the banquet Spencer attended, where the "president of Columbia University announced that Spencer was 'the most powerful intellect of all time.'" (paragraph 3). Even if Spencer, rather than Darwin, had coined the expression "survival of the fittest," however, that fact would not necessarily support this hyperbolic claim.

Suppose that some influential American critics have always expressed strong opinions about the characteristics of good art and of bad art. This supposition implies that the passage author has probably: A. correctly assessed the gullibility of the American public. B. misunderstood the comments made in the Times reviews. C. overestimated the influence of criticism on artists. D. underestimated the merits of recent American art.

The solution is C. A) If some critics have always expressed strong opinions about what makes art both good and bad, this might qualify the author's assessment that the overvaluing of art by critics has "spread to the American public" (final paragraph). It would not support the claim that the author has "correctly assessed the gullibility" of that public. B) Even if "some critics have always expressed strong opinions" about what makes art good and bad, this doesn't mean that the passage author's description of the unfailingly positive gallery reviews in "a recent Weekend section of the New York Times" (paragraph 3) is mistaken. C) Much of the passage is devoted to the argument that it is the "timidity" of critics, their hesitance to take a strong aesthetic stand and to criticize as well as praise artists, that has contributed to the "depletion" of the quality of contemporary art: "But whatever the history of our artistic recession, its immediate cause is precisely the pervasive and unchecked reverence in which art is held by the critics" (paragraph 3). If, however, some critics have indeed always expressed strong opinions about what makes art both good and bad, and the artistic "recession" has nonetheless occurred (in the author's view, in any case), then it is likely that the author is overestimating the impact of critics on the quality of artistic production. D) Whatever the critics may have said about the qualities that produce good (or bad) art, this does not have any kind of necessary effect on the actual merits of contemporary art, nor on the appropriateness of the author's judgment about this art.

For which of the following passage statements is evidence NOT provided? A. "Although this kind of intervention [changing the medical education system so that it selects, trains, and deploys more health care workers who choose to practice in rural areas] does not lend itself to controlled experiments, ample evidence exists that such an approach works" (paragraph 2). B. "[P]hysicians and others act as rational economic beings" (paragraph 3). C. "These two federal programs [community health centers and the National Health Service Corps] remain the preeminent safety net programs for rural America" (paragraph 4). D. "Currently, telemedicine is dynamic, but relatively unstructured" (paragraph 5).

The solution is C. A) Support for this statement is provided in the last two sentences of paragraph 2: "At publicly owned medical schools in rural states...virtually no rural graduates." B) The fact that "this approach has been used with some success in Britain, Canada, and Australia, where a variety of bonuses increase reimbursement for selected rural practitioners" (paragraph 3) provides support for the notion that physicians do act as rational economic beings. C) There is no evidence for this claim, no illustration of its preeminence as a safety net. D) The last sentences of paragraph 5, as well as paragraph 6, support the claim that telemedicine is "unstructured."

What aspect of the commodity musical does the author most rely on in characterizing it in the last sentence? The author views the commodity musical as: A. innovative. B. ambiguous. C. derivative. D. ambitious.

The solution is C. A) The author describes the commodity musical in the final sentence of the passage as "watered down" and "parasitical," which is the opposite of original. B) There is nothing in the last sentence to suggest the author considers the musical to have an ambiguous character. C) The author calls the commodity musical both "watered-down," and a "parasitical imitation of the real thing," which is another way of saying highly derivative. D) The author implies in "watered down" precisely the opposite of ambitious.

Which of the following of the archaeologists' assumptions described in paragraph 3 is NOT supported by evidence or reasoning in the passage? A. The mound builders had a large labor force. B. The mound builders had ample leisure time. C. The mound builders grew maize, beans, and squash. D. The mound builders included specialized craftspeople.

The solution is C. A) The author provides some support for this claim: "The great size of some of the mounds implied the existence of a large labor force..." (paragraph 3). B) The author supports this claim with some reasoning: "The great size of some of the mounds implied the existence of a large labor force with enough leisure time to devote to construction projects" (paragraph 3). C) The author provides reasoning for the assumption that the mound builders were farmers but does not provide reasoning or support for the assumption that they were "farmers who grew maize, beans, and squash" (paragraph 3). D) The author offers some reasoning to support this claim: "The work of specialist craftspeople, freed from subsistence tasks, seemed to be manifest in the fine quality of artifacts found in the mounds" (paragraph 3).

The author assumes that the diffusion of squash throughout the Woodlands region: A. occurred alongside the diffusion of pottery. B. is evidence of its hardiness. C. was not due to Mexican influence. D. explains why maize was not discovered at Poverty Point.

The solution is C. A) The author writes that "One could construct a neat diffusionary model.... Mexican emigrants brought their staple crops [including squash], pottery making, and mound building into the Woodlands" (paragraph 3). However, the author does not endorse this model. In the final paragraph of the passage, the author notes explicitly that the pottery found at Poverty Point (in the southern Woodlands region) "did not derive from Mexico," while acknowledging that "squash might have been grown there." The author, then, does not suggest that the diffusion of squash accompanied the diffusion of pottery. B) There is no discussion in the passage of the "hardiness" of particular crops, or of hardiness explaining the presence of particular crops in the Woodlands region. C) The author writes that "While squash might have been grown [in Louisiana's Poverty Point region], its presence at Poverty Point does not indicates a close relationship to Mexico because squash had diffused throughout the southeastern Woodlands region as early as 2000 BC" (final paragraph). D) The author does not suggest that the "conspicuous absence" of maize at Poverty Point was due to the possible concurrent presence of squash there.

Passage context suggests that when Spencer came to the United States in 1882, he probably planned to: A. write a book about his experiences. B. introduce modifications to his theory. C. spend much of his time alone. D. leave again within a short time.

The solution is C. A) The author writes that when Spencer came to the U.S. in 1882, he did so to "reinvigorate his 'greatly disordered nervous system,'" and also points out that he "no longer needed money, thanks to the immense popularity of his books" (paragraph 2). There is no indication that he planned to write a book about his experiences in the U.S. B) The passage suggests only that Spencer came to the U.S. for a rest, and it does not indicate that he expects to modify or alter the theories that had already brought him great renown. C) The author writes that Spencer "had come to the U.S. to reinvigorate his 'greatly disordered nervous system,' and he withstood all inducements to what he called 'social excitement'" (paragraph 2). This clearly suggests that Spencer's intention was to find solitude in the U.S. D) There is nothing in the passage that indicates the length of the visit to the U.S. that Spencer had planned.

Which of the following statements best conveys the main point of the passage? A. The Neandertals did not actually disappear. B. The Neandertals invented jewelry and hunting. C. The emerging view depicts Neandertals as having a capacity for creative, flexible behavior. D. Most researchers credit Neandertals with being well-adapted creatures who survived the cold climate of Europe.

The solution is C. A) The final sentence of the passage makes clear that this not the author's main argument. B) There are different ideas advanced in the passage about the source of Neandertal jewelry and to what extent they created it themselves (paragraph 2), and there is no suggestion that they invented hunting. C) Much of the passage focuses on challenging the portrayal of Neandertals as "lacking the language skills, foresight, creativity, and other cognitive abilities of modern humans" (first paragraph). Paragraphs 2 through 4, in particular, describe the complexity and flexibility of the Neandertals' achievements. D) The "well-adapted" part may be correct, but there is nothing in the passage about climate. And this would be too narrow a claim to account for the passage emphasis on the cognitive capacities of the Neandertals.

Which of the following policy suggestions demonstrates acceptance of one or more of Talley's claims, as they are described in the passage? A. An increase in the number of total slots available at urban medical schools B. Classification of all family medicine as primary care medicine C. Travel subsidies for urban physicians interested in providing health care to rural Americans D. Tuition subsidies for people from rural areas who apply to medical schools

The solution is D. A) Talley is not concerned about the number of slots available in urban medical schools. B) None of Talley's points is affected by this change. C) None of Talley's points is affected by this change; Talley does not claim, nor imply, that travel costs are a barrier for urban physicians who would want to provide health care to rural residents. D) Talley's first point, that "students with rural origins are more likely to train in primary care and return to rural areas," suggests that if there were more students from rural areas in medical school, there would ultimately be more health care professionals in rural areas. An increase in medical students from rural areas would likely result if tuition subsidies were offered to them.

Which of the following findings would most weaken the Mexican hypothesis as presented in the passage? A. An abundance of leisure time is not correlated with an abundance of fine-quality artifacts. B. Pottery from ancient Mexico is stylistically similar to Native American pottery found in the Woodlands. C. There is evidence that emigration took place about 1200 BC from the area that is now Mexico to western areas of the U.S. D. Leisure time tends to decrease when a people switches to farming from hunting and gathering.

The solution is D. A) The absence of this correlation would merely challenge the assumption that the presence of such artifacts implies leisure time (the passage indicates that specialized craftspeople were "freed from subsistence tasks" (paragraph 3). That the presence of large mounds implies leisure time, and therefore agriculture—a crucial building block in the "Mexican hypothesis"—would still stand. B) Stylistic similarities between Mexican pottery and Native American pottery does not affect the Mexican hypothesis in one way or another. While such similarities might be interpreted as support for a Mexican influence on Native American culture (and thus could support the Mexican hypothesis), it is also possible that Native American pottery styles could have influenced Mexican styles. C) There is no discussion in the passage of mound-building in the western areas of the U.S., so such evidence is irrelevant. One might speculate that people who emigrated to western areas of the U.S. might then have moved to the eastern areas, but that would support—rather than weaken—the Mexican hypothesis. D) The Mexican hypothesis rests on a series of assumptions about the conditions that would have enabled the construction of the large mounds found in the Ohio and Mississippi River valleys. The author writes that "The great size of some mounds implied the existence of a large labor force with enough leisure time to devote to construction projects. The work of specialist craftspeople, freed from subsistence tasks, seemed to be manifest in the fine quality of the artifacts found in the mounds. Leisure time and craft specialization were inconceivable except as by-products of agriculture, and so archaeologists assumed that the mound builders were farmers who grew maize, beans, and squash. These staple crops were, of course, of Mexican origin" (paragraph 3). If, however, leisure time decreases with the advent of agriculture, then it would no longer follow that Mexican farmers were likely to have had the time to devote to the building of the mounds, which would of course weaken the hypothesis.

The author's attitude toward commodity musicals is one of: A. enthusiastic endorsement. B. grudging admiration. C. contingent disappointment. D. strong disapproval.

The solution is D. A) The author calls commodity musicals "inoffensively entertaining at best, numbingly tedious at worst" (paragraph 2); they are "a watered-down, parasitical imitation of the real thing" (final paragraph). B) The author expresses no admiration for the commodity musical form, grudging or otherwise. C) There is nothing contingent about the author's disapproval of the commodity musical. D) The author calls commodity musicals "inoffensively entertaining at best, numbingly tedious at worst" (paragraph 2). Further, the author complains, they are "a watered-down, parasitical imitation of the real thing" (final paragraph). Thus, the author clearly expresses "strong disapproval."

Based on the first paragraph, which of the following statements best summarizes Stendhal's view of fiction? A. Fiction should reveal the author's inner feelings. B. Fiction should reorder life's events in a pleasing manner. C. Fiction should illustrate life's important ideas. D. Fiction should strive to duplicate life's events.

The solution is D. A) The author describes Stendhal's view of fiction as "the mirror in the roadway" and explains, "One mirror is like another, a mechanical reflector" (first paragraph). This suggests that Stendhal believed that that fiction should be as "objective," or impersonal as possible. B) Because Stendhal viewed fiction as functioning as "the mirror in the roadway," it should seek to reflect reality as accurately as possible, rather than attempting to "reorder" or transform life to be more attractive or appealing. C) The author claims that Stendhal viewed fiction as ideally functioning as a "mirror in the roadway...a mechanical reflector" (first paragraph). This would suggest that Stendhal believed that fiction should record all of reality as it occurs, rather than representing "important ideas" in a selective fashion. D) This option is consistent with the way that the author represents Stendhal's view of fiction. The author writes that Stendhal saw fiction as "the mirror in the roadway" (first paragraph) and comments that "one mirror is like another, a mechanical reflector." This view certainly implies that fiction should seek to duplicate life's events.

What could be interpreted as a paradoxical aspect of Spencer's behavior was his: A. obvious affluence, despite an explicit disdain for wealth. B. fervent belief in capitalism, despite evolutionary evidence against its merits. C. apparent sense of inferiority, despite glowing praise from his followers. D. voluntary public appearance, despite an aversion to social excitement.

The solution is D. A) The author does not indicate that Spencer has any sort of "disdain for wealth." Rather, the author indicates that Spencer "admired [the] material achievements" of American capitalism (paragraph 4). B) The passage does not suggest that "evolutionary evidence" discredits capitalism. It links evolutionary theory and capitalism only by indicating that Spencer was an enthusiast of both. C) The passage does not indicate that Spencer has any kind of "sense of inferiority." It indicates that he "detested public speaking" (paragraph 2) and also that he did not "graciously accept [the] tributes" that were given to him (paragraph 4), but the passage indicates that this was because he wanted to correct a mistaken understanding of his evaluation of the American attitude towards work (paragraph 4), not because he felt inadequate to receive these tributes. D) The passage says of Spencer that he "detested public speaking"; moreover, "he had come to the U.S. to reinvigorate his 'greatly disordered nervous system,' and he withstood all inducements to what he called 'social excitement'" (paragraph 2). Nonetheless, and without any need for money, he chose to attend a banquet in his honor.

Based on the passage, the author feels that his readers are generally: A. very understanding of the mistakes he makes in his work. B. unconcerned with how they are being manipulated by him. C. confused by the way that he imposes order on chaos. D. quick to assume that he is writing from his own experiences.

The solution is D. A) The author implies that he may well make "mistakes" in his work; he points out that his writing "may reveal optical opacities or incapacities of which I am totally unaware" (paragraph 2). He does not, however, suggest that his readers are understanding (or intolerant, for that matter) of any mistakes he might have made. B) The author does suggest that writers may well "manipulate" their readers: "Writers are far more cunning than the credulous reader supposes. We are all practiced shape-shifters and ventriloquists" (paragraph 3). The author does not, however, imply that readers are unconcerned with any such manipulation; the passage indicates that the author is more likely to view his readers as unaware of manipulation: "No novel...can be read in so naively literal a fashion [as to be seen as transparent autobiography], though it is evident from my mail that people do it all the time" (paragraph 2). C) The author refers to authorial work: "There is more than one way to impose order on your personal chaos" (paragraph 4). The author does not, however, suggest that his readers are "generally confused" by his imposition of order on his own chaos. D) The author describes the viewpoint that sees fiction as a collection of "personal confessions...and [the belief] that when an 'I' character speaks in one of my novels or stories he is to be read as me, sneakily expressing my attitudes and feelings from behind a mask" (paragraph 2). The author then dismisses this view as "irritating nonsense. No novel, even one meant to be autobiographical, can be read in so naively literal a fashion, though it is evident from my mail that people do it all the time" (paragraph 3). The mail is apparently mail from readers, whom the author indicates are reading his work as if the characters in his novels are simply the author himself.

Which of the following comparisons is consistent with the opinions expressed by the author of this passage? A. Shakespeare was a much better writer than Hemingway. B. Faulkner's fiction is much more personal than Hemingway's fiction. C. Hemingway's fiction has a wide variety of characters. D. Hemingway's fiction is more clearly personal than Faulkner's fiction.

The solution is D. A) The author says that Shakespeare was possessed of more "negative capability, the capacity to make ourselves at home in other skins" than was Hemingway. The latter was among those "writers [who] have, or use, that capacity hardly at all" (paragraph 3). But the author does not suggest that the extent to which a writer has that capability determines how "good" a writer he or she is: "there is more than one way to impose order on your personal chaos," the author writes in paragraph 4. B) The author clearly takes the opposite position: "You could no more reconstruct William Faulkner's life or personality from his fiction than you could reconstruct Shakespeare from his plays. On the other hand,...[t]he fictions of Hemingway [and others] are haunted by recurring characters that look like clones of their authors" (paragraph 3). C) To the contrary, the author suggests that Hemingway's work contains a host of similar characters, as his work is "haunted by recurring characters that look like clones of their author" (paragraph 3). Further, this option does not describe a "comparison." D) The author writes: "The fictions of Hemingway, Thomas Wolfe, and Katherine Anne Porter, for three, are haunted by recurring characters that look like clones of their authors." On the other hand, the author tells us, Faulkner's work was replete with "negative capability, the capacity to make ourselves at home in other skins.... You could no more reconstruct Faulkner's life or personality from his fiction than you could reconstruct Shakespeare from his plays" (paragraph 3).

According to the passage, the key distinction between what most people think of as a small-world experience and Watt's mathematical model of small worlds is that the mathematical model: A. limits the number of possible targets. B. relies on the cooperation of participants. C. cannot take sociable people into account. D. selects targets randomly.

The solution is D. A) The passage does not indicate that a limitation on the number of possible targets is a difference between "what most people mean by a small-world experience" (final paragraph) and Watts's mathematical model of a small-world experience. B) Milgram's experiment relied on the cooperation of participants, but the description of the difference between Watts's mathematical model of a small world and most people's understanding of a small world is not that Watts's model relies on participant cooperation. C) Watts's mathematical model does take sociable people into account, as described in paragraph 6: Watts's "mathematical models...show how a small world could work. Random connectors in a network, such as especially sociable people who have friends across subcultures, can vastly decrease the distance between points in a network" (paragraph 6). D) "There is a...difference between what most people mean by a small-world experience and what mathematicians mean. The chances of meeting a person who knows someone from one's past are not the same as the chances of connection between two people taken at random" (paragraph 8).

According to the passage, access to health care in the United States depends on which of the following? I. The individual's demonstrated need for health care II. Whether the individual has access to health insurance III. The proximity of healthcare professionals to the individual A. I only B. II only C. I and III only D. II and III only

The solution is D. A) The passage does not mention need as a factor determining access to health care. B) The passage supports access to health insurance as a factor in health care access, but it also supports proximity of health care providers to the individual as a factor in health care access. C) Although the passage supports the proximity of health care providers to the individual as a factor in determining access to health care, it does not mention need as a factor in health care access. D) The author writes: "This problem [having limited access to health care] stems from two aspects of the U.S. healthcare system: the many Americans without health insurance and the tendency of healthcare professionals to locate and practice in relatively affluent urban and suburban areas."

How would the passage argument be affected if it were demonstrated that most Americans are interested in the fine art now being produced? A. Its analogy between art and a recession would be false. B. Its wish for real critical debate would be encouraged. C. Its discouraging conclusion would not be justified. D. Its disparaging remarks would apply to more people.

The solution is D. A) The proportion of Americans interested in the fine art now being produced is irrelevant to the applicability or "truth" of the author's analogy between the state of art and the state of the economy (paragraph 2). B) The author does not suggest that the cause of a lack of real critical debate about art is a lack of people interested in art. Rather, the author says that the problem is critical "timidity," a fear of passing negative judgment on art that is later received as great art. C) To the contrary, the author's negative conclusion applies precisely to those "fifteen million who take an interest in art" (final paragraph). If most Americans were interested in the fine art now being produced, this would merely extend the reach of the author's discouraging conclusion that art lovers, in particular, are afraid to make critical judgments, as they live in fear of failing to understand the work of an artist who will later be viewed as a genius of some kind. D) The author writes in the final paragraph, "This attitude [fearing to make a negative judgment about art that could later be assessed as 'great'] has spread to the American public, at least those fifteen million who take an interest in art.... As a result, the more fervid their interest in art, the less confident they are of their ability to view it with the necessary discernment" (final paragraph). If there were more Americans interested in art, this opinion would simply apply more widely, because the author suggests it applies to all who are interested in art. The author does say that those who are interested in art want to avoid being part of the "philistine majority" who believe that any young child could "paint a Picasso" (final paragraph). Were "most" Americans interested in art, this concern with being part of the "majority" would be less salient, but an interest in belonging to a "select" group is ultimately less central to the author's argument than is the general fear among those interested in art of overlooking genius.

The setting of the Broadway musical West Side Story, an adaptation of Shakespeare's play Romeo and Juliet, is drastically different from the Shakespearean play. What is the relevance of this to the passage? A. It supports a claim about commodity musicals. B. It challenges a claim about Broadway musicals and adaptations. C. It challenges a claim about the titles of commodity musicals. D. It supports a claim about Broadway musicals that are not commodity musicals.

The solution is D. A) There is no claim about commodity musicals that is supported by this information. B) There is no claim about Broadway musicals and adaptations that is challenged by this information. C) There is no claim about the titles of commodity musicals that is challenged by this information. D) The author writes that "These [non-commodity Broadway] musicals were usually given brand-new titles, to emphasize their originality" (first paragraph). They were not simply attempts to replicate or recall a commercially successful predecessor.

What result would be most predictable if conditions analogous to those described in the passage prevailed in the automobile industry? A. Manufacturers would try to hide any problem associated with a model. B. Consumers would rely on personal experience to compare the models. C. Consumer advocates would report the defects of unreliable models. D. Automobile ratings would emphasize the advantages of each model.

The solution is D. A) There is no implication in the passage that artists try to "hide" any "problems" with their art. B) There is no evidence that the author believes that members of the art-viewing public rely on their own personal experience to assess art, so there is no reason to believe that if analogous conditions to those described in the passage prevailed in the automobile industry, that automobile consumers would rely on their own personal experiences. C) The author argues that critics—analogous to "consumer advocates"—have precisely failed to report the "defects" in contemporary art, instead offering almost uniformly positive reviews. D) The author complains: "The most striking feature of recent artistic discourse, the quality that sets it apart from all earlier discussion, is its chronic unwillingness to address the weaknesses inherent in the productions of our artists" (paragraph 3). The author goes on to decry the relentless positivity in reviews of ten art gallery shows. If similar conditions prevailed in the automobile industry, positive ratings of cars would be the norm.

Which of the following is most like the current situation on Broadway, as it is described in the passage? A. Popular vocalists are more successful when they incorporate dance moves into their performances. B. Art museum curators prefer collections that are positive or inspiring in some way. C. Most comedians perform in small, little-known venues before moving into larger, more prestigious venues. D. Concert programmers put only one innovative work into an evening's program of otherwise familiar classical pieces.

The solution is D. A) There is no indication that Broadway musicals (which typically feature music and dance) are more successful than shows that feature only one or the other, so this option would not provide a good analogy to the current situation on Broadway, as described in the passage. B) Although the author refers to the "old-fashioned Broadway musical" as including "optimism" (final paragraph), those are not the ones that the passage describes as successful today. C) There is mention of "off Broadway" and "theaters outside New York" which may (or may not) be smaller venues than those on Broadway, but there is no indication that performers play on the smaller before the larger. D) The author indicates that original shows simply cannot flourish on Broadway because audiences want what they are already familiar with, including "the presence of a movie or television star" (paragraph 6). The author also indicates that it is difficult for songwriters to work on Broadway "without conforming to the restrictive requirements" of the commodity musical (paragraph 5).

Assume that a study is performed in which only the physiological aspects of emotion are chemically suppressed and that the participants are then placed in a situation that would normally anger them. The passage author would presumably expect the participants to experience: A. an abnormally intense disapproval of the situation. B. an apparently normal response to the situation. C. indifference and inattention to the situation. D. a calm negative assessment of the situation

The solution is D. A) Without the physiological dimension of anger, as described in paragraph 2, one would experience anger less intensely, rather than more intensely, than if it were present. B) The author contends that there are "at least two components" of emotion: "measurable physiological events and a belief or thought about some person or situation" (paragraph 4). With the measurable physiological dimension of emotion withdrawn, participants in the described experiment would not have a "normal" response to the situation—the physiological dimension of their anger, as described in paragraph 2, would be absent. C) The author indicates that "a belief or thought about some person or situation" (paragraph 4) is one component of emotion. If this component is present, then "indifference" or "inattention"—a lack of emotion—is unlikely to result, based on the author's model. D) The author contends that, "emotion requires at least two components: measurable physiological events and a belief or thought about some person or situation, real or imagined" (paragraph 4). If the physiological element of the emotion were withdrawn from the experience, but an event likely to incite anger was still present, then the participants would be without the "agitation" the author describes as associated with anger in paragraph 2. Participants would still assess the situation negatively, though, as the "belief or thought about some person or situation" would still be present (paragraph 4).


Conjuntos de estudio relacionados

Prep-U: Chapter 47: Management of Patients With Intestinal and Rectal Disorders

View Set

DOCUMENTATION AND REPORTING NCLEX

View Set

cholinergic neurotransmitters and receptors

View Set

Chapter 16: Nutrition and Fitness

View Set

304 EAQ Alterations in Glucose Regulation

View Set